You are on page 1of 80

‫ﻣﺼﺪر اﻻﺳﺎﻟﺔ ﺟﻠﻮري ﺗﯿﻢ ﻓﻘﻂ ‪ ،‬ھﺬا اﻟﻤﻠﻒ اﻟﺠﺮاﺣﺔ ‪ surgery‬ﺷﺎﻣﻞ ﻣﻦ ﺳﯿﺒﺘﻤﺒﺮ اﻟﻰ ﯾﻮم واﺣﺪ دﯾﺴﻤﺒﺮ ‪ ،‬ﻧﺴﺎل

ﷲ اﻟﺘﻮﻓﯿﻖ وان‬
‫ﯾﻨﻔﻊ ﺑﮫ ‪.‬‬
‫ﺳﺒﺤﺎن ﷲ واﻟﺤﻤﺪ ‪ R‬وﻻ اﻟﮫ اﻻ ﷲ وﷲ اﻛﺒﺮ وﻻ ﺣﻮل وﻻﻗﻮة اﻻ ﺑﺎ‪R‬‬
‫ﺑﺴﻢ ﷲ اﻟﺮﺣﻤﻦ اﻟﺮﺣﯿﻢ‬
1Breast tumor
🌹Birad 3 =Follow up for short time. F = 3
🌹Birard 4 = core biopsy 🌹 core ‫ار< ـﻊ ﺣﺮوف‬

🌷- female with ULQ lump increasing with time, size 15x15 cm, us show solid mass
with multiple cystic spaces with posterior costic enhancement, management?
a. neoadjuvant chemotherapy b. simple mastectomy✅✅ (case of phylloid tumor)

🌹 Female in her 47 , single, positive family history of breast cancer.


Underwent routine mammography which showed bilateral increased density
and glandular pattern. Core needle biopsy showed atypical ductal hyperplasia.
What’s the appropriate management? A. wire surgical excision. B. Simple mastectomy

🌹 Intraductal hyperplasia in patients with breast cancer family history:


A. simple mastectomyB. wire excision C. radical mastectomy D. preventive chemo

🌷 ULQ lump increasing with time, size 15x15 cm, us show solid mass
with multiple cystic spaces with posterior costic enhancement== (PHYLLOID TUMOR) ==
simple mastectomy🌹
....
🌹patient after modified radical mastectomy complaining of loss of sensation in inner side
of right arm, injury to which nerve is suspected:
A.Long thoracic B. Inter-Costo-Bracial nerve ✅
Inter = inner Costo = mastect Bracial = arm
..........

🌷lactating came with mastitis with abcsess 5 cm tx?


A. I&D ( for abcsess)✅. B. Mastectomy. C. needle aspiration. D. Antibiotic
🌷mastitis with abcsess treat by = incistion and drainge🌷

🌹Acute mastitis organism =staph aureus🌹

🌷mastitis alone treat by = Antibiotic🌷

🌹Lactaional mastitis case = oxacillin with continu breast feeding 🌹

🌹Lady 26 Y with lift breast redness around areola, mass in the right outer quadrant
tender and red , there is lymph node enlargement,temp high, dx
A. inflammation cancer B. abscess C. fibroadenoma

🌹young woman has painful breast lump with redness and tenderness. vitals are given
and show T 38.9 what will u do? A. incision and drainage. B. excision C. FNA

………..
🌹40 years old lady underwent mamogram and US Mamo showed fibroglandula tissue US
shows multiple cysts, BIRAD 3 What to do?
- core bx. -exision bx. -annual screening -short duration follow up✅
🌹What is the meaning of Birads 3?
BIRADS 3 often indicates the need for a 6-month follow-up mammogra

🌹 BIRAD 4, next step in management :? core tissue biopsy. 4= CORE ‫ارﺑﻊ ﺣروف ﻣﻊ ارﺑﻌﺔ‬

🌹 40 y/o female patient presented with breast mass and tethering, mammogram and U/s
were done and showed BIRADS IV, your next step == Core biopsy
…….
🌹25 y/o female with Breast mass 2*2 describe as Oval shape painful:
A. Duct ectasia B. breast cyst C. breast cancer

🌹Young female complaining of breast tenderness associated with menses, US done left
breast showing 3 cysts right breast 2 oval masses, diagnosis? - Fibrocystic change

🌹Young patient with breast lump on exam: Oval with smooth surface 2*2cm dx
A-fibroadenoma✅ B-cyst C-breast cancer

🍓20 years old female , presenting with recent mass in right breast which is mobile =
fibroadenoma
‫ ﻟﮫ ﺛﻼث ﻋﻼﻣﺎت ﻣﮭﻣﺔ أوﻻ ﺗرﺗﺑط ﺑﺎﻟﮭرﻣون‬fibroadenoma ‫ ھو ان‬fibrocyst ‫ و‬fibroadenoma ‫اﻟﻔرق ﺑﯾن‬
‫ ﻟذﻟك ﻏﺎﻟﺑﺎ ﯾﺟﻲ ﻓﻲ وﻗت‬، ‫وﺧﺎﺻﺔ ﺑﺎﻻﺳﺗروﺟﯾن ﯾﻌﻧﻲ ﻛل ﻣﺎزاد اﻻﺳﺗروﺟﯾن ﯾزﯾد ﺣﺟﻣﮭﺎ وﻛﻠﻣﺎ ﻗل ﯾﻘل ﺣﺟﻣﮫ‬
‫ ﺛﺎﻧﻲ ﺷﻲء اﻧﮭﺎ‬، menstrual cycle ‫ او ﻗﺑﻠﮭﺎ ب أﺳﺑوع وﺗﺧﺗﻔﻲ او ﯾﻘل ﺣﺟﻣﮭﺎ ﺑﻌد‬menstrual cycle
‫ اذا ﺿﻐطت ﻋﻠﯾﮭﺎ ﺗﺣس ﺑﺎﻟم وﻣﺎﻟﮫ أي ﻋﻼﻗﺔ ﺑﺎﻟﮭرﻣوﻧﺎت‬cyst ‫ ﻣﺎﻓﻲ أي اﻟم ﻟﻣﺎ ﺗﺿﻐط ﻋﻠﯾﮭﺎ ﻋﻛس‬painless
. ‫ ﺛﺎﺑت ﻣن اﺳﻣﮭﺎ ﻛﯾس = ﺗﺎﺑت ﻣﺎ ﺗﺗﺣرك‬cyst ‫ اﻟﻛﺗﻠﺔ ﺗﺗﺣرك ﻟﻛن‬adenoma ‫ ﻣن اﺳﻣﮭﺎ‬mobile ‫ﺛﺎﻟﺛﺎ‬
firm or smooth ‫ ﯾﻛون ﺳطﺣﮭﺎ‬adenoma ‫ال‬

🌹 50 y women no family Hx of breast ca , mammogram normal When to do next


mammogram ? A. after 1 year B. 2 y C. 3 y D. 5 y

🌹40 years Pt with hx of bloody discharge from breast ? P/E normal Initial invest
A. US B. Mammogram C. MRI

🌹32year old complain of left breast mass with bloody discharge .. on examination the left
breast and axilla is normal what will do next A. Bilateral US B. FNA C. Bilat mammography
‫ﻻن ﻓﯾﮫ دم‬
🌹 37 y/o Female with family hx of breast cancer complain of breast mass , She was concern
of breast cancer by ex of there was a breast mass with skin tethering Which of the
following is the most important next step ?
A. Bilateral Mammo ✅✅ B. Breast MRI C. Biopsy D. US
🌹 35 year old pregnant came with bilateral breast tenderness and mass what
investigation you’ll do? A. US B. mammography C. reassess after delivery
us ‫ وﻣﺎﻓﻲ دم وﻋﻣرھﺎ اﻗل ﻣن أرﺑﻌﯾن ﻓﻧﺧﺗﺎر‬mass ‫ﻓﻘط‬
🌹Pregnant patient with breast mass. What the best investigations? Bilateral breast US

🌹breast mass behind nipple, on US there is hypoecoich lesion, what next :


A- FNA ✅✅ B- core biopsy C- exicional biopsy D- reassess after ...

Pregnant with increasing breast mass 3 to 4 cm what to do?


A-Bilateral ultrasound ✅
B-Bilateral memo
C-Delay after pregnancy
D-Reassurance
🌹increasing breast mass 3 to 4 cm =Bilateral ultrasound🌹
.....
🍒Breast screen how frequent = Every 2 yrs 🍒
🌹breast mass, periareolar, nipple pulled inside, unilateral what next step:
first breast imaging, then biopsy, then staging 🌹
.....
37 y/o Female with family hx of breast cancer complain of breast mass , She was concern of
breast cancer by ex of there was a breast mass with skin tethering Which of the following is
the most important next step ?
A. Bilateral Mammo ✅✅
B. Breast MRI
C. Biopsy
D. US
..........
🍓Female pt going for breast examination but she wants female doctor to examine her
what you will do:
A. refuse
B. respect ✅
.....
..........
🍓breast pain pre menses by 3 day Px no rmal next = reassurance.
...........
🍓strong family hx of breast come with identified breast mass what next
Mammogram
.......
🌹strongest risk factor of breast ca in female : age
-young woman has painful breast lump with redness and tenderness. Vitals are
given and show T 38.9. What will u do?
a. incision and drainage✅
b. excision c. FNA

🌹A 50 year old lady presents with bloody nipple discharge, and it was found to be invasive
intraductal papilloma. The most appropriate management is= Wide local incistion.
🌹Patient with bloody stained nipple discharge, you’re suspecting intraductal papilloma.
What’s the next step? Excision ✅
....
🌹Color of discharge in intraductal papilloma ? A. Red B. Green C. Black
🌹duct ectasia, color of discharge?
A. red
B. blue
C. green ✅
Duct ectasia :Green 💚
Ductal papilloma : red ❤
🌷 breast biopsy shows intraductal papilloma what will u do
a. breast preserving✅(Surgical excision of involved duct to ensure no atypia with
breast preserving) B_radical mastectomy‫ﺧطﺎ‬

🌹breast biopsy shows intraductal papilloma what will u do A. breast preserving B. radical
mastectomy

*Breast Mass:-
After history and clinical examination.
🌷Breast Mass If patient less than 30= *Ultrasound*🌷
🌷Breast mass patient 30 or older ? *Mammogram*🌷
‫ ﺑﻌد ﻛذا اذا ﻟﻘﯾت ﺑﻧﺗﯾﺟﺔ‬FNA ‫ ھﻧﺎ ﻧﺳﺗﻌﻣل‬cyst ‫ ﻓﻲ اﻟﺗراﺳﺎوﻧد وﺷﻛﯾت اﻧﮫ‬mass ‫ﯾﻌﻧﻲ اذا اﻧﺎ ﺑﺎﻟﻌﯾﺎدة ﻓﺣﺻت وﻟﻘﯾت‬
cyst ‫ ﻛﺎن ﻧظﯾف وﺷﻔﺎف وﻣﺎﻓﻲ دم ﺧﻼص اطﻣن اﻟﻣرﯾض واﺗﺎﺑﻌﮭﺎ ﺑﯾﻧﻣﺎ ﻟو طﻠﻊ‬cyst ‫اﻟﻌﯾﻧﺔ اﻧﮫ اﻟﺳﺎﺋل اﻟﻲ طﻠﻊ ﻣن‬
‫ اذا طﻠﻊ‬core biopsy ‫ أﯾﺿﺎ ﻣﺗﻰ اﺳﺗﺧدم ال‬، core biopsy ‫ﻓﯾﮫ دم ھﻧﺎ ﻻزم اﺧذ ﻋﯾﻧﮫ ﺛﺎﻧﯾﺔ ﻣن اﻻﻧﺳﺟﺔ اﺳﻣﮭﺎ‬
. ‫ ﻓﯾﮫ ﻛﺗﻠﺔ واﺿﺣﺔ‬mammogram ‫ﺑﻧﺗﯾﺟﺔ ال‬
🌷Clinically suspicious Breast Mass =Fine needle aspirantion .
🌷 breast mass on mammogram = Core needle biopsy🌷
*Cystic on Ultrasound or FNA results:-*
-if cyst aspirantion show Clear fluids *Reassure & follow*
-if cyst aspirantion show Residual mass or Bloody= *Core needle biopsy*
*Solid on ultrasound results:-*
Likely benign? *Treat as needed*. -suspicious? *Core needle biopsy*
...... .... ....

🌹Management of atypical ductal hyperplasia?


A. Wide local excision B. Simple mastectomy
‫اﻟﻲ ﻓﯿﮭﺎ ﺳﺮطﺎن واﻻﻧﺴﺠﺔ اﻟﻲ ﺣﻮﻟﮭﺎ اﻟﺴﻠﯿﻤﺔ‬ Duct ‫ ﻓﺎﻟﻐﻼج ﻧﺸﯿﻞ ھﺬه‬Duct ‫ﻗﺎﻟﻚ ورم ﻓﻲ‬
32 yo I think female concerned about breast cancer, because her mother
have it and her sister has ovarian cancer. What to do?
A. Mammogram B. US C. BRACA

15*15 mass in the breast getting larger and the skin overlying is intake and thin
due to mass effect what to do
A. Chemo. B. Radio. C. Radical mast. D. Simple mastectomy

1 phylloid tumor
Phelloid Tx:
Ÿ If he say in the case Benign > WLE
Ÿ If he say in the Case milgnancy>Simple mastectomy
Ÿ If he say Suspescion > You should start Mx as it Benign By WLE

🍒Benign breast phyllodes tumor = Wide local excision🍒 ‫ﻓﯿﻞ رﺟﻠﮫ واﺳﻌﮫ‬
. . .. . . . .
🌹4-5 cm malignant phyllodes mx? A.simple mastectomy B. wide local excision 🌹✅

🍒Malignancy phyllodes tumor = Simple mastectomy 🍒‫ﻻن ﺧﺒﯿﺚ ﻻزم اﺳﺘﺌﺼﺎل‬


...... . . . .. .
🌹years lady with breast lump 2*2 i think for year suddenly the mass increased in size?
phylloids
….
🌹Case showed cystophylloides breast treatment=
a. wide local excision✅ b. mastectomy c. follow up after 6 months
.....
🌹Lady breast mass 2 years ago but 6 month ago started to get bigger, mass well defined
mobile not attached diagnosis = A) fibroadenoma B) phyllode✅
….
with painful breast mass since 6months. It get increased.it is in the outer upper surface
(around 9 clock) Histopathology confirmed a benign What is the best next step?
A. simple mastectomy B. Wide local excision
......
🌹A 24 year old lady presents with a hard, mobile, well-circumscribed painless left breast
mass that has been increasing in size from the past few months, and was NOT related to
her menstrual cycle. The most like Dx is A. fat cyst B. Fibroadenoma C.Phylloid
……
🌹Pt with malignant phylloid tumor what is next? A.Contrast ct of chest B.pet scan

🌹women with 11*12 breast mass, examination showed no palpable LN. Core
biopsy was taken and showed malignant phylloid tumor, what’s the next appropriate
step? A.WLE B.PET scan C.Chest CT witout contrast
1pancreatitis
🌹typical pancreatitis: epigastric pain, high amylase what is next step? A. US B. CT scan

🌹Pancreatitis case borderline to high amylase + k/c of gallstone what you will do next? A. US B. CT scan

🌹patient who presented w/ signs and symptoms of cholangitis of biliary pancreatitis, q was "What is the
most important initial mx"? A. U/S abdomen B. CT abdomen C. Urgent ERCP D. IV fluids

🌹Scenario of abd pain post meal with a hx of pancreatitis month ago Fluid collection found what is it?
A. Cyst. B. Pseudo. C. Hematoma

🌹Pancreatitis 5 weeks ago. Now she has epigastric tenderness and cannot tolerate food with
vomiting each time. By ultrasound you found large about 12X10 mass With thick wall and fluid
inside. Labs: 346 amylase, Wbc 15k. What is the diagnosis?*
A. Pseudocyst B. Abscess C. Walled off necrosis

🌹A Case of chronic pancreatitis what you will find?!@


A. Increase insulin resistance B. Decrease gluconeogenesis C. increase lipolysis ‫اﺻﺣﮭم‬
D. Hypoglycemia ( Answer is : hyperglycemia, hypocalcemia)

🌹40 years old woman with no pain but you noticed jaundice. She has high direct
bilirubin and high ALT what is your diagnosis? Carcinoma in head of pancreas ✅

🌹 pt with pulstile epigastric mass and ask about investigation :
Serum k, Serum amylase ✅. Serum cl
Abdominal aortic anyurysm AAA ‫طﯾب ﻋﺷﺎن ﻧﻔﮭم اﻟﺳؤوال اوﻻ ﺗﺷﺧﯾص اﻟﺣﺎﻟﺔ ھو‬
: Ischemic pancreatitis ‫ ھو‬AAA ‫ طﯾب ﻣن ﻣﺿﺎﻋﻔﺎت اﻟﻲ ﺗﻌﻣﻠﮭﺎ‬epigastric ‫ ﻓﻲ ﻣﻧطﻘﺔ‬mass pulstile ‫ﻻن ﻗﺎل‬
.‫ وھو اﻧزﯾم ﯾرﺗﻔﻊ ﻓﻲ ﺣﺎﻟﺔ ﺣدوث ﻣﺷﻛﻠﺔ ﻓﻲ اﻟﺑﻧﻛرﯾﺎس‬amylase . ‫وﺑﺎﻟﺗﺎﻟﻲ ﻧﺧﺗﺎر‬

🍓patient with epigastric pain physical exam showed diffuse guarding and sluggish bowl
sound amylase was hight what is the next step:
erect chest xray✅. abdominal xray CT abdomen

🌹 case of acute pancreatities : Iv fluids and analgesic Abx

🌹 Diagnosed case of acute pancreatitis. Received IVF + analgesic Next step?


- Urgent surgical consult'tion✅✅ - Antibiotics
......
🌹 30 y/o Cystic fibrosis with GI Symptoms? what complication he might have?
A- *Acute* pancreatitis ✅ ✔ B-Chronic Pancreatitis C- Pancreatic cancer D- Stricture
‫ (ﻛﯾف ﯾﻌﻧﻲ ؟ ﯾﻌﻧﻲ اي ﻣﺧﺎط ﺑﺎﻟﺟﺳم او اﻧزﯾم ﺳﺎﺋل ﻣﻊ‬mucus) ‫ اوﻻ ھو ﻣرض وراﺛﻲ ﯾﺎﺛر ﻋﻠﻰ‬، ‫ ﺑﺷﻛل ﻣﺑﺳط ﺟدا‬Cystic fibrosis ‫ﺑﺣﺎول اﺷرح اﯾش ھو‬
‫ اﻟﻣﺧﺎط ﯾﻛون ﻟزج وﯾﻘدر ﯾطﻠﻊ ﺑﺳﮭوﻟﺔ ﻟﯾﮫ ؟ ﺑﺳﺑب ﻓﯾﮫ‬، ‫ ﻟﻠﻣﺧﺎط‬water ‫ ﻻن ﻓﯾﮫ ﺧﻠل ﺟﯾﻧﻲ ﻣﺎﯾدﺧل‬، ‫ ﻟﯾﮫ ﺛﻘﯾل‬، ‫ھذا اﻟﻣرض ﯾﻛون ھذا اﻟﻣﺧﺎط ﺛﻘﯾل ﺛﻘﯾل ﺟدا‬
‫ ﻓﺑﺎﻟﺗﻠﻠﻲ ﯾﻛون اﻟﻣﺧﺎك ﺛﻘﯾﯾﯾل ﻟدرﺟﺔ ﯾﻌﻣل‬، ‫ ﻟﻛن ﻣﻊ ھذا اﻟﻣرض ﻣﺎﻓﻲ ﻣﺎء ﯾدﺧل وﯾﺧﻠﯾﮫ ﻟزج ﯾﺣﺻل ﻓﯾﮫ ﺑﻠوك‬، ‫ ﯾدﺧل ﻟﻠﻣﺧﺎط وﯾﺧﻠﯾﮫ ﯾﺗﺣرك ﺑﺳﮭوﻟﺔ‬h2o ‫ﻧﺳﺑﺔ‬
‫ طﯾب‬، ‫ ﯾﻌﻣل اﻧﺳدادات‬thick ‫ و ﯾﺎﺛر ﻋﻠﻰ ﻣﻧﺎطق ﻛﺛﯾرة ﺑﺎﻟﺟﺳم ﻓﻠﻣﺎ ﯾﺳﯾر‬thick and sticky mucu‫ وﯾﻌﻣل اﻧﺳداد ﺑﺎﻟرﺋﺔ‬، ‫اﻧﺳدادات ﻣﺎﯾﺧﻠﻲ اﻻﻧزﯾم ﺗﺧرج‬
‫ ﯾﺎﺛر ﻛﻣﺎن ﻋﻠﻰ‬، ‫ ﻓﻲ اﻟﺷﻌب اﻟﮭواﺋﯾﺔ ف ﯾﺣﺻل اﻧﺳداد ﻓﻲ اﻟرﺋﺔ ﺑﺳﺑب اﻟﺳﻣﺎﻛﺔ اﻟﺷدﯾدة‬، ‫ ﻓﯾن ﻣوﺟود ﺑﺎﻟطﺑﯾﻌﻲ ؟ ﻓﻲ اﻣﺎﻛن ﻛﺛﯾرة ﻣﻧﮭﺎ اﻟرﺋﺔ‬mucus ‫ھذا ال‬
‫ ﻣﺎﯾﻘدر ﯾطﻠﻊ‬bile ‫ ﺑﺣﯾث‬liver ‫ ﻛﻣﺎن ﯾﺎﺛر ﻋﻠﻰ‬obstruction ) , ‫ ﻓﺎﻻﻧزﯾﻣﺎت اﻟﺑﻧﻛرﯾﺎس ﻣﺎ ﺗﻘدر ﺗطﻠﻊ وﯾﺣﺻل ﻓﻲ ﻗﻧوات اﻟﺑﻧﻛرﯾﺎس )اﻧﺳداد‬، ‫اﻟﺑﻧﻛرﯾﺎس‬
. ‫ (( وﻟﯾس ﻛروﻧك‬acute pancreatitis)) ‫ ﻓﯾﻌﻣل‬: pancrease and lung ‫ ﻟﻛن اﻛﺛر واﺷﮭر اﺷﮭر ﻣﻛﺎﻧﯾن ﯾﺎﺛر ﻓﯾﮭم ھم‬bile duct‫ﻓﯾﺣﺻل اﻧﺳداد ﻓﯾﮫ‬
Cystic fibrosis The body produces thick and sticky mucus that can clog the lungs and obstruct the pancreas. Cystic
fibrosis (CF) can be life-threatening, and people with the condition tend to have a shorter-than-normal life span. Sixty
years ago, many children with CF died before reaching elementary school age

1pancreatic pseudo cyst


pancreatic pseudocyst
Pancreatic pseudocysts are fluid collections around the pancreas. They arise due to sudden
or long-standing inflammation of the pancreas. While some will disappear when the
inflammation of the pancreas settles down., Serum amylase and lipase levels (limited
utility) – Often elevated.
Management
Most pseudocysts resolve without interference and only require supportive care. For some,
drainage is indicated.

🌹 best method for pancreatic pseudocyst drain ?


A. Laparoscopic B. Endoscopic C. Open D. US guided
🌹 Treatment of *Infected* pancreatic pseudo cyst: Percutaneous drainage ✅ ✔
pseudocysts, with final success rates of >80%. Or can treat by Percutaneous drainage)
، drain ‫ وﯾﻌﻣﻠوا ﻟﮭﺎ‬cyst ‫ ﯾدﺧﻠوھﺎ ﻣن ﺑرة اﻟﺑطن ﻣن اﻟﺟﻠد اﺗﺟﺎه اﻟﻣﻌدة ﺣﺗﻰ ﯾﺻﻠوا ﻟل‬tube ‫ ﻣﻌﻧﺎھﺎ ﯾﺟﯾﺑوا ﺗﯾوب‬Percutaneous drainage ‫ﻟﻣﺎ ﯾﻘول‬
‫ ﻏﯾر ﻛذا ﻟو ﻣﺎھﻲ اﻧﻔﻛﺗد ﻧﺳﺗﻌﻣل طرﯾﻘﺔ ﺛﺎﻧﯾﺔ اﻧﮭم‬infected cyst ‫ وﺗﺳﺗﻌﻣل ﻓﻘط ﻣﻊ ﺣﺎﻻت ال‬، ‫ ﻣﺛل اﻻﺑرة اﻟطوﯾﻠﺔ‬، ‫ ﻣن اﻟﺟﻠد‬percutneos ‫ﯾﻌﻧﻲ ﻣن اﺳﻣﮭﺎ‬
drain‫ وﯾﻌﻣﻠوا ﻟﮭﺎ‬cyst ‫ ﺣﺗﻰ ﯾﺻﻠوا ﻟل‬endoscopic ‫ﯾدﺧﻠوا ﺑﺎﻟﻣﻧظﺎر‬
🌹 Patient recovered from acute pancreatitis episode , presents 6weeks after with vomiting and epigastric
fullness CT done:showed cystic collection behind the pancreas what is the most likely dx:(no fever)
A-Pseudo cyst ✅ ✔ B-Pancreatic abscess C-Pancreatic necrosis
. . .. . . . .. .
🌹 pancreatic pseudocyst, one is 18 cm x 24 cm , how to manage ?
A- per cutaneous aspiration B- endoscopic drainage ✅ C- surgical drainage
🌹 -one is a different scenario, duration of cyst is 6 weeks after acute pancreatitis, how to
manage: A- per cutaneous aspiration B- endoscopic drainage C- surgical drainage
* Percutaneous catheter drainage for (( *infective* pseudocysts )) .
* Endoscopic drainage = ‫ﻜﺘﻒ‬‰‫ ﻣﺎ ﻋﺪا اﻧﻔ‬، ƒ„‫ﺴﺘﻌﻤﻠﻬﺎ ﻣﻊ •ﻞ اﻟﻤﺮ‬v ‫ ﻫﺬا ﻋﺎدي‬.
.......
🌷 Treatment of pancreatic pseudocyst :
a- Percutaneous Drainage✅✅✅ b- Open drainage c- Laparoscopic drainage
.... .... .......
Pancreatic pseudocyst ttt:
- less than 6 cm and 6 week > observation
- more than 6 cm and 6 week > drainage
🌷 Pancreatic pseudocyst 5 week and 18 cm ? A. Medical B. Surgical ✅
🌷-Pancreatic pseudocyst if there is sign of infection as fever and leukocytosis do
Percutaneous drainage 🌷 -if sterile do Endoscooic drainage🌷
.... .... ......

🌹 Alcoholic pt has severe abd pain radiate to back ?


A- stomach cancer B_ alcoholic pancreatitis
....
🌹 Gray turner sign= its sign of Necrotizin pancreatitis = causes of =abdominal hemorrhage
(bleeding) 🌹
Causes of Grey Turner's Sign
Grey Turner's sign may be come after Cullen's sign. Both signs may be indicative of
pancreatic necrosis with retroperitoneal or intra-abdominal bleeding.
The underlying cause of Grey Turner's sign is generally thought to be abdominal
hemorrhage (bleeding), the main source of which is commonly accepted as the pancreas
............. ........
🌷 Poor prognosis of pancreatitis:
A. Hematocrit. B. Lipase above something. C. Amylase above something

🌹IHD with high WBC , acidosis and high amylase 600 ,dx?
A. Acute pancreatitis. B. Vessel occlusion
…………………….
🌹Epigastric pain radiate to the back x Ray normal, high amylase, ct showed pancreatic
swelling, what is this complication? pancreatic pseudocyst

🌹Evacuation of pancreatic pseudocyst by? percutaneous 🌹

🌹Old with painless jaundice . Us show dilated gall bladder,intra and extra hepatic bile
duct . Ask about dx :- A. Klutskin Tumor B. Pancreas Adenocarcinoma ✅. C. Mirrizi

🌹Pancreatitis? IVF 🌹

🌹Case of pancreatitis Admitted, given ivf and i think pain killer Next to add in
managment? A. Ppi B. Antibiotics C.Urgent surgical consultation✅

🍓13X15 pseudo cyst of the pancreas in the lesser sac Patient vitals showed high
temperature treatment: A. endoscopic. B. per cutaneous✅
🌹Pt. With pictures of pancreatitis ( epigastric pain + high amylase) hxof gall bladder stone
,Next important initial step ? A. Crystalloid fluid iv ✅. B. ERCP. C. US
🌹What is the best initial drainage of pancreatic pseudocyst?
A. Percutaneous B. Open surgery C. Endoscopic
🌹 Pancreatic pseudocyst classical presentation. No signs of infection, there was 15×16 cm
fluid collection in the lesser sac. What is the management?
A. Imaging guided aspiration B. Surgical drainage C. Cyst excision D. Endoscopic fluid
aspiration.
🌹 Old patient with painless enlarged gallbladder ? . pancreatic Ca

🌹 A 50 year old man presents with progressive jaundice, dark urine, and right upper
quadrant pain and distention. On physical examination he has a palpable gall bladder.
Imaging shows an enlarged gall bladder and dilated Intrahepatic duct. Amylase = 481.
diagnosis is A. klatskin tumorn B. Pancreatic cancer C. Cholecystitis D. Mirrizi’s syndrome

🌹Asking about complication of pancreatitis after conservative treatment was done: :


Pancreatic pseudocyst

🌹pt had pancreatitis and gall stone , had been managed with fluid and stable what to do :
A. Cholecystectomy at the same admission B. Cholecystectomy after 4-6 weeks
N.B: Acute biliary pancreatitis: First resuscitation specially IVF.
Then : ERCP (stone extraction) followed by lap chole. Abx not indicated unless severe
necrotizing pancreatitis. If it was mild, moderate: lap chole in same admission.
Severe: lap chole in 4-6 weeks.
....
🌹Patient has pancreatitis with necrotic tissue, what is the most important step in
management? A. Antibiotics B. IV fluid C. percutaneous drainage

🌷65 y old patient presented with painless jaundice and wt loss = carcinoma head of
pancreas .

🌹Carpal tunnel syndrome


🌹Regional complex pain syndrome after carpal tunnel surgery release, complain of wrist
and hand pain , been treated with ortho, and pain clinic,, What is the appropriate mx?
A. Opiate. B. Triptan. C. physiotherapy
......
🌹Carpal Pain directly at the incision is typically only present for days or
weeks after the surgery. Protecting the incision can help alleviate pain, and it's important
to avoid lifting or gripping for several weeks after carpal tunnel surgery. ... Treatments for
pillar pain may include rest, massage, and hand therapy
🌹Carpal tunnel syndrome still in pain after surgery?
A- Physiotherapy✅. B- Analgesia. C- Open again
‫ھو اﻧﮫ اﻻﻟم ﯾﺳﺗﻣر ﺑﻌد اﻟﻌﻣﻠﯾﺔ وﻣﻣﻛن ﻛﻣﺎن اﺷد ﻣن اﻻول‬...... pillar pain carbal tunnel =
🌹Patient after carpal tunnel realese complaining of chronic pain what’s ur management
? A.opioids B. physio ✅✅ N.B if the pain was immediate post op most likely incision site
so opioids But if pillar pain > physiotherapy

🌹Patient has nerve radial injury , where level of injury ? spiral groove humerus 🌹
...............‫ = رادﯾﺎل رﯾدي = ھل اﻧت رﯾدي ﻣﺳﺗﻌد ﻟﻠﮭﺟووم ؟‬radial = ‫ = ھﯾوﻣﯾرال = ھﺟوووم‬humeral = ‫رﺑط‬
Patient had loss of sensation on the snuff box and dorsum of the medial hand, hehad wrist drop.
At which level is the radial nerve injured? A. Axilla. B. Humerus groove✅ C. Olecranon D. Carpal
tunnel. Snuff box> radial> spiral groove of humerus
= humeral = humen .🍏🍏🍏🍏 ‫رﺑط = ﺳﺑﺎﯾر = ﺳﻣﻌﺗﻲ = رادﯾل = اﻟرادﯾوا ﻋن اﺧﺑﺎر اﻟﺑﺷر واﻟﻧﺎس‬

🍓Paraesthesia and numbness with wrist drop (shows radial nerve injury) at
which Part A. Groove of the humerus ✅. B. Carpal tunnel. C. Olecranon
Wrist = Radial.

🍓Typist c/o tingling and pis in left thumb, index and middle &
problem in dorsiflexion and fingers extending which nerve affected ?
A. Median. B. Radial ‫ﯾﺴﺎل ﻋﻦ اﻟﻌﺼﺐ‬

Patient with pain when writing at keyboard, some test showed hypoperfusion
to superficial palmar arch, what’s the artery affected? ‫ﯾﺳﺎل ﻋن اﻟﺷرﯾﺎن‬
A. ulnar. B. Radial. C. Ant interosseous. D. Post interosseous

🌹Thenar muscle atrophy ? Median nerve🌹

🌹Female complaining of tingling sensation in her ring finger increase when she raise
her hand , in the examination positive arm elevation test = A) carpel tunnel syndrome ✅

🍒 Pt do surgery then Loss of sensation in ear pinna and upper neck what nerve is injured
? Great auricular nerve
:) ‫ ﻋﺷﺎن اﻟﻣﺟوھرات‬pinna ‫ ﺟزء اﻟﻛﺑﯾر واﻟﻣﮭم ﻓﻲ اﻻذن ھو‬great ‫ و‬، ‫ = ﻣﻌﻧﺎھﺎ اذن‬aurical ‫رﺑط‬

🍒 Numbness on thumb and index finger what is the nerve affected? Radial nerve

🌹Male with left little finger and ring numbness or pain , rais stress test increases the
symptoms, what’s dx: A. carpal tunnel sy B. thoracic plexus
‫ ﯾﺑدء ﻣن‬throcic ‫ ﺑﯾﻧﻣﺎ‬thumb, index finger ‫ ﺗﯾﺟﻲ ﻓﻲ اﻻﺻﺑﻊ اﻟﻛﺑﯾر واﻻﺻﺑﻊ اﻟﻲ ﺟﻧﺑﮫ‬carpal tunnle ‫ال‬
. Little finger . ‫اﻻﺻﺑﻊ اﻟﺻﻐﯾر‬
‫ ﻛرب = ﺷﻲء ﻛﺑﯾر = اﻻﺻﺑﻊ اﻟﻛﺑﯾر‬carpal = ‫ﻣن اﺳﻣﮭﺎ‬

🌹Athlete with pain during standing PE: tenderness in medline planter sarface
A. plantar fasciitis. B. Tarsal tunnel syndrome

🌹Obese patient presented with little finger tingling and +ve raise stress test. the Dx?
A. chronic shoulder subluxation B. Ulnar artery thrombophlebitis
C. Tunnel syndrome D. Thoracic outlet syndrome
1Gallbladder
● Most common cause of biliary colic
A. cholydocolithaiasisis B. Gallstone

An elderly with IHD day 2 post cholecystectomy presented with sudden chest
pain SOB and vitally hypotension and tachycardia whats best ?
A. . CXR. B. ECG. C. CT angio D. LL duplex US

Cause of biliary colic?


A. Choledocholithiasis B. Gallbladder stone C. Gallbladder sludge

● pt with typical cholecystitis Mx? A. immediate lap chole B. wait

🍓what’s the common cause of cholesterol gallbladder stone : A. obesity B. rapid weight loss✅✅

🍒 Colicy abd pain with dilate CBD and intra hepatic duct causes of pain?Construcion of Sphincter of Oddi

Abdominal pain and US show stones in gall bladder with normal wall what is the
management ? a) Lap chole b) Ursodeoxycholic ✅
🌷Ursodeoxycholic Acid, is a naturally occurring bile acid and is used to dissolve gallstones
that are rich in cholesterol. It is also used to improve the flow of bile in primary biliary
cirrhosis.

🌹Old man came with jaundice and dark urine , palpable gallbladder , lab shows direct
belirubin = A) klatskin tumor. B) Perihailar tumor. C) CBD stone

🌷30 y/o male patient presented with abdominal pain related to meals, radiated to the
back, labs resulted high amylase, U/S showed bile sludge with no gallbladder stones were
notified, no CBD dilatation, what’s your management?
A.Labroscopic cholycystectomy . ✅✅ B. Endoscopic U/S C. Endoscopic sphenoidotomy
D. I don’t remember but it wasn’t MRCP or conservative management, so the question
directed to the definitive management to prevent future pancreatitis

🌷Patient during complicated lap chole the surgeon accidentally transected the CBD
(common bile duct( above the level of cystic duct, what’s your management?
A. hepaticoduodenostomy. B. hepaticojejunostomy ✅ C. choledocho- duodenostomy.

🌷Lowest risk for cholesterol gallstones


○ Nulliparity ✅ ‫ ○ اﻟﻤﺮءة اﻟ ” •“ ﻟﻢ ﺗﻠﺪ‬Sudden weight loss ○ Obesity ○Hypertriglyceridemia

🌷pt. With RUQ pain, US showed fluid around gallbladder, management:


A- US guided aspiration ✅ B- lap choly

🌹Recurrent RUQ pain aggregation with morphine, temperature 36.5 ?


y x ‫دواء اﻟﻤﻮرﻓ‬
a) Biliary colic ✅ ‫•ادﻳﻦ‬Ž‫ﻞ ﻟﻪ ﻣﻴ‬€‫ﺪ‬‰ ‫ …ﺴﺘﺨﺪم‬، ‫ﺨﻔﻔﻪ‬€ ‫ ﻳﺰود ﺟﺪا اﻻﻟﻢ وﻻ‬w
b) Cholecytities ‫ ˜ﻌﻤﻞ ﺣﺮارة‬c) Pancreatitis ‫ﻤﺘﺪ ﻟﻠﻈﻬﺮ‬£‫ﻣ›ﺎن اﻻﻟﻢ ˜ﻜﻮن ب ﻗﻤﺔ اﻟﻤﻌﺪة و‬
** all of them can have pain aggravated by morphine due to constriction of sphincter of
oddi analgesia of choice is meperidine

🌹Pt came to ER with ruq pain and jaundice, with fever i think, 2 h later patient improved, us
showed multiple gallstones within normal walled gallbladder,
management: A. lap choly B.follow up C.ursodeoxycholic
....
🌹Pt post PTC has fever, on PR exam ant. Boggy mx ? A. Percut. Drainage B. IV Abx (if small)
...
🌹 RUQ pain on US Dilated common bile duct with intrahepatic biliary dilatation ,
what is the cause ? A. cholelithiasis(gallstone) B. Cholangitis C. Cholecystitis
‫ﻠﻤﺔ‬a ‫ ﻻن ﻗﺎﻟﻚ‬dialantion ‫ﻪ‬j‫ ﻣﻌﻨﺎﻫﺎ ﻓ‬stone
🌹 Patient with jaundice, abd pain and on US u see stones and dilated CBD vitals show fever
only asking about diagnosis...
a. ascending cholangitis✅ b. choledocholithiasis c. cholecystitis
Cholangitis is an inflammation of the bile duct system ( dilated CBD so inflammantion on
bile duct).

🌹Pt with DM and HTN and SCA , type of gallbladder stones will form?
A. Mixed B. Cholesterol stone C. Pigmented >> IF SICKLER
➔ SCD > pigmented
➔ sickle cell trait = Mixed
‫ﺎ ﺣﻤﺮاء =ﺣﻤﺮاء =ﺻ|ﻐﺔ‬v‫ﻞ =ﺧﻼ‬j‫ﻜ‬j‫ر =ﺳ‬.
🌹Diabetic obese elderly female with sickle cell trait has Lithiasis what kind of stone she
has:? A. Mixed B. Fat C. pigmented

.....
Least risk factors for gallbladder stone A.obes B-Hypetrglesride C.nullparty

Patient with history of lap chole few weeks ago presenting with right upper abdominal pain ,
respiratory symptoms , US done and show pus collection at site of gallbladder 12*6 cm What is the
most appropriate in management?
A. antibiotic B. precautions guide drainage✅ C. laparotomy D. laparoscopy
.....
🌹Patient post bariatric surgery complains of on and off fever for one week On
examination chest, abdomen and wound were normal How are you going tomanage:
A. Reassure B. CT abdomen C. Chest x-ray
N.B : Spiking fever > deep collections

🍓female pt present with right upper Q pain , febrile Lab test show: increased WBCs increase ALP
increase Direct bilirubin US Show : fluid around gallbladder , multiple stone What the most
appropriate management ?
A. Lap chole B. laparotomy C. precautions guide dringe D. antibiotic✅
🌹 Elderly pt admitted to Icu with acute MI , and developed pneumonia on tazocin , he
also have RUQ pain and tenderness. Management ?
A. ercp drainage B. emergency cholecystectomy
C. convert from tazocin to meropenem D. us guided cholecystostomy drainage

🍓 post cholecystectomy came with perihepatic collection, what will you do?
A- Laproscopic driange B- Percutanous driange✅✅ Dr.abdullah’s answer:
Any Post-Op Abdominal collection should be drained Percutaneously.

🍓 patient with diverticultits did sigemoidectomy , 5 or 7 days after had fever on per rectal
exam there is bogginess anteriorly what to do : A-drainage ✅ b-reassure

🌹 Patient with RLQ pain ct showed collection 13*10 =


A. percutaneous drain B. open appendectomy C. lap appendectomy

🌹 Post colectomy paracolic collection:management?


-Ultrasound guided drainage ✅ -open drainage
.....
🌹Patient with vomiting, constipation and fever Known case of crohns CT showed large collection
and fistula *initial* management? A. Percutaneous drainage ✅. C.Lap drainage and fistulectomy
It’s not recommended to do both procedure at the same time, you should drain first and later
fistulotomy
🌹UC pt with y shape something with very enlarged transverse colon and no haustrea what is Rx
A.proctocolectomy with ileal pouch B. pan colectomy with ileostomy✅
Answer is:IV Steroids if failed : total proctocolectomy or subtotal colectomy. Dx: toxic megacolon
. If no steroid in choice then choose B
🌹Pt with jaundice ; us showed 1.1 cm common duct dailated and dilated intra and extra hepatic ;
and very distended gallbladder: A. panceriatic cancer B. Mrizzi syndrome C. Klitskin D. Cbd stone
Kelastin or perhailar cancer >shrunken GB Pncreatic cancer >distended

🌹pancreatitis case managed conservatively at the day 3 the pt improving clinically and laboratory
. Every thing normal except Amylase 250 . US showed>> Gallstone at the gall bladder with dilated
extra hepatic ducts ( nothing was mentioned about CBD). What is the appropriate management:
A. lap chole before discharge B. elective lap chole C. ERCP
….

1ERCP
🌹 Post cholecystectomy ( collection was found behind gallbladder) what to do:
_Percutaneous drain ✅ _Open drainage Laparoscopic drain

🌹 -Obstrictive jaundice pic and cholangitis on IV abx ,US (dilated ducts, gallbladder
has stones) what else in the mx? -ERCP✅ -cholecystectomy

🌹Typical pic of acute cholangitis, next investigation? A. US B. MRCP C. ERCP


Cholangitis is an inflammation of the bile duct system it happened bez there is stone on bile duct
and any stone on bile duct best manag with ERCP
‫ ﻣﺜﻞ ﻣﺎ ﯾﻘﻮﻟﻜﻢ‬dialated ‫ ﺳﻮاء ﻛﺎﻧﺖ‬bile duct ‫ او ﺟﺎﺑﻮا ﺳﯿﺮة‬bile duct ‫ ﻓﻲ‬stone ‫أي ﻣﺮة ﻧﺸﻮف‬
ERCP ‫ اﺧﺘﺎروا داﺋﻤﺎ‬common bile duct dialated
🌹Pt with fever, jaundice and RUQ pain,initial management= A. abd US B. ERCP C. MRCP
cholangitis ‫ھﻧﺎ ﺗﺷﺧﯾص اﻟﺣﺎﻟﺔ‬

🌹patient with pneumonia take tazocin develop RUQ pain, ultrasound show dilated
common bile duct accumulation of fluid thick wall of gallbladder how to manage?
ERCP
‫ راح‬، ‫ ﻓﺣﺗﻰ ﻟو ﻋﻣﻠﻧﺎ اﺳﺗﺋﺻﺎل ﻟﮭﺎ‬gallbladder ‫ وھذه اﻟﺣﺻوة ﺧرﺟت ﺑرة‬، ‫ ﻟذﻟك ﺗراﻛم اﻟﺳﺎﺋل وﺣﺻل ﺗﻣدد ﻓﻲ اﻟﻘﻧﺎة‬bile ‫ﻣﻌﻧﺎھﺎ ﻓﯾﮫ ﺣﺻوة ﺳدت ﻣﺟرى ﺧروج‬
ERCP.‫ ﯾﻌﻧﻲ ﻣﻧظﺎر ﯾﻔﺣص وﯾﻌﺎﻟﺞ ﺑﻧﻔس اﻟوﻗت وھو‬، ‫ ﻓﻲ ﻣﺛل ھذه اﻟﺣﺎﻻت ﻻزم ادﺧل ﺑﻣﻧظﺎر واﺷوف ﻣﻛﺎن اﻟﺣﺻوة واﺷﯾﻠﮭﺎ ﺑﺎﻟﻣرة‬، ‫اﻟﺣﺻوة ﺗظل ﻣوﺟودة‬
🌹Pt with history of Cholelithiasis, today present with abdominal pain after fatty
meal, on US : multiple gallbladder stones, thick wall , CBD 12 mm , what’s next:
A. ERCP✅ B. MRCP C. lap cholecystectomy D. open cholecystectomy

🌹Pt with calculous cholecystitis come with acute attack, When to do operation to
Her = A. Lapcholy as soon as possible ✅ B. Wait 2 or 3months

🌹 jaundice with us show stone in gall bladder and in common bile duct , tx.
a. Laparoscopic cholecystectomy b. ERCP✅
ERCP .‫ ھذا ﻻزم‬commin bile duc t ‫ ﻓﻲ‬stone ‫اذا ﻗﺎﻟك‬
🌹 patient presented with signs and symptoms of cholecystitis, days after hospitalization
the patient recovered clinically and her labs returned back to normal except for AMYLASE,
what is your management: ‫ﺐ ﺣﺼﻮة‬-‫ﺎس ¬ﺴ‬£ª‫ﻣﻌﻨﺎﻫﺎ ﺣﺼﻞ اﻟﺘﻬﺐ „ •¨ اﻟﺒﻨﻜ‬
A. ERCP ✅. B. Interval cholecystectomy 6-8 weeks
cholecystectomy is recommended for patients with gallstone-induced pancreatitis. ERCP
with endoscopic sphincterotomy (ES) within 24 to 48 hours is also suggested for the
treatment of acute gallstone pancreatitis.
......
🍓Case of post cholecystectomy and you found another stone in CBD how to ttt: ERCP✅

🍓 patient present with recurrent right upper pain , jaundice Vital sign :36.7tempratue and
stableLab test :Normal WBC increased direct bilirubin increased ALP US Show:Thick wall
gallbladder , multiple stone , and CBD more than 12 cm What is the next step:. ERCP✅

🍓 Post lap chole presented after few days with abd distension and ascites and abd pain
what will u do? a. open b. lap c. tapping D . ERCP

🌹 After long ERCP, patient hypotensive with pain ..Most common site injured:
A. Esophagus B. Duodenal C. Gastric
N.B: Injury > ، Duodenum . ‫ اﺛﻧﺎء اﻟﻌﻣﻠﯾﺔ ھو‬injury ‫ اﻛﺛر ﻣﻛﺎن ﯾﺣﺻل ﻟﮫ ﺿرر‬ERCP ‫ﺑﻌد ﻋﻣﻠﯾﺔ اﻟﻣﻧظﺎر‬
Complication >pancreatitis duodenal
🍓 Most common organ damage in ERCP: A. Duodenum B. Esophagus

🍓 Patient post cholecystectomy day 8or 9 develops right mouth corner pain and
fever 38.5 what is your mx: Paracetamol. Antibiotic✅✅. CT

🍓 Pt do cholecystectomy 8 days ago now he have pain in angel of the mouth for one day?
A.ABx B. ct abd C. xray D.paracetamol

🌹 Patient came with RUQ pain , ultrasound findings : thickining gallbladder wall ,
pericystic fluid and stones . What's best intervention :
A) percutaneous drainage B) Laparoscopic cholecystectomy ✅

🌹 6 weeks Post MI patient, planned to do cholecystectomy:


A.Do Lap cholecystectomy now ✅🌹 B.Do Open cholecystectomy now
C.Do lap cholecystectomy after 6 months D.Do open cholecystectomy after 6 months
.........
🌹Gallbladder polyp *less than 10cm* :-
*if elderly or with stone* > cholecystectomy
*if less than 50y >* follow up with us every 6m

🌹 Pt did abdominal US as htn screen and the results show gallstone polyp what to do?
A.Follow up in 6 month ✅ B.Surgical consult

🌹 Patient with biliary polyp, 0.6cm. Management:


A. reassure. B. follow up C. cholecystectomy
N.B: Gallbladder polyp less than 10mm :-
if elderly or with stone > cholecystectomy
if less than 50y > follow up with us every 6m

....
🌹Cholecystectomy 6 years back, presented with vomiting and increased bowel sound
movement (exaggerated) A. Adhesion ✅✅

🌹Kc of small gallstones presented with jaundice, Labs show high alk phos, high cong
bilirubin, Next? A. Ercp B. Abd us ✅✅
N.BYou have to start by US first If revealed dilated CBD go for ERCP

🍓Pregnant 10 weeks, complaining of attacks of biliary colic in the past 5 weeks TTT:
A. laparoscopic cholecystectomy now B. laparoscopic cholecystectomy after delivery
C. laparoscopic cholecystectomy in second trimester✅
D. laparoscopic cholecystectomy in third trimester

🍓Acute cholecystitis presents with right upper quadrant pain, fever, and leukocytosis.
Patients with acute cholecystitis should be treated with
laparoscopic cholecystectomy within 72 hours

🍓30y female patient, came to ER with abdominal pain... She has hx of elective
cholecystectomy due to Gallstones. Now she has high amylase, 9mm CBD and
jaundiced diagnosis= missed stone in CBD

🌹Lady post cholecystectomy , due to cholothiasis. week later presented with jaundice ,
pain , U/S: showed dilated CBD. Dx: = Retained stones

🍓70 years old patient with a history of Myocardial infarct that was 6 months ago, on this
presentation he has cholecystitis and requires cholecystectomy. when will
you operate? A. now✅✅ B. After 6 months

🍓cholecystectomy the surgeon accidentally cut off the Common bile duct at a level that is
just above the cystic duct, how to repair?
A. hepaticojejunostomy✅ B. Choledocojeujunostomy

🍓 laparoscopic cholecystectomy come with discharge from middle management “• „ ‫˜ﻌ‬


‫ل ﻣﻨﻪ ﺻﺪ˜ﺪ‬±„ „ ‫ح ﻳ‬ª‫ اﻟﺠﺠ‬A.daily dressing B- wound inspect. C- exploration

🍓Middle discharge after cholecystectomy =A. Abs B. Exploration✅


No dressing or inspiction

🌹Patient on 3rd day post cholecystectomy develops fever, has no abdominal pain.What is
the likely cause of fever: A.UTI B.Wound infection C.Chest infection
3 day = UTI = ‫ﺛﻼث ﺣﺮوف‬
...
🌹Patient post cholecystectomy, presented early (don't mention which day) with fever and
lung consolidation (atelectasis), and US showed 10*12 collection of fluid in lesser sac ,Mx?
A. Physiotherapy B. abx C. Precut drainage
......
🌹Pt have fever and jaundice and RUP pain dx? Cholingitis
.....
🌹Charcot triad = fever , jaundice, RUP pain = Cholingitis ( not cholecystitis)
...
🌹Post cholecystectomy recently (not mentioned the day) with fever spiking,
decrease air entry in right lower lung, dullness in percussion and tenderness in
right hypochondrium US showed collection 10x15cm in gall bladder fossa, ttt:
A. physiotherapy B. iv abx C. drainage
.....
🌹klatskin tumor= is cholangiocarcinoma located at bifurcation of common hepatic duct =
CA 19-9.

🌹Patient after prolonged difficult ercp, developed neck, chest and abdomen surgical
emphysema. What was injured? A. Esophagus B. trachea C. Duodenum D. bile duct

🌹RUQ pain and dilated common bile duct, what is the diagnosis? Choledocholithiasis ✅
Common bile duct stone, also known as choledocholithiasis, is the presence of gallstones in
the common bile duct (CBD) (thus choledocho- + lithiasis)
....... . . .. ..
🌹Choliduocholithisis managment?- ERCP ✅

1GIT
🌹Pt alcoholic ,smoker came with sever pain generalize in the abdomen In
examination was tenderness and guarding= . Duodenal perforation
🌹Post percutaneous transhepatic cholangiography developed upper gi bleeding, invx?
A. Endoscopy B. CTangio C. US

🌹Elderly smoker with dysphagia , showed high grade dysplasia , management?


A. Add ranitidine B. Ask him to stop smoking C. CBT D_ endoscopic mucosal resection

🌹best diagnostic test to detect ischemia :


A. stress Echo ( Approved)✅ B. CK

🌹What is sign in radiology for duodenal ulcer ==clover leaf


‫ﻂ‬š‫ = ر‬doudenum = ‫ = ودود‬love = clove

🌹 Old man suddenly had vomited two episodes of bloody vomit, Physical Exam shows no
Tenderness over his abdomen or sternal area, he is stable, what is your diagnosis:Mallory
weiss tear
🌹 24 hours vomiting after food poisoning develop hematemesis on endoscopy blood
streak on stomach Dx? Mallory weiss

🌹 adult patient ( i forgot the age may be in 30 ) have repetitive vomiting come
with mild bleeding ( mallory weiss syndrome )what is the appropriate
management?
A. conservative management B. laparotomy exploratory C. laparotomy with resection

🍓C/I to liver transplantation= Cirrhosis with active alcohol✅

🍓patient with hepatitis B cirrhosis with ascites and have 6 cm lesion found in the with
high vascularity = treatment? A. anti viral. B. resection. C. chemo embolization✅✅
Transcatheter arterial chemoembolization is a minimally invasive procedure performed in
interventional radiology to restrict a tumor's blood supply.

🌹 pt with Chronic hepatitis b then , then discover 70% of the liver was multiple lesions,
next: Colonoscopy.
...

🍓Male Pt, smoker, have a history of appendectomy, his brother have crohn’s ,
what’s the risk factor to develop crohn’s : A. male B. smoking C. family history✅ D. history
of appendectomy

🌹Case of GERD with High grade dysplasia what you will do


A.Endoscopy 6 m B. Laparoscopic esophagectomy ✅✅

🍒 pt underwent pneumodilation for achalasia cardi , present after while with signs of
esophageal perforation . ask about mx ?
A.stent B.esophagectomy Answer is : stable > stent , unstable > surgery

🌹Healthy female with dysphagia diagnosed as achalasia, best treatment


A- pneumatic dilatation B- botulinum injection C- fundoplication

🌹Valvulas location: A.Cecum B.rectum C. sigmoid ✅

🍓Y shaped colon, Sigmoid volvulus what to do? Colonoscopy or sigmoidoscopy🍓

🍓Old patient history of vomiting with palpable mass imaging finding Y shape next app
action? A. Fleet enema B. Ct C. Sigmoidoscopy D. Colonoscopy

🍓Old male with generalized abdominal pain, ascites, constipation X-ray >> massively
enlarge loop up to right part ? A. Sigmoid volvulus ✅ B. Acute diverticulitis
🍓52-Patient came with abdominal diatention x ray showed y shaped colon what’s your
management? A. Fleet enema B. Sigmoidectomy with end colostomy C.
colonoscopy✅✅(obstruction)

🌹 Pt with UC the most associated risk of Cancer is?


A. primary sclerosing cholangitis✅ B. More than 3 years UC illness duration
C. UC confined to part of colon. D. Mild inflammation

🍒 Case of Sclerosing cholangitis, what you do for the patient? A- CT B- Colonoscopy ✅


‫ ﻟﻣﺎ‬، ‫ ﻋﺷﺎن ﻛذا‬ulceraitive colitis ‫ ﯾﻛون ﻋﻧدھم‬Sclerosing cholangitis ‫ ؟ ﻻن ﻛﺛﯾر ﻣن اﻟﺣﺎﻻت اﻟﻲ ﻣﺻﺎﺑﯾن ب‬Colonoscopy ‫ﺗﻌرﻓوا ﻟﯾﮫ اﺧﺗرﻧﺎ‬
‫ ﻻن ھذا اﻟﻣرض ﻓﻲ اﻟﻐﺎﻟب‬، ‫ ﻋن طرﯾق ﻋﻣل ﻣﻧظﺎر اﻟﻘوﻟون‬ulceraitive colitis ‫ ﻻزم ﻧﺳﺗﺑﻌد وﺟود‬Sclerosing cholangitis ‫ﻧﺷوف ﻣرﯾض ﻣﺻﺎب ب‬
.‫ ﷲ ﯾﺷﻔﯾﮭم‬، ‫ ﻻزم ﯾﻛون ﻣﻌﺎه ﻣﺻﯾﺑﺔ ﺛﺎﻧﯾﺔ‬، ‫ﻣﺎﯾﺟﻲ ﻟﺣﺎﻟﮫ‬
🍒 Abdominal distended, vomiting, picture of obstruction. Dx? volvulus

🍒 What is a volvulus? A volvulus causes bowel obstructions, which may cut off the blood
supply to areas of the bowels. A volvulus occurs when part of the colon or intestine twists.
The twisting causes bowel obstructions that may cut off the blood supply to areas of the
bowels

🍒 Management of unstable patient with sigmoid volvulus?


A-Sigmoidectomy B-the other opsions were all conservative management.

🌹Most common site of sigmoid volvulus?


➔ Elderly > sigmoid ➔ Children > Ceacum

🌹Patient with basal ganglia disorder and absent gag reflex. How to provide nutrition?
A-Gastrostomy B-Jejunostomy✅ . C-NGT!@

🌹Pudendal nerve block. Which organs are still sensitive?


A-Vulva. B-Perineal body. C-Urogenital diaphragm D-Rectum✅✅

🌹Colorectal surgeon performing a low anterior resection for CRC and the pelvis won’t
stop bleeding, so he consults a vascular surgeon and he does:
A-Heavy packing of pelvis B-Arteriography intra operatively C-Infraceliac clamp !@

🍓bleeding aortic in OR and you did packing but he is still bleeding. Next?
A. clamping infrarenal B. clamping supraceliac C. cable CTA!@

🌹Patient had RTA with seat belt sign x ray of the spine showed chance fracture what most
probably you will find in = Duodenal perforation
‫ او ﺗظﮭر ﻋﻠﻰ ﺑطن اﻟﻣرﯾض اﻟﻲ ﻋﻣل ﺣﺎدﺛﺔ اﻟﻌﻼﻣﺔ ھذه ﻣﻌﻧﺎھﺎ ﻣﺛل )رﺑطﺔ ﺣزام اﻻﻣﺎن( ﺑﺎﻟﺿﺑط‬x.ray ‫ ﯾﻌﻧﻲ ﻋﻼﻣﺔ ﺗظﮭر ﻓﻲ‬seat belt ‫اول ﺷﻲء ﻋﻼﻣﺔ‬
‫ ﻟﻛن ﻣﺎﻓﻲ ﻣﺻدر ﯾﻘوﻟك اذا ﺷوﻓت ﻋﻼﻣﺔ‬bowel ‫ وﺧﺎﺻﺔ‬organic damage ‫ ھذه اﻟﻌﻼﻣﺔ اذا ﺷوﻓﺗوھﺎ ﻓﻲ اﻟﻣرﯾض اﻋرﻓوا اﻧﮫ ﺣﺻل ﻟﮫ‬، ‫وﺷوﻓوا اﻟﺻورة‬
‫ ﻟﻛن اﻟﻣﺻﺎدر ﺗﺗﻛﻠم ﺑﺷﻛل ﻋﺎم ان ھذه اﻟﻌﻼﻣﺔ اذا ﺷوﻓﺗوھﺎ اﻋرﻓوا اﻧﮫ ﻓﯾﮫ‬، ‫ ﺗﺣدﯾدا‬dudenal perforantion ‫ ﻣﻌﻧﺎھﺎ ﻓﯾﮫ‬seat belt ‫ﺣزام اﻻﻣﺎن اﻟﻲ ھو‬
abdumen. ‫ اﻻﻋﺿﺎء اﻟﻲ ﻣوﺟودة ﻓﻲ‬pancrias , dudenum ‫ﺳواء‬organic damage
The seatbelt sign is both a clinical and radiological sign. It is simply the presence of bruising/abrasions in the distribution
of a seatbelt (i.e. horizontal and/or diagonal) across the abdomen, chest and sometimes neck. A positive seatbelt sign, in
combination with abdominal pain or tenderness, results in a higher likelihood of intra-abdominal injuries, in
particular bowel/mesenteric injury
Seat belt signs remain an important physical finding following motor vehicle crashes. Should be aware of seat belt sign and
there should a higher index of suspicion to rule out underlying organ injuries.

🌹Rta patient his speed was 130 and he was on seat belt came with stable status no hypotension or change in
consciousness what you will do = A. CT B. us of abdomen. C. laparotomy

🌹Pt elderly known to have IHD come with sever central abd.pain Amylase: slightly
elevated = Mesentric emboli of occlusion

🌹Patient known to have ischemic heart disease complain of abdominal pain (the only complain)
with lab showing amylase of 600 and x-ray showing signs of obstruction ?
A.Acute appendicitis B. intestinal obstruction C. mesentric ischemia ✅✅

55 years old male presented with sudden abdominal pain radiated to the back the patient has
history of cardiomyopathy, the patient mentioned that he passed small amount of loose stool,
during the examination there is exaggerated bowel sound Labs: Amylase = mildly elevated above
normal What is the Diagnosis?
AAcute pancreatitis B. Mesenteric vascular ischemia✅ C.Ruptured aneurysm

🌹seat belt sign = bowel/mesenteric injury like Duodenal perforation.


….

🌹pt complain of malena and epigastric pain diagnosed as peptic ulcer and received ppi but
not improved endoscopy done showed multiple ulcers in the antrum what is the most
accurate management a.antrectomy✅✅. b.partial gastrectomy. c.total gastrectomy
##multiple ulcers in the antrum = antrectomy ##
....
● Multiple antrum ulcers , by Bx >H pylori what is the mx ?
A. start Abx B. Chemo C. Radiation

🌹 GIST , 5 cm in posterior wall, mx: A-Total gastrectomy. B. partial gastrectomy. C. WLE

Gastrointestinal stromal tumors are soft-tissue sarcomas that can be located in any part
of the digestive system. Their most common sites are the stomach and small intestine.
🌹gastrointestinal stromal tumors with metastasis = treat by = tyrosine kinase inhibitors
(imatinib)

🌹 GIST (Giant gastrointestinal stromal tumor ) in body stomach 3x4 cm no lymph node
metastasis , ttt? A- gastrectomy B- wedge wide local exsion ✅✅
🌹Giant gastrointestinal stromal tumor = wedge wide local exsion 🍬== Dont choice (gastrectomy) 🌹

🌹pt with GIST (Gastrointestinal stromal tumor less than 2 cm )


, what is the most accurate
management:- a.total gastrectomy b.partial gastrectomy c.observation ✅. d.ppI
if 2 cm or less then observation
....
🌹Gastrointestinal stromal tumor 3x4 cm. No infiltr'tion, no lymph nodes. Wh'tʼs the
m'n'gement? - G'strectomy- _Wedge resection with clear margins✅ - Chemo

🌹stomach tumor with positive secretin stimulation test


a.VIPoma. b.glucagonoma c.gastrinoma✅✅

‫ ﻣﻦ اﺳﻤﻬﺎ ورم •¨ اﻟﻤﻌﺪة ﻣﻊ ﻫﺮﻣﻮن‬gastrin ‫ج ﻣﻦ اﻟﻤﻌﺪة‬ª‫ ا•¾ ˜ﺨ‬.

🌹pt with LLQ pain, and hx of constipation, on exam, bulky mass in LLQ, with wild
discomfort, no worrying sings or old age, dx: A- constipation ✅ B- others on related

🌹50 yrs man diabetic, well controlled had colon cancer surgery ( coloectomy)
they kept him on insulin and dextrose, after surgery by 2 days he became irritable, in
shock, his electrolyte ( Na 129 ) ( K 3.2 ) urine and serum osmolality Normal, what's the Dx:
Fluid overload ✅ ✔

(( Hypervolemia is a condition in which there is too much fluid in the blood. It is also known
as fluid overload. , Hyponatremia is decrease in serum sodium concentration <
136/L caused by an *excess of water relative to solute* .))

🍒Long case IBD and do surgery not mention it and now have to take fat soluble vit which
part of intestine had been rescued = terminal iluem Vitamins A, D, E, and K, fats, and
cholesterol are absorbed in the lower third the ileum. ) 🍒
🥦What vitamins are absorbed in the ileum?🥦
Protein is absorbed approximately midway through the ileum. Vitamins A, D, E, and K, fats,
and cholesterol are absorbed in the lower third the ileum. Vitamin B12 is absorbed just
before the small intestine joins the large intestine. Bile salts are reabsorbed in the
distal ileum and the ascending colo
🍉If the terminal ileum is missing a deficiency of vitamins A, D, E and B12 may occur. 🍉
Vitamins A, D, E, and K = Fat soulebal vit
🌹Patient while doing laparoscopy for esophageal perforation with Bp 80/50,,RR25 pulse
120 which type of shock?
A.Cardiogenic B.Hypovolemic ‫ﺐ اﻟﺜﻘﺐ‬-‫ﻒ ¬ﺴ‬£À‫ ﻧ‬C.Septic D.Anaphylactic
🌹patient with GERD underwent endoscopy show multiple antral ulcer Ttt?
A.Total gastrectomy B. Partial distal gastrectomy ✅ C. Pylorectomy ❌
🌹 Succession splash -‫ﻄﻦ ﻋﻦ اﻟﻔﺤﺺ‬Ç‫ﺴﻤﻌﻪ „ •¨ اﻟ‬v ‫ﺴﻜﻮب‬Æ‫ﺘ‬Å‫ﺎﻟﺴ‬Ä ‫ ﻫﺬا اﺳﻢ ﺻﻮت‬symptoms of
obstruction patient have? A.met acidosis B.Met alkalosis ✅ C. Compensated met
alkalosis D. Compensated met acidosis =
= alk ...........‫ =ﺻوت = ﻗﻠق‬succ = ‫رﺑط‬
….
🌹Symptoms of obstruction and X-ray done showed Dilated loops towards RUQ = A.loop
obstruction B. Sigmoid megacolon ✅🌹
..
🌹Bowel obstruction symptoms. Imaging showed stricture. How to treat a stricture
A.Bowel resection ( 2 options of different parts of bowel) B. Stricturoplasty ✅🌹
….
🌹 A case of crohn's and abdominal obs due to stricture It was single and 1cm away from
ileocecal valve, Mx? A. Stricturictomy B. Observe C. Resection D- laparoscopy with surgical
resection!@

🌹 K/c of chrons with perianal mass , painful , there discharge when touch it , Mx?
A. Increase dose of infliximab and reduce B. fistulotomy. C-MRI pelvic
….
🌹Crohn's patients on steroids and infliximab came with perianal pain and
discharge and low grade fever since 3 weeks what will you do?
Antibiotic Pelves MRI ✅ c. Increase the dose of infliximab!@
Examination under anesthesia (EUA) is considered the gold standard to diagnose and
classify perianal fistulas in CD patients. An accurate diagnosis is also possible using imaging
modalities such as pelvic MRI and/or endoanal ultrasound (EUS), and, in some cases,
transcutaneous perineal ultrasound (TPUS). Any of these methods should be combined with
the endoscopic examination to assess the presence or absence of active inflammation in
the rectosigmoid colon, the presence of in
(Angiodysplasia is a small vascular malformation of the gut. It causes sever bleeding
therfore must be treat )

*Treatment for angiodysplasia*


- If the anemia is severe, blood transfusion is required before any other intervention
is considered.
- Endoscopic treatment is an initial possibility, where cautery or argon plasma
coagulation (APC) treatment is applied through the endoscope.
- Failing this, angiography and embolization
- Resection of the affected part of the bowel may be needed if the other modalities fail.

🌹Angiodysplasia in 60 year patient, how to manage? argon plasma coagulation (APC) and
bipolar electrocoagulation (BEC) = Both are safe and effective ✅ ✔

🌹Patient with rectal bleeding done technetium scan and Dx with angiodysplasia in left
colon TTT: -
A. Conservative B. laser ablation C. angioembolization > IF SEVER D. left hemicolectomy

🌹management of angiodysplasia= A) left hemicolectomy B (cautery embolization (if


failed choice A)
….
🌹Old p'tient >50 ye'rs old. Alternating diarrhea and constipation. Physic'l is normal.
Underwent sigmoidoscopy, revealed multiple lesions in the dist'l sigmoid. Biopsy =
'adenocarcinoma'. Next step in management? Sigmoidectomy Colonoscopy CT bdomen!@
........
🌹Case of perianal swelling , perrectal bleeding , investigated : it is 1 cm from anal
verge , biopsy: ((adenocarcinoma )) (rectal) Mx
- abdomen perineal resection ✅. -low anterior resection. -chemo/radio
….
🌹 20-year-old weight lifter developed abdominal hernia that is irreducible, tender. Imaging
shows air-fluid levels in small bowel and no free air in large bowel. Type?
○Irreducible. ○ Incarcerated. ○Strangulated
Incarcerated hernia and strangulated hernia both can cause SBO.
Incarcerated hernia presents an irreducible hernia with tenderness and abdominal
distension.
Strangulated hernia: ischemic segment will cause symptoms such as erythema,
leukocytosis and fever. (Toxic patient)
-- ----- ----- -----
🌹4th day post OP (appendicitis) presented with diffuse abd pain, distension , vomiting
and sluggish bowel sounds Abd xray: Multiple air/fluid level
-ileus ✅ ‫ ˜ﻜﻮن ﻣﻌﺎﻫﺎ ﺻﻮت اﻻﻣﻌﺎء ﻛﺴﻮل او ﺧﺎﻣﻠﺔ‬-adhesions and SBO ‫ﺪ ﺻﻮت اﻻﻣﻌﺎء‬£À‫ﻳ‬
(( sluggish‫ ﻛﺴﻮل او ﺧﺎﻣﻞ‬bowel sound == ileus))
((Incresse bowel sound ==adhesions and SBO ))
...... . .. . . .. . . .. .
🍓Most common cause of Small bowel obstruction > adhesions✅ Large bowel > neoplasm✅

...... . .. . . .. . . .. .
-Pt 6 yrs post abd operation has bloating and signs of obstruction cause?
a. adhesions✅ b. perforation

🌹case scenario of intussusception what is the gold standard investigation= barium enema
best intinal = us Gold stander =enema
‫ﺎﻻﻣﻌﺎء‬Ä ‫ﻌﻴﻨﻚ اﻧﻪ ﺣﺎﺻﻞ ﺗﺪاﺧﻞ‬Ä ‫ﺸﻮﻓﻬﺎ‬Ò ‫ﻐﺔ ﺗﻮﺿﺢ •ﺎﻧﻚ‬Ç‫ ﻻن اﻟﺼ‬Í• ” ‫ ﻣﻨﻄ‬.
...... ....... ..... .
Patient with symptoms of intussusception what is true about it:
A/reduction using enamas only in case of peritonitis
B/Treated by immediate surgical intervention
C/Recurrence is high after surgical treatment✅✅
….
🌹-What type of cancer that should be screened for with out symptoms :
Colon ✅ Pancreas Skin
........

🌹 28 male healthy, hx 2 month of abd pain with 2 bloody stool, proctoscope done with
numerous polyps covered the linings and multiple biopsy taken , no details in hs of family
hx or sexual.. Dx:
A. familial polypoid✅ B. UC C. diverticulosis coli D. human papillomavirus polyp

🍓Multiple polyps in colon and duodenal And bloody diarrhea?


A, Familial Colon Cancer Syndromes✅. B. UC. C. Crhrons

🌹Long case with Thumbprint sign on abdominal x-ray ? Pancritits or Ischemic colitis 🌹

🌷Thumbprint sign in abdomen = Pseudomembranous colitis AND ischemic colitis🌷

🌹40 yo male K/C of hyperlipidemia present to ER with epigastric pain, and normal Amylase
and lipase, slightly elevated AST, ALT, abdominal xray show air in LT hypochondrium " and
give the Name of sign" Dx?A. Acute pancreatitis. B. Ischemic colitis

🌹Anterior duodenal perforation tratment = : Graham omental patch✅


Graham ‫( وﺑﺎﻟﺗﺎﻟﻲ ﯾﺣﺻل ﻧزﯾف ﻋﻼﺟﮭﺎ ب ﺷﻲء ﯾﺳﻣﻰ ب‬perforation) ‫ وﻣﺎ ﺗﺗﻌﺎﻟﺞ ﯾﺗطور اﻟﻰ ﺛﻘب‬duodenal ‫ﻟﻣﺎ ﯾﺣﺻل ﻗرﺣﺔ ﻓﻲ‬
:duodenal ‫ ﻓﻠﻣﺎ ﻧﺷوف ﻣرﯾض ﻋﻧده‬، ‫ﯾﻐﻠق ھذا اﻟﺛﻘب‬omentum ‫ﯾﻌﻧﻲ ﻧﺧﯾط اﻟﺛﻘب ﺑﺎﻻوﻣﯾﻧﺗم ب ﻏرزة ﻣﻌﯾﻧﺔ ﺑﺣﯾث ال‬omental patch
perforation = treat by = Graham omental patch.
Graham omental patch ‫ﺎل‬Ä ‫ﻂ اﻟﺜﻘﺐ‬‰‫ ا•¾ ﻫﻮ ﻧﺨ‬oment .
🌹Man with high grade dysplasia of the esophagus== Refer him for surgery

🌹A young healthy man medically free came complaining of indigestion problem


otherwise he is normal no vomiting no nausea no bloody diarrhea you gonna do ? A.
Routine abdominal x ray B. Urgent surgery C. Routine referral to gastro D.functional dyspepsia

1Truma
🌹Patient post RTA with warm peripherals Which type of shock?
A. Cardiogenic B. Septic(cold priphral) C. Neurogenic bez (warm peripheral)
🌹15year, fell and revived a supracondylar fracture above the elbow. Management?
If Brachial pulse present >> Reduction If Not present >> Surgical Exploration

🍓Patient in ER due to RTA , patient denial any history of loss of consciousness ، GCS 15 ,
suddenly patient deteriorated and loss of his conscious with dilated pupils hats is the most
likely diagnosis : A. AV malformation B. subdural hematoma C. Epidural hematom✅

🍓What kind of shoulder dislocation.. difficulty in addiction and internal rotation?


A. anterior B. posterior dislocation ✅✅

🌹abdominal trauma (by wooden stick) with painful RLQ pain and grey discharge, pain
increased by extending the leg. Next step? A.Culture B. Give antibiotics ✅ C.CT
abdomen

🌹Female had femoral fracture then after I think a week developed respiratory symptomes
? Fat embolism syndrome ✅
¨• „ ×‫ﺸﻮﻓﻮا ﻛ‬Ò ‫ اي ﻣﺮة‬Femoral ‫ﺎﻟﻚ ﻫﻮ‬Ä ¨• „ ‫ ˜ﺨﻄﺮ‬ÚÛ
• ‫ﺎﻟﺘﻨﻔﺲ اووول‬Ä„ ‫ﻌﻈﻬﺎ ﺿﻴﻖ‬Ä ‫ت وﺣﺼﻞ‬ ‫ﺎﻟﺬا‬Ä fat
ß
embolism ‫ ﻻن ﻛ× ﻋﻈﻢ‬، ‫ﺴﻮﻫﺎ‬Ý‫ﻢ ﺗ‬Ü‫ ا˜ﺎ‬femoral ‫ﺲ ﻋﻨﺪە ﺧﻄﻮرة •¨ ﺣﺼﻮل‬Æ‫ ﺷ‬±Þ‫ ﻫﻮ ا‬fat
Fat =Femoral
.......
🌹5 days post orthopedic surgery had sudden dyspnea and confusion on
examination shows rash on neck and on cxr bilateral lower lobe infiltrates cause?
a. Fat embolism✅ b. PE c. pneumonia

🌹Colles fracture in pedia what is the most important mangement ? closed reduction
Colles = Close ‫ﻠﻮز‬¥= ‫ﺲ‬£‫ﻣﻦ اﺳﻤﻬﺎ ﻛﻮﻟ‬
🌹 Picture of green stick and ask for treatment?
A. close reduction with cast

🌹 20 y MVA with normal vitals except pulse increased. Which type of shock
A.cardiogenic B.anaphylactic C. hypovolemic ✅ D.neurogenic
.‫ﻟﻠﻣﻌﻠوﻣﯾﮫ اﻟﺳوال ﻛذا ﺟﺎ ﺣرﻓﯾﺎ وﻻ ﻓﻲ ﺷﻲ زﯾﺎده ﻓﻲ اﻟﮭﺳﺗور ﻣﺎﻓﻲ ﺳواﺋل ﻛﺛﯾر ﻓﺎﻟﻧﺑض زاد ﺣﺗﻰ ﯾﺣﺎول ﯾﻌوض اﻟﺟﺳم ﻋن اﻟﻧﻘص اﻟﻲ ﺳﺎر‬
...........
🍒Patient 72 years old have DM,,,For 3 month cant be stand from the chair What can
happen to him ? A-Fall ✅. B-Rta. C-gunshot. D-Fire

🍒Most common site of aortic injury ? Proximal to left brachiocephalic

🌹Blunt truma in chest or thoracic what to injured?


a-Left brachiocephailc b-Aortic arch c-Brachiocephalic something D- left subclavian
artery
….

🌹 Most common site of thoracic aortic blunt trauma :


A. Aortic arch B. aorto-ligament C. proximal to subclavian... artery
D. distal to left subclavian artery
‫ = ﷲ‬subclavian = sub= ‫ ﺻﻌب ﺣﺎدث ﺑﺎﻻورطﻰ ﺷرﯾﺎن ﻣﮭم = ﺻﻌب‬distal = ‫رﺑط = ﺣﺎدث ﻓﻲ اﻻورطﻰ = ﺑﻌﯾد اﻟﺷر ﻋﻠﯾﻛم = ﺑﻌﯾد‬
= left . ‫ﯾﺑﻌدﻛم ﻋﻧﮫ = ﯾﺑﻌدﻛم‬
.....
🌹 Most common injury in blunt chest trauma :
A. aortic arch B. proximal left proximal subclavian artery
C. Left distal medial subclavian artery

🌹after RTA pt hospital is away 40km what would you do??
A-call surgical oncall✅. B-chest tube

🌹A traumatic pt lost 25% of his blood..which of the following is the most suspected to be
effected first A-Urine output. B-Pulse pressure ✅. C-Respiratory Rate. D-Glasco-coma scale
‫ﺪء‬Ç‫ ﻳ‬، ‫ﺔ دم‬‰‫ﺴﺎن •ﻤ‬v‫ اول ﻣﺎ˜ﻔﻘﺪ اﻻ‬، ‫ﺾ‬Ç‫ء ﻳﺘﺎﺛﺮ ﻫﻮ اﻟﻨ‬ÚÛ Û Û
• ‫ﻦ اول‬ç‫ ﻟ‬، ‫ء ﻣﻤﺘﺎز‬Ú
• ‫ و•ﻞ‬å
• ‫ و اﻟﻮ‬±ä‫ﻴ‬Å‫ ﻇﻤﻜﻦ واﺣﺪ ﻓﻘﺪ دم ﻛﺜ‬، ‫ء ﻳﺘﺎﺛﺮ‬Ú
• ‫˜ﻘﻮﻟﻚ اول‬
‫ﻋﺘﻪ ﺣ”“ ˜ﺤﺎول ˜ﻌﻮض ﻋﻦ اﻟﻨﻘﺺ‬ê ‫اﻟﻘﻠﺐ ﻳﺰداد‬

🌹male involved in RTA in ER he is fully conscious and no abnormality in vital signs admitted
for observation and a FAST scan done ,.what do you expect to find
a. pneumothorax b.major vessel bleeding in the thorax c. peritoneum free fluid✅✅

🌹Traum' patient.Intra abdominl hemorrhage and seen injury. Underwent laparotomy and
splenectomy. Which of the following will be low? - Vasopressin. - Insulin✅. - Gluc'gon.
Paintnt ‫ ﻣن ﻣﺿﺎﻋﻔﺎﺗﮭﺎ ﯾﺳﺗﺄﺻﻠوا ﺣزء ﻣن اﻟﺑﻧﻛرﯾﺎس ﻓﯾﺣﺻل ﻧﻘﺻﺎن ﻓﻲ اﻻﻧﺳﻠﯾن‬spleen ‫ ﻓﻠﻣﺎ ﯾﺳﺗﺄﺻﻠوا‬head of pancreas ‫ﻻﺻق ﻓﯾﮭﺎ‬spleen ‫ﻻن ال‬
MVA,hypotensive tachycardia tachypnia can shrug the shoulder but cant

🌹best method to clear cervical trauma in ICU patient after motor vehicle accident
A-clinical judgement B-ap lateral xray. C. CT✅

🌹after RTA pt hospital is away 40km what would you do? ABC .

🌹Case of pt with neck injury after RTA ?? O2 mask. ETT. Coricothyrdoctomy


Remember ABCDE of primary survey.. O2 mask > intubation > chest tube

🌹45 years old man had MVA presented with isolated head injury and coma for 5 days in
ICU , the best way of feeding?
A- Nasogastric tube feedings ✅. B- Gastrostomy feeding C- Central line feeding
D- Peripheral line feeding
🌹Isolated head ‫ اﺳﺎﺑﯾﻊ ﻓﻲ ﻏﯾﺑوﺑﺔ او ﻣﺎﯾﻘدر ﯾﺎﻛل‬٤ ‫ ؟؟ اذا ﻛﺎن اﻛﺛر ﻣن‬Gastrostomy feeding ‫ﻣﺗﻰ ﻧﺳﺗﻌﻣل‬
injury and coma for 5 days = Nasogastric tube feeding

🌹Gastric cancer metastasizing to the liver= chemotherapy .🌹

● Pt e gastric cancer & gross ascites , next step in management :
A. Abdominal paracentesis B. Endoscopy .

🌹 A soldier walks for a long time with pain in his foot And have flat foot which the
tendon affected ? spring ligament

pt fallen down from 3 meter height he felt severe pain and swelling at the lower Rt leg xray
showed commonuted fracture of the lower tibia what is the most accurate management
a. closed reduction, cast and elevation
b.open reduction, internal fixation and elevation ✅✅✅ c.external fixation
.....
Mid shaft Femoral fracture 30 d angulation child 4 years old what management ==
A)_Traction. B)_ ORIF (( open reduction and internal fixation)) C- ORIF with IM nail.
......
🌹 6 years patient with mid shaft femoral fracture, 30% anterior angulation, what is the
management:
A.Reduction with bed rest B.Closed reduction with hip spica cast C. ORIF D.OR and
intra medullary nail !@

🌹Most important thing to do before reduction of fracture))) ==== Check vascular


status(pulse) ✅
×ç‫ع ﻟﻠ‬í‫ﻒ وﻛﺬا ا ﺧﻄﺮ ﻧﻌﻤﻞ رﺟ‬£À‫ﻪ ﻧ‬‰‫ﺔ ﻣﻌﻨﺎﻫﺎ ﻓ‬‰‫ﺔ ﺟﺮاﺣ‬‰‫ﺾ ﻻزم ﻓﻮرا ﻋﻤﻠ‬Ç‫ﺾ ﻻن ﻟﻮ ﻣﺎ „ •¨ ﻧ‬Ç‫ﺪ ﻣﻦ اﻟﻨ‬Þ‫ء ﻫﻮ ﻧﺘﺎ‬ÚÛ
• ‫ ﻻ اﻫﻢ‬، ‫ ﻣﺴﻜﻦ‬ï

• ‫ ˜ﻘﻮل ﻧﻌ‬ð• ‫ﺎر ﺛﺎ‬‰‫اﺧﺘ‬
....... ....... ....... ........
🌹Head trauma post MVC at speed 130 km/hr. Tight seat belt. Patient is stable and alert.
Most appropriate next step in management?
○Abdominal US ○ CTabdomen ✅. Diagnostic laparotomy ○ Exploratorylaparotomy

🌹A scenario of patient with vascular problems in lungs causing him pulmonary HTN, which
class/group of pulmonary HTN ? A. 1. B. 2. C. 3. D. 4. E. 5

🌹Elderly with MVA on exam "Battle sign" what's the diagnosis?


A. fracture of the mandible B. Le froke. C. Basal skull fracture ‫ﻣن اﺳﻣﮭﺎ‬

🌹Case of head trauma, presented with ear bleeding, ruptured eardrum, what would be
the cause? A-Basal Skull fracture ✅ ✔. B-Subarachnoid hemorrhage

🌹 Ear secretion after trauma external ear was intact ? Basal skull fracture
‫ﻟﻤﺎ ˜ﺤﺼﻞ ﻛ× „ •¨ ﻣﻨﻄﻘﺔ‬basal ‫ﻤﺔ‬‰‫ ﻟﺬﻟﻚ ﻟﻤﺎ ﻧﻔﺤﺺ اﻻذن ﻣﻦ ﺑﺮة ﻧﻼﻗﻴﻬﺎ ﺳﻠ‬، ‫ﺎﻻذن‬Ä ‫÷ﺔ ﺟﺎﺋﺖ‬ö„‫ﺲ ﻣﻌﻨﺎﻫﺎ اﻧﻪ اﻟ‬Æ‫ ﻫﺬە ﻟ‬، ‫ﺔ‬‰‫ج ﻣﻦ اﻻذن دم واﻓﺮازات ﺛﺎﻧ‬ª‫ ˜ﺨ‬.

🌹RTA patient injures jugular foramen, what will happen?


A. paralysis of muscles of mastication B. something happening to the vocal cord ✅✅
.‫ ﻓﻼزم ﻧﻠﺑس ﺟﺎﻛﯾت‬vocal cord = ‫ ﺟﺎﯾﻛﯾت = ﻧﻠﺑﺳﮫ ﺑﺎﻟﺑرد = ﻓﻲ اﻟﺑرد ﯾﺳﯾر ﻟﻧﺎ اﻟﺗﮭﺎب ﺑﺎل‬jugular = ‫رﺑط = ﺟﺎﻛﯾوﻻر‬
......
🍓Battle' sign indicate which fracture: = posterior cranial fossa.
🍟🍟 = battle . 😋...... ....‫ = ﻋﺻﯾر راﻧﻲ 🥤 ﻣﻊ ﺑطﺎطس‬cranial = ranial = ‫رﺑط‬

🌹Pt had trauma cause base skull fracture .. jugular foramen:


A. ipsilateral vocal cords injury B. something with mastication muscles

🌹The most probable viral cause of repture of ear drum with abcess or pus ?
A.Rhinovirus B.Adenovirus C. RSV ✅

🍒Case of alcoholic patients with epigastric pain radiating to the back with x ray showing
air under diaphragm= A-acute pancreatitis. B- chronic pancreatitis.
C- duodenal perforation✅

🌹Pt have RTA can shrug shoulder but can’t move elbow and lower limbs ?
High spinal cord injury

🌹 Man who was in MVA, hypotensive with slow heart rate and can't move legs or
hands, what caused his shock? high spinal cord injury
🌹What is the cause of this hypotension? : A-Upper spinal ingury ✅✅ B-Abdominal
bleeding
upper spinal injury :
Patients with upper cervical spinal cord injuries the more dysfunction can occur.:
*Inability to breathe if injury on (C1-C4) ...
*Paralysis in arms, hands.
*Numbness, tingling, or loss of feeling below the level of the injury.
............
🌹Young male, post MVA Opens eyes spontaneously Responds to verbal
commandsShrugs shoulders Shallow breathing Left chest wall
contusion Cannot flex elbows or move lower limbs Respiratory rate: ?
BP: hypotensive Most likely diagnosis:
A. Cardiac tamponade B. Left tension pneumothorax C. High spinal injury
….
🌹Football player received a trauma to lateral side of his left knee, the patient now is
complaining of severe pain and swelling of the medial side of his left knee, positive valgus
and (-) anterior drawer and lachman, most likely diagnosis:
A.Medial meniscus tear B.Lateral meniscus tear C.Medial collateral ligament sprain
D.Lateral collateral ligament sprain !@

🌹Player professional football with knee Injury in lateral side , medial knee
swelling , lachman and MCmurray test ( negative) ?
A. Medial meniscus tear B. Lateral meniscus tear C. Medial collateral ligament
Pt came with instable knee. Ex femur come in front of tibia, which ligament
injured ? A. Pcl B. anterior cruciate ligament (ACL)
1Thyroid
🍓After total thyroidectomy hypocalcemia what to do?!
A. Give PTH* B. give potassium C. measure magnesium D. give levothyroxine

🍓Patient admitted for thyroid surgery because of progress compression symptoms


Hemithyrodectomy done and biopsy showed 8mm papillary carcinoma next? radio
ablation B. no need for further intervention C. complete thyroidectomy. D-follow up

🍓After right hemithyroi dectomy On biopsy found follicular lesion 8cm from the
original. Mx? A. Radio iodine B. Followup C. Complete thyroidectomy!@
‫ﺎﻟﺠﺮاﺣﺔ‬Ä ‫ﺜﺔ ﻣﺎﺷﻠﻨﺎە‬Å‫ﺔ ﺧﺒ‬‰‫ﻘ‬Ç‫ﻪ ﺧﻼ˜ﺎ ﻣﺘ‬‰‫ﻤﺎوي او رادﻳﻮا ﺣ”“ ˜ﻘﺘﻞ ﻟﻮ ﻓ‬‰‫ﺚ ﻻزم ˜ﺎﺧﺬ ﻋﻼج ﻛ‬‰‫اي ورم ﺷﻠﻨﺎە و•ﺎن اﺣﺘﻤﺎل ˜ﻜﻮن ﺧﺒ‬

🍓After thyroidectomy history follicular ca small with lesion 8mm from the lesion
what will do :A. Total thyroidectomy B. Iodine scan C. Radio ablation * D. Flow up

🌹Pt for thyroidectomy and with mitral valve prolapse?


A. Prophylactic for endocarditis B. Cefazolin for surgical site infection
C. No need D. Prophylactic for endocarditis and surgical infection

🌹neck lump with hot nodule = hemithyroidectomy

🌹Young male with midline neck mass that ascends and descends with swallowing,
diagnosis:Thyroglossal cys✅ A. Cystic hygroma B. Midline dermoid cyst

🌹pt after thyroidectomy , develop hypocalcemia despite calcium replacement more than
one time , next : check serum magnesium level

🍓A patient post thyroidectomy can not make high pitch sounds. Damage to which
nerve is responsible? A. Inferior laryngeal nerve. B. Recurrent laryngeal nerve. C.
Glossopharyngeal nerve. D. Superior laryngeal nerve ‫ ﺗﻘﻊ ﻓﻮق‬.‫اﻟﻐﺪة‬

........
Pt came after thyroid surgery with dysphagia and horseness of voice. Which
nerve injured? A. Recurrent. B. Internal. C. External

🌹Patient with hyperthyroidism for 10 m and treated with anti hyperthyrdism


But his condition did not improve ultrasound done and show multi nodule and
diagnosed as Graves’ disease what is the best next step ? A.total thyroidectomy
B.subtotal thyroidectomy C. iodin therapy ✅ D.increase the dose of drug
Treatment Graves’ disease
First start with propylthiouracil (PTU) or methimazole (MMI)
PTU also inhibits peripheral deiodination of T4 to T3
• symptomatic treatment with `-blockers
• thyroid ablation with radioactive 131I if PTU or MMI trial does not produce disease
remission
• subtotal or total thyroidectomy (indicated rarely for large goitres, suspicious nodule for
CA, if patient is intolerant to thionamides and refusing RAI ablation)

🌹Neck mass biopsied showing (thyroid folicullar cells)? Ectopic thyroid ✅


J ‫ اطﻠب ﺳﺎﻧدوﯾﺗش ﻓول ﻣن ﺑرة‬follical = ‫ﻓول‬
🌹thyroid nodule hard ultrasound report Next step? A.FNA ✅. B.lobectomy

🌹Question about pt with symptoms of hyperthyroidism and he is on medical


treatment for 10 months with no improvement what next!@
A-Total thyroidectomy B-Partial C-Radioactive iodine✅✅ ‫ اﻟﺤﻮاﻣﻞ‬û‫ع ﻋ‬í‫ﻟﻮ اﻟﺤﺎﻟﺔ وﺣﺪة ﺣﺎﻣﻞ ﻻ ﻧﺨﺘﺎرە ﻻن ﻣﻤﻨ‬

🌹Thyroid enlargement (hot nodule) FNA normal What is next step?


A- thyroid scan B- repeat FNA C.start antithyroid therapy
....
🍓Thyroid nodule, cold, investigation? A. fna
……
Patient with thyroid nodule asymptomatic And thyroid function test all within normal
US done What’s next:
A. FNA.(bez function thyroid normal) B. Start antithyroid medication. D. thyroid scan
Answer is: A
🌹 Pt with thyroid nodule and hyperthyroidism lab , radioactive iodine shows the nodule
hot, but all the remaining thyroid is cold. Initial Management:
A. hemithyroidectomy B. tota ectomy C. antihyperthyroid drugs D. radioactive ablation
....

🌹 Hashimoto’s increases risk of: A. thyroid lymphoma B. papillary thyroid


...
Pt with hx of hashimoto’s thyroiditis for years. Presents with of thyroid mass
around 2x3, what is it? A. Papillary cancer B. Thyroid lymphoma C. Subacute thyroiditis

🌹A patient post total thyroidectomy developed neck swelling 5 hours after the
surgery. most appropriate management?
A. tracheostomy B. bedside evacuation C. percutaneous aspiration D. observation
....
🌹 27 yrs female c/o neck pain and tender thyroid, hx URTI weeks ago, sx of
hyperthyroidism, mx? A. Methimazole B. PTU C. Thyroid scan
.N.B: In another recall there was 4th choice propranolol if was there choose it
hot or cold.....‫ ھل ھﻲ‬nodule ‫ ﻓﻲ اﻟﺛﺎﯾروﯾد ﻻزم ﺑﻌدھﺎ ﻧﻌﻣل ﺛﺎﯾروﯾد ﺳﻛﺎن ﻋﺷﺎن ﻧﺷوف ھذه‬nodule ‫اذا ﺷوﻓﻧﺎ‬

🌹Medullary thyroid cancer mange? Total thyroidectomy

🌹2 cases about asymptomatic neck node lymph node, no thyroid signs , normal labs, FNA :
normal follicular thyroid cells the dx: A. ectopic thyroid B. lymphoma C.Metastatic CA
.‫ ﻟﻛن ھﻲ ﻟﯾﺳت اﻟﻐدة اﻟﺣﻘﯾﻘﺔ ﻟﻛن ﺳرطﺎن ﯾﺷﺑﮫ ﺧﻼﯾﺎ اﻟﺛﺎﯾروﯾد ﻓﻼزم ﻧﺳﺗﺎﺻﻠﮫ‬thyroid ‫ اﻋرﻓوا اﻧﮫ ﻓﯾﮫ ورم ﺧﻼﯾﺎه ﺗﺷﺑﮫ ﺑﺎﻟﺿﺑط‬normal cell folical ‫ﻟﻣﺎ ﯾﻘوﻟك‬

🌹 normal follicular thyroid cells , how to manage :


A. repeat FNA B. reassure C. follow up clinic D. hemithyroidectomy
....
🍓female , with lateral mass (lymph node) , thyroid was normal , biopsy showed normal
follicular cell, what is the diagnosis? follicular metastasis
..
🌹Normal thyroid, cervical lymph node bx showed normal follicular thyroid cell dx?
A.Ectopic thyroid B.follicular carcinoma C. lymphoma D. apparent ✅✅
..
pt with left Lateral neck mass ((third triangle just below angle of mandible)) ... U/S thyroid
normal and post. Cervical lymph node enlargement, on specimen it’s shows follicular
thyroid cells = A- thyroid carcinoma/ or lateral aberrant. B- apparent thyroid gland.✅

🍓Case of nodule in the cervical LN show follicular cell :


A. aberrant✅ B. ectopic

🌹 Pt with normal thyroid and swelling in the neck this swelling is cervical LN and FNA
showed normal follicular thyroid tissue :
_Remove this LN. _Refer to surgery ✅ ✔. _Radio. _Chemo

🌹 Thyroid nodule measuring 4 cm hemithyroidectomy


....
🌹Thyroid nodule measuring less tnsn 1 cm =follow up

🍓All things is normal but have pain in his nick they chick his left neck and found small mass
5*7 mm what is the most important thing to do :
A. FNA. B. complete his thyroid investigation✅
......
🍓medullary thyroid carcinoma management? total thyroidectomy
🍓hematoma after thyroidectomy. Next? A. OR. B. bedside evacuation✅
........
🌹Patient post thyroidectomy, during the ward he has expanding neck swelling with
difficulty of breathing what to do?
A. Immediate bed side opening of wound B. neck US C. take to OR

🌹High TSH , low T4 ,T3 , high ESR diagnosis? Subacute thyroiditis 🌹


….
🌹Complication after thyroid surgery > nerve injury ( Hoarseness of voice = recurrent LN )
....
🌹Insufficient thyroid FNA, what would you do next?
• thyroid scan. • repeat FNA✅
When to repeat thyroid fine needle aspiration cytology?
The aim of of the study was to investigate the possible changes of primary fine needle aspiration (FNA) diagnoses after
subsequent check ups. We investigated 948 thyroid nodules and the main indications for repeat FNAs were
inadequate/indeterminate FNA findings and growing nodules at ultrasound check up. FNA findings were subdivided
into inadequate, benign, low-risk lesion
.......
🌹Strongest indication of thyroid surgery
A. -pediatric B. -presence of eye symptoms. C. -presence of anti tsh . D. -failed antithyroid meds.

......
🌹pt with hyperthyroidism feature and irregular irregular pulse first test?
Thyroid function tests
….
🌹Thyroid Bethesda IV management? A.Total thyroidectomy B.Hemithyroidectomy ✅
‫ = ر÷ﻂ‬beth = ‫ﺖ‬Æ‫ = ﺑ‬hemi = home .
🍒25 years female with thyroid nodule TSH and T4 normal , FNA cytology done and
according to Bethesda classification Stage IV what is the most accurate management ==
LOBECTOMY
🍒25 years female with thyroid nodule TSH and T4 normal , FNA cytology done and
according to Bethesda classification Stage III what is the most accurate management =
repeat FNA and observation
...... ....
🍒 same question and same choices but besthesda Stage VI = Near total thyroidectomy
....
=‫رﺑط‬
🍒 Stage lV =LOBECTOMY
V‫ ﺑﻌدﯾن ﯾﺟﻲ‬I ‫ اﻟﻲ ھو اول ﺣرف ﻣﻊ أول‬L ‫ﺣرف‬
🍒 Stage III=repeat FNA and observation
‫ﻠﻤﺔ‬¥ repeat ‫ ﺣﺮف‬I ‫ﻣﺘﻜﺮر ﺛﻼث ﻣﺮات‬
🍒 Stage VI= Near total thyroidectomy= ‫ﻞ اﻟﻐﺪة •ﻠﻬﺎ‬‰‫ﺸ‬v ، ‫ة‬±ä‫ اﻟﻤﺮﺣﻠﺔ اﻻﺧ‬.
🌹Patient with large left thyroid mass, FNAC shows follicular cells(bethsada 4) what is the
management? A. Repeat FNA B. left lobectomy C. total thyroidectomy

🍒 unilateral neck swelling in the RT side by investigations : hot thyroid nodule TSH is high,
T3, T4 low No LN enlargement Treatment?
A- antithyroid drug✅. B- RT thyroidectomy. C- Hemithyroidectomy. D- radioactive iodine
...
🍒 Patient came for 3 months neck pain, thyroid function test all normal, ultrasound:
single solid thyroid mass in right lobe. Best next management ?
A.Thyroid scan. B.FNA✅. C.Imaging Follow up. D.Right lobectomy.
.... .... ....
🍒 neck mass, pt had hashimotos thyroditis, found malignant cells, dx:
A- follicular. B- medullary. C- anaplastic. D- lymphoma ✅✅
Hashimoto's thyroiditis may increase the risk of developing a rare type of cancer called
thyroid non-Hodgkin's lymphoma
. . . .. . . .
🍒 Euthyroid thyroid nodule, what you will do? Fine Needle Aspiration ✅ ✔
Fine needle.. .. . .. . .‫ ﺑﻣﺎ اﻧﮫ طﺑﯾﻌﻲ ﻧﺣﺗﺎج اﻟﻰ اﺑرة رﻓﯾﯾﯾﻌﺔ ﺟدا‬، ‫ ﯾﻌﻧﻲ وظﺎﺋف اﻟﻐدة طﺑﯾﻌﯾﺔ‬: Euthyroid ‫رﺑط‬

🍒 pt with enlarged lymph node and normal thyroid FNA of LN showed norm'l follicul'r
thyroid tissue : Abx. B- Remove the LN C- Refer to surgery ✅

🍒 toxic nodule and rest of the gland is suppressed


A-Radio ablation B-Thyroidectomy
If small go for radio If large or cuusing compression, go for hemithyroidectomy
.. . . . . . .
🌹Patient with congested throat symptoms and 2cm palpable cervical lymph nodes. What is
the most appropriate investigation?
Chest x-ray. Neck CT. Lateral neck x-ray. FNA of the lymph nodes✅ N.B: FNA if there’s
worrisome features:- 6 weeks without resolution + lymph node more than 2 cm in siz

🌹Patient in his fifties K/c of hashimotoʼs thyroiditid for years Presents with neck
mass, it is engulfing carotids , invading capsule bx showed malignat thyroid what is the
type: Medullary Follicular Lymphoma ✅ Anaplastic

🌹Pt with diffuse thyroid enlargement on exam there is 1 nodule in each lobe, labs showed
hyperthyroidism what will you do? A. Thyroid scan. B. FNA from both nodule
….
🌹 48 yo lady with diffuse goiter, high T4 low TSH, US show bilateral thyroid nodules , right
3x4 in size , left is 1x2 size what to do? A. FNA both B. FNA the larger one C. total
thyroidectomy!@

🌹Patient with spiral fracture, parathyroid hormone high, ca high.. Most likely diagnosis?
a) Parathyroid adenoma ✅🌹 b) Parathyroid carcinoma ❌
….
🌹A guy came with swollen legs with after hitting something. Examination and imaging
showed spiral thigh fracture with reabsorbed periosteal something? Labs
showed high PTH and Ca. Asking about the disease == Primary hyperparathyroidism
...........
🌹parathyroidectomy pt have increase in ca ? Missed adenoma
….
🌹 Indication of elective parathyroid gland removal?
A. Age > 50 B. Evidence of osteoporosis.
.....
After near total thyroidectomy the patient have persistent calcium decreased despite
giving him calcium multiple times. What to give him ? Mg

1Post ope
🌹 Pt Postoperative blood transfusion develope fever and pain at site of infusion
A. febrile non hemolytica reaction B. hemolytic reaction C. bacterial contact

🌹 Pt post op triple A repair become unstable even with fluid replacement and
have negligible urine in cath decreased vascular resistance and increased
cardiac out put.Type of shock ?
A. cardiogenic B. Hemorrhagic C. anaphylactic. D. septic

🌹 female had a hip fracture now before 2 weeks I think she had a surgery.with a
complication of DVT as I remember now she is on heparin but her platelet is low 58 so
what will you do?
A. Shift to SC enoxaparin B. Stop heparin and start other anticoagulant✅

....
🌹 Patient morbid obesity for treatment what will u do before?
A. endoscope B. ultrasound C. CT D. barium swallow

🌹Pt with progressive abdominal pain post sleeve, what to do:


A. Laparoscopy B. CT C. Exploration
....
🌹 POST bariatric 3 months with worsening abd pain : A. us B. Endoscopy C. Laparotomy

🌹 pt. with heart failure and admitted for surgery for some disease and connected
to IV fluid, post-op 2 days later complained of SOB and bilateral basal.crepitation, how
could this be prevented:
A. IV Furosemide immediate post-op B. monitoring IV fluids daily✅
‫• ﻻن ﻓﺸﻞ اﻻﻋﻀﺎء ¿ﺬە ﺻﻌﺐ ﺟﺪا ﺗﻄﻠﻊ اﻟﻤﺎء‬x‫ﺎﺧﺬوا ﺳﻮاﺋﻞ ﻛﺜ‬€ ‫ﻮا او‬š·¸¶ ‫ اﻧ´ﺒﻬﻮا‬²±‫ﻣﺮض ﻣﺜﻞ ﻓﺸﻞ اﻟﻘﻠﺐ واﻟ‬
‫ﺔ اﻟﺴﻮاﺋﻞ‬Ì‫ﻤ‬¥ ‫ﻓﺮاﻗﺒﻮا‬،‫ﺎﻟﺠﺴﻤﻢ‬‰ ‫ﻢ‬Ê‫•ا‬É‫ﺑﺮة اﻟﺠﺴﻢ ﻓﻴ‬
...

🍓Scar post surgery increasing in size?Keloid

🍓during laparoscopy doctor just start the procedure pt become hypotensive 84/50 what’s the
cause:
A. cold gas B. increase preload C. rapid inflation of the abdomen✅
Peritoneal stretching > vagal stimulation

🌹 Most common complication post operative?-


wounds infection. - pneumoni'. -Atelectasis ✅

- 24 hrs postop patient has dyspnea and hypoxia cause?


a. lower lobe atelectasis ✅b. PE

.....
🌹patient with abdominal mass after lifting heavy objects mass not change with cough. Dx?
a) rectus sheath hematoma b) hernia

🌹Pt 56 c/ o sudden RLQ pain after lifting heavy object ; and mass in RLQ ; Cough
negative ; abd muscle tense even with clenching . He is on anticoagulation due to
A fib What the most appropriate mgx ? (Rectus sheath hematoma)
A. angioembolization and .. B. rest and analgesic and stop anticoagulants

. . . . . .. . .
🌹Morbid obese pt with GERD wich procedure is good for him ?
A. Sleeve gastrectomy B. Biliary C. Roux y

🌹13yo obese boy found to have large hiatal hernia and grade 3 reflux asking
about bariatric surgery choice: A. Roux-en-Y Gastric Bypass B. Sleeve C. Balloon

🌹Obese male tried lifestyle not effective He is diabetic hypertensive what to do , Weight
125 ,Hight 173 , What next ? ( You should calculate BMI )
A. Medication lower weight B. bariatric surgery C. Exercise D. diet

🌹Women obese HTN diabetes controlled BMI 28 Tried excersise and lifestyle
modification but didn’t work What to do?
A. Bariatric surgery B. Orlistat (druge for obsity)

🌹Case of pt with constipation for 1 week then developed watery diarrhea,


he has tender abdomen and small distention = A. Constipation B. Acute colitis
🌹29 Years old male did sleeve gastrectomy 6 days ago, came to the ER complaining of
persistent vomiting since operation, on exam nothing significant,ABG normal( there were
numbers),Us normal. Best management?reassure
‫ﻄﻨﻪ او‬Ä ¨• „ ‫ﻨﻤﺎ ﻟﻮ ﺣﺼﻞ ﻟﻪ اﻟﻢ ﺷﺪ˜ﺪ ﺟﺪا‬Å‫ﺾ ﺑ‬£ª‫ ف ﻧﻄﻤﻦ اﻟﻤ‬þ•‰‫ف ﻫﻨﺎ اﻻﻋﺮاض ﻃﺒ‬. ‫ء وﺗﻌﺐ وﻫﺬ˜ﺎن‬Í• ” ‫ﻢ اﻟﻤﻌﺪة ˜ﺤﺼﻞ ﻫﺬە اﻻﻋﺮاض وﻫﻮ اﺳﺘﻤﺮار اﻟ‬‰‫ﺎت ﺗ›ﻤ‬‰‫ ا ﻣﻊ ﻋﻤﻠ‬þ•‰‫ﻻن ﻃﺒ‬
Í„‫غ دم ﻫﺬا ﻣﺎﻳﻨﻔﻊ ﻧﻄﻤﻨﻪ ﻻزم ﻳﺮوح ﻟﻠﻤﺴ"ﺸ‬ª‫ اﺳﺘﻔ‬.
-- ----- ----- -----
🌹Diabetic and HTN .., BMI > 40 (you calculate .. height and weight were given) .. he tried
to reduce weigh but still obese .. he is not on medications .., NEXT:
A- Medications reduce weigh B- Sleeve surgery ✅✅. C- Diet D- Exercise
‫ﻨﻤﺎ‬Å‫ ﺑ‬، ‫ول‬±”‫ﺴ‬Æ‫ و'ﺸﻜ ” •“ ﻣﻦ اﻋﺮاض اﻟﺴﻤﻨﺔ وﻣﻀﺎﻋﻔﺎﺗﻬﺎ ﻣﺜﻞ ﺳﻜﺮ وﺿﻐﻂ و•ﻠ‬35 ‫ او اذا ﺷﺨﺺ ﻛﺘﻠﺔ ﺟﺴﻤﻪ ﻓﻮق‬، ٤٠ ‫ﺔ اﻟﺴﻤﻨﺔ ؟؟ اذا ﺷﺨﺺ ﻛﺘﻠﺔ ﺟﺴﻤﻪ ﻓﻮق‬‰‫ﺷﻮﻓﻮا ﻣ”“ ﻧﻌﻤﻞ ﻋﻤﻠ‬
‫ ﻧﺨﺘﺎر‬٣٥ ‫ ﻟﻮ اﻗﻞ ﻣﻦ‬A
-- ----- ----- -----
🌹Patient done gastric sleeve after 3 days come irritable and have some blotting
what to do? A-upper endoscopy B-laparotomy C-stool analysis D-reassurance ✅

🍒-Patient morbid obesity for treatment what will u do before?


a. endoscope b. ultrasound c. CT d. barium swallow

🌹70 year old pt fall on the ground surgeon will do hip replacement surgery before surgery
procedure what is the best prophylaxis thrombolytic drug for him ? LMWH , enoxaparin .
🌹After surgery by 4 days patient developed MI .. what to give ? angioplasty
thrombolytic .‫ ﻟﻮ ﻣﺮ ﻋﻠﯿﮭﺎ اﯾﺎم ﻧﻌﻄﯿﮭﻢ ﻣﺬﯾﺐ ﻟﻠﺠﻠﻄﺔ‬، Angioplasty ‫ او ﯾﺴﻤﻮھﺎ‬PCI ‫ دﻗﯿﻘﺔ ﻧﻌﻤﻞ ﻟﮭﻢ ﻗﺴﻄﺮة اﻟﻲ ھﻲ‬90 ‫ﻟﮭﺎ‬MI ‫ﻟﻮ ﻣﺪة ال‬

🌹Pt with pelvic fx and bleeding per rectum >> urethrogram > retroperitoneal
urethra injury Mx?
A-Folly's cath B-Supra pupic cystostomy C-Labroscopic repair

🌹Membranes urethra injury, management?


A. Fully Catheter B. Retropubic catheter C. Retropubic repair D. Penial repair
🌻 Membranous urethra injury is posterior injury
/….
Old patient had percutaneous cholecystostomy, presented after 24hrs with
upper GI bleeding what is the most important investigation?
A. angiography. B. Endoscopy. C. Ultrasound D. CT (they didn't mention w contrast)!@
Pseudoaneurysm of hepatic artery is a rare but known complication of laparoscopic cholecystectomy (LC). Such
pseudoaneurysms may bleed in biliary tree, upper gastrointestinal (GI) tract or peritoneal cavity leading to life-
threatening internal haemorrhage. It is very rare for them to present as lower GI bleeding.
……..
Female with thyroid mass asymptomatic us shows hard nodule what are the next steps?
A. Thyroid scan with iron B. Fna C. Ct D. Mri

1Hydatid cyst
🌹Echinococcus syptomatic calcified Rx? A.Surgery(cystopericystectomy)
✅🌹 B.Metronidazole C.Aspiration

🌹 Case of liver cyst (echinoccosis) treatment ?. Albendazole ✅ ✔
(‫= اذا )ھﺎي ﻣﻊ ﻗﻠب‬. Hydatid ‫ ﻣن اﺳم‬+ Hy= ‫ ﻗﻠب‬، ‫ = أﻟب‬Albendazole = ‫ﻧرﺑطﮭﺎ‬
🌹 hydatid cyst 10*15 cm with multiple daughter cyst inside the main one.
What is the treatment? A. Albendazole B. PAIR. C. deroofing surgery
....
🌹Picture of Ct hydatid liver cyst: One large, manage: A. open cyst deroofing ✅B.
albendazole
.‫ اﻟﻣﺗﺟﻣﻊ داﺧﻠﮭﺎ‬pus ‫ ﯾﻌﻧﻲ اﻓﺗﺢ اﻟﻛﯾس ﺑﺎﻟﻣﻧظﺎر واﺷﯾل‬deroof ‫ ﻟﻛن ﻟو ﻛﺑﯾر ﻻزم اﺷﯾﻠﮫ ﻛﻠﻣﺔ‬، ‫ وھو ﯾﺧﺗﻔﻲ ﺑﺎذن ﷲ‬alben ‫ ﺻﻐﯾرة ﻧﻌطﯾﮫ دواء‬cyst ‫ﻟو ﻛﺎن‬

🌹Pic of Hydatid cyst on U/S it was big but daughter cysts was mentioned. What should
you do: Surgical Deroofing
‫ة‬±ä‫ = دﻳﺮﻓﻴﻨﺞ =ﻇﻔﻴ‬daughter.
....
amebic liver disease, what drug to give: Metronidazole
‫وا ب اﻟﻤﻴﺒﻮا‬±”‫ اﻟﻤ‬Í• „ ‫ﻖ ﻧﻄ‬£ª‫ﻪ ﺣ‬‰‫وا ﺣﺼﻞ ﻓ‬±”‫ و ﻣ‬، ‫ﻢ ﺣﺮوق‬£ª‫ ﻣﻴﺒﻮا ﻛ‬، ‫🔥🔥 ر÷ﻂ‬
..... .....
🌹 Classic amebic abscess scenario with clinical and imaging findings. Management?
○ Metronidazole ✅. ○ Drainage. ○ Aspiration
-- ----- ----- -----
🌹 Long scenario with liver abscess culture reveals anti amoeba histolytica treatment?
A-liver transplant B-metronidazole ✅✅. C-drainage ‫ء ﻣﻀﺎد ﺣﻴﻮي و÷ﻌﺪﻳﻦ دراﻳﻦ‬Ú Û
• ‫اول‬
M=M Firstly metronidazole then drainage
.......
🌹Large cyst >10cm or complicated:- Surgery 🌹

🌹Patient with hydatid cyst, complex with septations and 10x15 in size, initial step in
treatment?
A. Cephalosporin B. Metronidazole C. Percutaneous aspiration D. Percutaneous drainage

1Operative
🍒Indication of surgery for Aortic stenosis?A. pt symptoms EF less than 50

🌹after surgery what is best dvt prophylaxis? enoxaparin and mechanical

🌹 lady was admitted and discharged just a day before she experienced severe right leg
pain and edema. (Basically she was discharged and the next day she got this presentation)
diagnosis:A. DVT B. Acute thrombotic disease C. Acute embolic disease

🌹 Patient has resection rectal surgery .. next days has left leg pain what do you give ?
Enoxaparin

🌹 Patient was stabbed in the right lower side of his chest, FAST revealed free abdominal
fluid = A-Thoracotomy B-Laparotomy ✅✅. C-Tube thoracostomy
Laparotomy ‫ ﻋﻣﻠﯾﺎت ﻻن ﻣﻌﻧﺎھﺎ ﻓﯾﮫ ﻧزﯾف داﺧﻠﻲ ف ﻧﺧﺗﺎر‬OR ‫ﻓورا ﻋﻠﻰ‬FAST ‫ ظﮭر ﻓﻲ ا‬free abdominal fluid = ‫اول ﻣﺎ ﻧﺳﻣﻊ ﻛﻠﻣﺔ‬

🌹 pt complain of malena and epigastric pain diagnosed as peptic ulcer and received ppi but
not improved endoscpy done showed multiple ulcers in the antrum what is the most
accurate management : a.antrectomy✅. b.partial gastrectomy c.total gastrectomy
antrum ‫ ﻓﻲ‬ulcer ‫ ﻓﺎﻟﺳؤوال ھﻧﺎ ﯾﻘول اﻧﮫ ﺣﺻل‬antrum ‫ ؟ اﻟﻣﻌدة ﺗﻘﺳم اﻟﻰ ﺛﻼث اﺟزاء اﺧر ﺟزء ﻣن اﻟﻣﻌدة اﺳﻣﮫ‬antrum ‫ﺗﻌرﻓون اﯾش ھو ال‬
.antrectomy. ‫ ﻋﻼﺟﮭﺎ‬antrum ‫ﻓﺎﻟﻌﻼج اﻟﻘرﺣﺔ ھذه ه ﺑﺎﺳﺗﺋﺻﺎل اﻟﺣزء اﻟﻲ ﺣﺻل ﻓﯾﮫ ﻗرﺣﺔ وﻣن اﺳﻣﮭﺎ‬
Patient known case of peptic ulcer disease in medications but failed to respond, uppe endoscopy done and revealed multiple
ulcers in antrum. What is the best treatment?
A. Pyloroplasty and vagotomy✅✅✅ B. Total gastrectomy C. partial gastrectomy

Vagotomy was a way to reduce the acidity of the stomach, by denervating the parietal cells that produce acid. This was done
with the hope that it would treat or prevent peptic ulcers. It also had the effect of reducing or eliminating symptoms of
gastroesophageal reflux in those who suffered from it.🍬

🌹 anterior abdominal stab wound , the omentum is bulging out through wound ?
Exploratory laparotomy✅
..... .. . .. .. . . . .
🍒 Patient with right lower chest stab wound. Fast showed free abdominal fluid. What is
your next step = exploratory laparotomy ( Not chest tube) 🍒🍒
‫ ﻋﻠﻰ طول ﻋﻣﻠﯾﺎت ﻻن دﻟﯾل ﻋﻠﻰ وﺟود ﻧزﯾف داﺧل‬free fluid by FAST ‫اول ﻣﺎﻧﺷوف ﻛﻠﻣﺔ‬
.......
🍒🍒Pt diabetes pregnant and baby has diaphragmatic hernia and baby driver and do for
him resuscitations and stabilization what next step = surgery🍒🍒
….
Pt is with non-acth dependent cushing for rr adrenalectomy ..what is the
postoperative management ?!@
A. postoperative fludrocortisone (my answer not sure )
B. postoperative metatone ?
C. preoperative hydrocortisone
D. pre operative (drug for pheochromocytoma forget its name )

1Peripheral artery dis


🌹Acute limb ischemia with absent distal pulse, diminished sensory and
altered motor ex, mx? A. Embolectomy. B. Above knee amputation. C. Cath and thrombolysis

🌹 pain ,paresthesia ,pallor ,,Hx of MI before weeks ,Dx?
A. Acute arterial thrombosis B. Acute arterial embolism C. DVT!@

🌹An elderly man came with lateral malleolus ulcer, he is hypertensive but
not diabetic What is the first thing to do A. venous duplex B. check pulse C. ABI

🌹Sudden right lower limb pain. Diminished sensation + altered


motor. Pt on anticoagulant for afib.Best next step:
A. CTA B. Us C. emblectomy

🌹What is the best to confirm arterial disease ?


A. Catheterization B. Ct angio C. Conventional ct

🌹Patient with HTN and diabetes came with ingrown toe : what is the most important thing
to do before the surgery :A- check pulse B- check the other toe

🌹 Old male with htn and dm presented with claudication examination reveals
right femoral intact pulse and diminished popliteal and distal pulse and left
diminished pulse intervention?:
A. CT angio B. conventional angio C. mra D. venous US

🌹Old male with htn and dm presented with claudication examination reveals
right femoral intact pulse and diminished popliteal and distal pulse and left diminished
pulse whats the most appropriate intervention:
A. ct angio B. conventional angio C. mra D. venous US

🌹 50 years old patient, know case of DM, HTN presented with leg pain associated
with hair loss , ABI more than 0.9, what is your diagnosis:
A. acute thrombus ischemia B. Chronic limb ischemia
🌹Diabetic patient with pseudo hyper epithelialization in situ= ulcer Debridement

🍒lateral malus ulcer with abaent pulse most IMP investigation?A. CTA✅ B. venous
doppler

🌹Dm pt with unilateral leg erythema Increases in dependant position, Cold, Femoral pulse
is present, distal pulse can be palpated No tenderness, no swelling, no fever Otherwise
unremarkable Dx?
A.Cellulitis B. Arterial insufficiency ✅✅ C.Superficial thrombophlebitis

🌹Patient with diabetes and hypertension smoking 40 c per day came to er complaining of
leg pain on examination on lower limp the pulse was intact on femoral and pupletial
artery’s and diminished on distal pulse first question was what is the best appropriate next
step ? A. morphin B. heparin ✅ C.ct angio D.vascular us

🌹 Pt have deep thigh injury what to do =A.torniquete B. put pressure above site of
injury ✅ C.put pressure on injury

🌹Pulse abdomen mass in middle abdomen what next step to confirm management ?
A. angiography B. CT C. US ✅
N.B: dx is abdominal aortic aneurysm intial US, confirmation and management plan
according to CT angio

🌹Pulsatile abdominal mass Abd X ray showed no air level What investigation to
order:A. abd UC B. CT angio

🌹60 years old patient come with epigastric pain radiating to back . He’s smoker and long
standing for DM and HTN . Abdominal examination show pulstile
subra umblical mass . What’s Dx ?
A. Abdominal aortic aneurysm B. secondary liver metastasis C. renal cell carcinoma

🍒Surgery was done for diabetic septic foot ,no wound healing for 2month what is the
cause? A.Neuropathy B. Poor blood supply ✅✅

🌹PT admitted due to MI after 2 days of discharge developed sever pain in his leg Dx?
1/ acute arterial thrombosis 2/ Acute Arterial Embolus ✅ 3/ DVT

🌹 claudication with distance peripheral artery disease. ABI = ankel brachio index🌹

🌹 what is the first symptom to be find in compartment syndrome =


a.pain ✅. b.parasthesia between toes. c. swelling

🌹 Patient with thigh hematoma, which of the following will indicate surgery?
Pain✅. Caludication. Cosmetic Congestive HF
‫ وھذا ﻣﻣﻛن ﯾﺳﺑب‬thigh hematoma ‫ﯾﻘوﻟك ﻋﻧده‬
Compartment syndromes = Due to bleeding will compresstion to Tissue and blood vessle .
‫ طﯾب ﻣن ﻋﻼﻣﺎت‬، ‫ ﻗﺑل ﻣﺎ ﺗﺗﻛور اﻟﺣﺎﻟﺔ وﯾﺣﺻل ﺑﺗر‬، ‫ ﻻزم ﻓورا ﯾروح ﻟﻠﻌﻣﻠﯾﺎت‬Compartment syndromes ‫ف ﻟو ظﮭر اﻋراض و ﻋﻼﻣﺎت‬
‫ ﻓﺎﻟﺟواب ھﻧﺎ ﻣﺗﻰ اطﻠﻌﮫ ﻟﻠﻌﻣﻠﯾﺎت ﻟﻣﺎ‬، ‫ وﻛﻣﺎن ﺗﺧدﯾﻰ وﺗﻐﯾر اﻟﻠون واﺷﯾﺎء ﺛﺎﻧﯾﺔ‬، ‫ھو اﻟﺷﻌور ﺑﺎﻻﻟم وﯾﻛون اﻟم ﺷدﯾﯾﯾد‬Compartment syndromes
‫ﯾﺣس ﺑﺎﻻﻟم ﻻن اﻻﻟم ﻣن اﻟﻌﻼﻣﺎت اﻟﮭﺎﻣﺔ ل ﺑداﯾﺔ ﺣﺻول ﻛوﻣﺑﺎرﻣﻧت ﺳﯾﻧدوم‬

🌹 Crush injury to his feet , after stebalization of the patient , o/e cold and
decreased pulse in the affected foot . What is the best next step u will do ?
A. ct angio B. angio C. duplex US D. compartment compression

🌹Patient came after limb trauma, with severe pain and paresthesia between his toes and
it was pale. X ray shows fractures, intracompartmental pressure was 35mmhg what to do?
A. Internal fixation B. closed reduction C. external fixation
D. external fixation with 4 fasciotomy
....
🌹pt involved in RTA stable, but there is left leg swelling and paresthesia between toes xray
showed fracture tibia pressure in posterior leg compartment is 35 mmHg :
a.internal fixation b.external fixation with multiple fasciotomies ✅✅
c.internal fixation with multiple fasciotomies
….
9/ pt involved in RTA stable, but there is left leg swelling and parasthesia between toes xray
showed fracture tibia pressure in posterior leg compartment is 35 mmHg :
a.internal fixation. b.external fixatin with multiple fasciotomies
c.internal fixatin with multiple fasciotomies✅
….
🌹 30 years old had trauma which resulted in fracture of his right tibia and fibula After a while he
started to complain of numbness and severe pain within that area What would you do?
A.Heparin B.Embolization C.Fasciotomy ✅✅

🌹 wi th Reversible Eye manifestation for 20 min? TIA ✅ ✔ transient ischemic attack


‫ ﯾﻌﻧﻲ ﻋﺎﺑر ﺷﻲء ﻣؤﻗت وﯾرﺟﻊ ﻟﻠطﺑﯾﻌﻲ‬transient ‫ﻣن اﺳﻣﮭﺎ‬
🌹 Female patient after cryotherapy for varicose 2years ago complainig of leg and
foot pain what nerve is damaged A- siatic. B- saphenus ✅. C- femoral D- obbirator

🌹Female 30s asking for cosmetic treatment of varicosity in her thigh,


asymptomatic, no other varicosity, how would u investigate?
A. Duplex B. CT venography C. No need for further investigation
/…
🌹lady who have Spider nevi she she wants to treat it cosmetically:( they mean before
treatment what should you do ) A) No need for investigation B) US doppler

🌹 ▪most common location for arterial ulcer >> Lateral. No pulse>> arterial
🌹 ▪Most common location for venous ulcer >> medial. pulse >>> Venous
*But the pulse will be the clue .*
🌷arterial ulcer= Lateral + No pulse🌷 🌷venous ulcer = medial + pulse 🌷
medial ‫ ﻋﺎدي ﻓﻲ اﻻﻣﺎن‬venus ‫= ﯾﻌﻧﻲ ﺟدا ﺣﺳﺎس ﻋﻠﻰ اﻟﺣﺎﻓﺔ ﯾﺎدوﺑك ﻟو ﺳﺎر ﻟﮫ اﺻﺎﺑﺔ راح ﯾﮭدد ﺣﯾﺎة اﻟﺷﺧص ﺑﯾﻧﻣﺎ‬latral ‫رﺑط= اﻟﺷرﯾﺎن ﻋﻠﻰ اﻟﺣﺎﻓﺔ‬

🌹 female present with bilateral lower limb swelling non putting not known to have any
medical illness : A-CT angio B-duplex ultrasound✅

🌹 Patient with unilateral left leg non-pitting edema, skin thickening and
hyperpigmentation (dermal fibrosis) for 2 months. Next best appropriate step?
○ CT angiography. ○ MRV. ○Duplex US ✅ (initial and the next)
NOTES: Non-Pitting edema is due to Lymphedema, Ultrasound is useful to exclude other
etiologies like DVT, venous insufficiency and can also help in identifying tissue changes and
masses that might be the cause of lymphatic compression.

🌹 Patient with unilateral left leg non-pitting edema, skin thickening and
hyperpigmentation (dermal fibrosis) for 2 months. Management?
Ÿ Lymphatic bypass
Ÿ Compression bandages (combination of physical therapies; two-stage approach) ✅
NOTES: Nonspecific treatment of lymphedema includes exercise, elevation, compressive
garments, manual lymphatic drainage, intermittent pneumatic compression, and surgery
(excisional procedures, microsurgery).
. . . .. . . .. . .
🌹 same question but swelling unilateral ask about management
- A- lymph by pass. - B- anticoagulant. - C-lymphatic massage ✅
‫ اوﻻ ﻣﺷﻛﻠﺔ اﻟﻠﯾﻣف ادﯾﻣﺎ ھو ﻓﻲ اﻻﻧﺗﻔﺎخ اﻟﺷدﯾد ﺑﺳﺑب ﺗﺟﻣﻊ اﻟﺳواﺋل و ﯾﺎﺛر ﻓﻲ ﻣظﮭر‬، ‫ اﻻن ﻧﯾﺟﻲ ﻟﻠﻌﻼج‬، Lymphedema ‫طﯾب ﺗﻛﻠﻣﻧﺎ ﻛﯾف ﻧﺷﺧص‬
‫ ف اول ك ﻋﻼج دواﺋﻲ ﻟﮭذا اﻟﻣرض ﻻااﯾوﺟد ﻣﺎﻓﻲ ﻋﻼج ﻧﮭﺎﺋﻲ ﻟﮫ وﻟﻛن ﻓﯾﮫ ﻋﻼﺟﺎت ﻣﺛل رﯾﺎﺿﺔ ﻣﻌﯾﻧﺔ ﻟﻠﻘدم‬، ‫اﻟرﺟل ﺗﻛون ﺟدا ﻣﻧﺗﻔﺧﺔ وﺛﺎﻧﯾﺎ ﺗﺳﺑب اﻟم‬
‫ ﻣﺎراح ﺗﺧﻠﯾﮫ ﯾﺧﺗﻔﻲ ﻟﻛن ﯾﻘﻠل ﻣن اﻻﻧﺗﻔﺎخ ﺑﺣﯾث ﯾوزع اﻟﺳواﺋل‬، ‫ او ﻧﻠﺑﺳﮭم ﺷراب ﺿﺎﻏط ﻛل ھدف ھذه اﻟﻌﻼﺟﺎت ﺗﻘﻠل اﻻﻧﺗﻔﺎخ‬masaage ‫ ﻧﺳوي ﻟﮭم‬،
....‫ھذه اﻟﻣﺗﺟﻣﻌﺔ وﯾﻘﻠل اﻻﻟم‬
There's no cure for lymphedema. Treatment focuses on reducing the swelling and controlling the pain. Lymphedema
treatments include:
1) _ Exercises. Light exercises in which you move your affected limb may encourage lymph fluid drainage
2)_ Compression garments. Long sleeves or stockings made to compress your arm or leg encourage the flow of the
lymph fluid out of your affected limb.
‫ اﻟﺮﺟﻞ‬û‫ﻞ ˜ﻜﻮن ﻣﺮة ﺿﻴﻖ ﻋ‬£í‫اب ﻃ‬êÛ ‫اب ﺿﺎﻏﻂ )ﻣﺜﻞ‬êÛ( ‫ ا•¾ ﻫﻮ‬.
3)_ Massage. A special massage technique called manual lymph drainage may encourage the flow of lymph fluid out
of your arm or leg.
. . .. . . . . . .. .
🌹Female on her fifties develop unilateral leg non-pitting edema with skin
discoloration (dark) and thickening of skin ,mx? -lymphathic bypass. -anticoagulant
-lymphatic massage and pressure dressing ✅✅ ‫اب ﺿﺎﻏﻂ‬Ņ̃ ‫ﻌﻄﻴﻬﻢ‬€ ‫ﺔ‬ÐÏ‫ﺔ اﻟﺪﻣ‬Ì‫ﺎن اﻟﺪﻛﺘﻮر اﻻوﻋ‬¥ ‫ﻣﻠﻴﻮن ﺻﺢ‬

🌹presentation lymphedema, what is the important thing in hx to ask about?


A. Medication B. Family hx C.Trauma D. Smoking
: ‫ ﻣﺜﻞ‬truma events ‫ ﻟﻠﻤﺮة اﻻوﻟﻰ ﺑﻌﺪ ﻣﺎ ﯾﺤﺼﻞ اي‬lymphedema ‫ﻣﻤﻜﻦ ﯾﺤﺼﻞ‬
such as bruises, cuts, sunburn, and sports
injuries
‫ﻓﻤﮭﻢ ﻓﻲ اﻟﮭﯿﺴﺘﻮري ﻧﺴﺎﻟﮭﻢ ھﻞ ﺣﺼﻞ ﺳﺎﺑﻘﺎ اي ﺗﺮوﻣﺎ ؟‬
.....
🌹Bilateral great saphenous vein mange ?
A. sclerotherapy B. something endoscopic C. ablation

🌹Lateral malleolus ulcer. Diabetic. Intact pulse. Best initial test?


A- Douplex affected leg✅ B- Douplex both legs C- CT angio

🌹progressive Non pitting edema, no DM or HTN. Best Investigation?


A- venous douplex B- CT C- MRA

🌹Male, diabetic, HTN, came with ulcer in lateral left malleolus for 3 months, pulse is
intact? A. MRA. B. Ct angio. C. Conventional angio. D. Venous duplex for left lower limb

🌹Pregnant lady in 15 WK gest comes with marked upper & lower limp oedema.
BP 150/95 What is your best management? A.complete evaluation ✅ B.atenolol
Note :Edema happens in normal pregnancy but upper & lower could be significant.
...... ....... .....
Lateral malleolus ulcer. Diabetic. Reduced pulse. Investigation?
A- Douplex affected leg B- Douplex both legs C- CT angio ✅
...... .. . . . . .. . . .. . .
🌹Diabetic patient present with leg redness increase with dependent position, on
examination, diminished pulse and cold skin What is the most likely diagnosis? Peripheral
arterial disease ✅🌹

🌹 Patient present with painful of cord like swelling of left leg and she has hx of vircous
vein 20 years back Best initial treatment? a) NSAID ✅. b) Celxan. c) Heparien!@

🌹Varicose veins Tx for therapeutic reason: - Endovascular laser ablation 🌹

🌹 Varicose veins Tx for cosmetic: = -Sclerotherapy 🌹

🌹Pt w/t varicose veins c/o swelling and heaviness, no pain, pulse intact .. everything
normal .., NEXT:A- Duplex US✅. B- MRV. C- CT. D- NO need for investigation
.. . . . .. .
🌷spider veinous case , what is the management? -Endoscopic laser ablation
‫ ﻓﻌﻼج ھذا‬، ‫ و دواﻟﻲ اﻟﻣريء ھذه ﺗﻌﻣل ﻧزﯾف‬esophageial varicosis ‫ ﺗﻌﻣل‬cirrosis ‫ وﻣن ﻣﺿﺎﻋﻔﺎت‬liver cirroais ‫ ﻋﻼﻣﺔ ﻋﻠﻰ‬spider vein ‫ﻻن‬
laser ablation. . . . ‫اﻟدواﻟﻲ ﯾﻛون ب‬
….
132-Female presented with vascular malformation in the leg (hemangioma
maybe) increasing. What is the indication of removal? A-Pain✅✅. B-Ischemia C-Cosmetic

🌹 What is hard sign of vascular surgery:-_Weak pulse _Skin color change. _bruit ✅
the presence of hard signs mandates immediate action. The presence of hard signs of
vascular injury mandates immediate operative intervention.
....... ........ ......
🌹 Hard sign to detect vascular injury
A. multiple fracture B. change color C. decrease pulse D_pulsetile bleeding

● Smoker elderly with preserved femoral and popliteal pulse put diminished
dorsalis pedis, Mx ? A. Localized thrombolysis B. Heparinization and observer

1Testicular
🌹Infant with Bilateral non tender scrotal edema and redness extending to groin. What's the diagnosis
A. Testicular torsion B. Epididymo-orchitis C. Edema of testicular appendages
D. Idiopathic testicular edema

🌹Patient came with testicular enlargement after an exercise what will you
do for him? US

🌹decrease in left testicular size post hernia repair ?


A. Testicular artery occlusion. B pampiniform plexus occlusion

🌹Pain with absent cremasteric reflex: A- Surgical consult ✅ B- US C- Culture


‫ وھذه ﺣﺎﻟﺔ طﺎرﺋﺔ ﻻزم‬testicolar torstion ‫ اﯾش اول ﺷﻲء ﯾﺟﻲ ﻓﻲ ﺑﺎﻟﻧﺎ؟؟‬: absent absent cremasteric ‫ﻣﺛل ﻣﺎﻗﻠﻧﺎ ﺳﺎﺑﻘﺎ ﻧﺷوف ﺑس ﻛﻠﻣﺗﯾن‬
.‫ﻓورا ﺟراﺣﺔ‬
🌹5 years old child found to have one testes in the scrotum and the other in
the ingunal area what to do? A-orchiopexy ✅ ✔. B-orchiectomy. C-wait tell puberty.

🌹Pt recent abdominal surgery presented by fever&otheres. what is the most cause
A-sepsis B- subphrenic abscess✅
(Other names. Subdiaphragmatic Abscess. Specialty. Infectious disease,
gastroenterology. Subphrenic abscess is a disease characterized by an accumulation of
infected fluid between the diaphragm, liver, and spleen.)
.....
🌹female patient complains of urinary dribbling, dyspareunia, dysuria. What is the most
likely diagnosis?
A. Overflow incontinence B. Urethral diverticulum✅ C. Stress incontinence
A urethral diverticulum occurs when an unwanted pocket or sac forms along the urethra.
......
🌹yearsold boy brought by his parents, he has nausea, severe
vomiting for 20 minutes and now semi comatose. The parents
mentioned that he has same episode two weeksago for 5 minutes
without deterioration in consciousness. On examination there is right
testicular mass that does not transilluminate with light. What is the best
action to do ?- Surgical exploration. ✅
🌹Testicular pain absent cremaster reflex (for 4hours) , clinical pic of (torsion) what
to do? Surgical exploration ✅

1Appendsitis\
🌹Female (obese) with typical appendicitis: Right iliac pain and tenderness
Nausea and vomiting. Loss of appetite. Leukocytosis What is the proper management:
A. Abd ct B. Abd us C. Expiatory laparo D. Open appendectomy

● Post appendectomy female came with LR abdomen mild tenderness Ex


Normal By CT there is 2*2 collection in Retrocecal :
A. Exploring laparotomy B. percutaneous drainage C. laparoscopic D. conservative

● Pathophysiology of appendicitis in 58 male ?


A. change in blood distribution B. peripheral vasoconstriction C. decrease heart index

🌹Patient obese came complaining of rt iliac fossa and tendrenss :


A- CT abdominal B- us abdominal Answer is: A, HE IS OBESE

🌹Young unilateral testicular swelling since 1 day, on exploration viable but cord
edematous :A-Torsion B- Incarcerated inguinal hernia C- Appendicular torsion

🌹Post appendectomy day-4 presented with abdominal pain and feculent discharge most
appreciate step? A. IV antibiotics✅ B. Ex lap
‫ ˜ﻘﻮﻟﻚ ﻋﻨﺪە‬pus ‫ﻪ ﻣﻀﺎد ﺣﻴﻮي‬‰‫ﺎ اﻫﻢ اﻫﻢ ﺧﻄﻮة وﻻزم ﻫﻮ ﻧﻌﻄ‬£±ä‫ﻜﺘ‬Ä ‫˜ﻌ „ •“ ﺗﺠﻤﻊ‬.

🌹Case scenario ....appendicitis but not ruptured best investigation


a. CT abd✅ b. US abd c. Exploratory
If child or female > US. If adult male > CT

🌹Peds 8 yrs old with RLQ pain and rebound tenderness what's confirmatory
test? A. US abdomen B. MRI abdomen C. CT abdomen

🍓Post appendectomy came with mild pain and collection 2x2 Asking about management:
A. Per cutaneous drainage B. Conservative with Abx✅
..

🍓Post-appendectomy, 0.5 cm carcinoid found on the tip of the appendix, mx?


Nothing = If carcinoid tumor less than 2cm > appendicectomy only
More than 2cm or at base of appendix> right hemicolectomy

🍓After appendectomy dr found a carcinoid mass on the tale of appendix Next? C.Tscan
abdomen and chest for staging

🌹Most surgical emergencies in pediatrics ? Appendicitis 🌹


🌹Old age came with symptoms if appendicular mass and treat it What you will do
A. Colposcopy 6 weeks ✅ B.12 weeks

🌹after initial conservative treatment of appendicular mass :


A.Open appendectomy after 12 weeks B.Laparoscopic appendectomy after 12 weeks ✅✅

🌹Appendicular abscess 15cm*17cm in a 33 y old man. How to manage


A. antibiotics ✅ B.Surgical (No drainage in the options)

🌹 During app. Surgery appendix not find !? . How to find it = Follow tenia colli

🌹pt febrile , abdominal pain , rebound tenderness at mcburney's point , wbc high , x ray
show right fossa ? A- appendectomy✅✅ B- gall bladder US C- urology consultation

🌹1 week post appendectomy pt came with right iliac tenderness not associated with nausea or
vomiting, infrequent loose stools . CT 2*2 collection behind cecum= A. percutaneous aspiration B. open.
C. laparoscopy D. consevative Mx✅ ‫ﻪ‬‰‫ﺾ ﻣﺴﺘﻘﺮة ﻣﺎﻓ‬£ª‫اﺷﻦ وﺧﺎﺻﺔ ﺣﺎﻟﺔ اﻟﻤ‬±ä‫ﻪ وﻻ اﺳﺒ‬‰‫ﻂ ﻣﺎ˜ﺤﺘﺎج ﻻ ﻋﻤﻠ‬‰‫ﺸﻜﻦ ¬ﺴ‬Æ‫ﻛﻮﻟ‬
‫ﺣﺮارة وﻻ اﻟﺘﻬﺎب ¬ﺲ اﻟﻢ‬

🌹After open appendix, seroma collection and leak from gap of sutures , no
inflammation, what to do? a-Dressing b-Evacuation c-Ab

🌹 Post appendectomy case 1 week , not complain ,but upon exam there was
seroma with in gaping wound .what u will do ?A. repeat and care of the dressing
B. drainage C. ct abdomen

🌹appendicular mass management


A- lap appendectomy after 12 weeks✅ B_open appendectomy after 12 weeks
The management of appendiceal mass is surrounded with controversy. Traditional management has been
conservative, with interval appendicectomy performed weeks after the mass had resolved.
..... ..... ....
🌹Perforated appendicitis post laparotomy, presented with fever, abdominal pain CT done
showed foreign material , reexploration done, gauze was found intraabdominal
،intraabdominal ، What to do?
A-Issue complaint against assistant B-Call your lawyer and report the incident
C-apologize to the patient and tell her what happened ✅ ✔
…..
🌹Young male with right lower abd pain and tenderness, low grade fever On abd xray:
radioopaque material (small at iliac crest site) What to do?
-urgent uro consultation-non contrast CT -appendectomy ✅
🌹Case of acute appendicitis, fecolith and abscess seen on CT management?
- open Appendectomy and drainage -lap appendectomy and drainage
-percutaneous drainage ✅
‫ ﺗﻜﻮﻧﺖ وھﻲ ﺗﺤﺼﻞ ﻧﺘﯿﺤﺔ اﻟﺘﮭﺎب اﻟﺰاﺋﺪة‬stool ‫ﯾﻘﻮﻟﻚ ﻛﺎن ﻓﯿﮫ ﻗﻄﻌﺔ‬
fecalith marked by the arrow which has resulted in acute appendicitis. Specialty. General surgery. A fecalith is a
stone made of feces.

((Drainage of Periappendiceal Abscess and Removal of Free Fecalith—‫))¿ﺬا ﻋﻨﻮان اﻟ×ﺤﺚ‬


.‫ وﺑﻌدھﺎ ﻧﻌﻣل اﻟﻌﻣﻠﯾﺔ إزاﻟﺔ اﻟزاﺋدة‬stool ‫ﯾﻔﺿل اول ﺷﻲء ﻧﺷﯾل ﻗطﻊ‬
🌹After laproscopic appendectomy . Pt come with retrocecal collections. :
A. surgical drainege ❌❌ B. percatenous drainage ✅✅

🌹Appendicitis case with abscess how would u manage it :


A.open B.laparoscopic C.antibiotics D. percuatanous drainage ✅
..... ..... . ......
8 days Post appendectomy with abdominal pain, distention, inability to
pass stools, In exam: feculent material discharge with signs of peritonitis
A- CT B- IV antibiotic C- Exploratory laparotomy

1prostat
A 73-year-old man presents pain in his right thigh. This has been getting progressively worse for the past 9
months despite being otherwise well. An x- ray is reported as follows: X-ray right femur Radiolucency of
subarticular region suggestive of osteolysis. Some areas of patchy sclerosis Bloods tests show: Calcium 2.38
mmol/l Phosphate 0.85 mmol/l Alkaline phosphatase 544 u/L Prostate specific antigen 4.4 ng/ml. What is
the most appropriate action?
A. Vitamin D supplementation B. Check serum testosterone C. Referral to an orthopaedic surgeon
D. Referral to a urologist E. IV bisphosphonates (could be prostatic Ca mets to the bones.)
…..
🌹75 y male c/o back pain, difficulty passing urine, psa: 84(high) , ALP: 410
(high) A. prostatitis B. prostatic cancer C. urinary bladder ca D. BPH
.N.B: High ALP indicates bone metastasis.

🌹 80 y old man with bilateral hydronephrosis on Us images


A. Prostate enlargement / Ca I'm not sure.
B. Bladder ca > come more aggressive pain, hematuria C. urethral stricture
IF THERE IS BPH, GO FOR IT. IF NO BPH, CHOOSE PROSTATIC CANCER.

-Part of urethra affected by surgical trauma in males


a. membranous b. penile D.Bulbar✅

🌹Part of urethra affected by surgical trauma in males ? bulbar if not in choices penile.

🌹Part of urethra affected by surgical trauma in males


A. membranous B. penile C. prostatic N.B: Membranous for pelvic fracture after MVA
🌹Old patient with loain pain . Us shows biliteral hydronephrosis =enlarged prostate🌹

🌹By Cyctoscopy: Redness in the dome of bladder What is the most likely diagnosis:
A-ischemic colitis. B-diverticular disease. C-Transitional bladder cancer✅
…..
🌹Old M c/o urinary sx, prostate median lobe hypertrophy, what is best for this pt?
Annual Prostate-specific antigen ✅
. ‫ ھﺬا اﻧﺘﯿﺠﯿﻦ ﯾﺮﺗﻔﻊ ﻟﻤﺎ ﯾﻜﻮن ﻓﯿﮫ ورم ﻓﻲ اﻟﺒﺮوﺳﺘﺎت ﻣﻦ اﺳﻤﮭﺎ‬high Prostate-specific: ‫🍬ﻟﻠﻌﻠﻢ‬
…..
🌹 Old pt came with difficulty urination and low back pain with high Prostate-specific
antigen dx?! A- Prostatic cancer✅✅. B- Benign prostatic hyperplasia. C- Prostatic
inflammation
Benign prostatic ‫ﺧذوا ھذا اﻟﺳر ) اي اﺣد ﻛﺑﯾر ﻓﻲ اﻟﻌﻣر وﺟﺎء ﯾﻘوﻟك ﻋﻧده اﻟم ﻓﻲ اﺳﻔل اﻟظﮭر وﻛﺎن ﻋﻧده ﺗﺎرﯾﺦ ﺳﺎﺑق ب‬
(metastasis ‫ وﺳﺎر ﻟﮫ‬prostata cance ‫ ﻧﺷك ﻋﻠﻰ طول اﻧﮫ ﻋﻧده‬difficult urination ‫ او ﻗﺎﻟك ﻣن اﻻول واﻧﺎ ﻋﻧدي‬hyperplasia
، ‫ ﻓﯾﺟﯾك ب اﻟم ﻓﻲ اﻟظﮭر‬spinal ‫ واﻛﺛر اﻛﺛر ﻣﻛﺎن ﯾروح ﻟﮫ ل ﻣرض ﺳرطﺎن اﻟﺑروﺳﺗﺎت ھو‬، ‫ ﯾﻌﻧﻲ ﺑدء اﻟﺳرطﺎن ﯾﻧﺗﺷر‬spinal ‫)ﻟل‬
:‫ ﻟو ﻓﯾﮫ ﻣرة ﺟﺎك ﻣرﯾض ﻛﺑﯾر ﺑﺎﻟﻌﻣر واﺷﺗﻛﻰ ﻟك ﻣن‬، ‫اﻋﯾد ﻟﻛم‬
🌹 difficulty urination and lower back pain plus high Prostate-specific + lower back pain = think about
prostatic cancer metastasis .
...... .....
🌹 Urethral trauma with bleeding post MVC. Next step? Suprapubic catheter✅✅
‫ﻤﺔ‬‰‫ ﺳﻠ‬/ ß
• ‫ا ﻣﺎ‬±£‫ﺔ ﻣﺎﻳﻨﻔﻊ ﻧﺪﺧﻞ اﻟﻘﺴﻄﺮة واﻟﻴﻮر‬Ä‫ﻣﺎﻳﻨﻔﻊ ﻧﺨﺘﺎر ﻓﻮ•¾ •ﺎث ﻻن ﻓﻴﻬﺎ اﺻﺎ‬

retrograde urethrogram is essential for diagnosis of urethral injury, or urethral stricture
-- ----- ----- -----
🌹 75yeard old male with ((( back pain ))) and urinary symptoms (obstructive) ,ALkaline
phosphatase hight and (((PSA 80))) what is the most likely Dx
-BPH. -prostatic cancer ✅✅
** PSA = Prostatic specific antigen = if above 4 = ‫وﺳﺘﺎﺗﺎ‬±1 ‫ ﻣﺎﻳﺮﺗﻔﻊ اﻻ „ •¨ اﻻورام اﻟ‬0„ ä ‫ﺠ‬‰"‫ﻃﺎن ﻻن ﻫﺬا اﻧ‬ê ‫ﻣﻌﻨﺎﻫﺎ‬
.......... ....... ...... ......
🍓65 yrs came with mild decrease in urination , us showed median lob hypertrophy of
prostate , PSA and digital rectal was normal , urine analysis and renal function normal
what to do? A. annual renal function tese ✅ B. Periodic investigation of PSA C. Beta
blockernD. Cystoscopy

1Hemorrhoids
🌹 sclerosing patient with history of rectal bleeding, anoscopy show swelling at 3,7 o'clock,
sclerosing therapy is planned what is the most appropriate to do sclerosing therapy!@
A. external hemorrhoids B. internal hemorrhoids C. prolapsed hemorrhoids D. thrombosed hemorrhoids

🌹 post hemorrhoidectomy pt has urinary retention. Cause: a. Trauma b. Anesthesia affect✅

🌹Patient after hemorrhoids operation develop Suprapupic pain with inability to pass urine what is
cause? Inadequate analgesic
...
🌹Patient 1 day post hemorrhoidectomy complaining of abdominal pain and distention
and he cant urinate, and there is anterior boggy aspect of anal digital examination . What
to do?A. Its Analgesia complication B. drainage C. CT

🌹 20s years old presented with pain during defecation menimum amount of blood, on
examination the doctor seen a posterior midline fissure. He couldn’t do PR exam as the
patient was in severe pain Vitally stable What’s your next step?
A. EUA (Examine under anasthesia EUA) ✅. B. LIS. C. LES D. injection of silicone
..
🌹 Old patient who has constipation on and off with streakin of blood in the stool with no fulness in the
rectum ( no mention of pain )? A. Sigmoid cancer B. rectal cancer !@ C. chronic hemorrhoid.

🌹 Old pt-fatigue sign of anaemia hb8 , stools with streak of blood -has hemorrhoids stage 2
? A. Sigmoid ca. B.rectal ca✅✅ c.chronic hemorrhoid
، Rectal cancer ‫ ﻻزم ﻻزم ﺗﺳﺗﺑﻌد وﺟود‬hemorrhoid ‫ اي ﻣرﯾض ﯾﻛون ﻛﺑﯾر ﺑﺎﻟﻌﻣر وﻋﻧده‬: ‫ﺳﻣﻌوا ھذه اﻟﻣﻌﻠوﻣﺔ‬
‫ ﻛوﯾس ﻻن‬rectum area ‫ ﻻاازم ﺑﺎﻟﻣرة ﺗﻔﺣص‬hemorrhoid ‫ﻟو اﻧت ﺟراح وﺟﺎك ﻣرﯾض ))ﻛﺑﯾر ﺑﺎﻟﻌﻣر (( وﻋﻧده‬
cancer ‫ﻛﺛﯾر ﺑﻧﻼﻗﻲ ﺻدﻓﺔ ﻋﻧدھم‬
🌹 Bleeding from the rectum is the most common sign or symptom that both rectal
cancer and hemorrhoids share.
symptoms include blood mixed with stool, a change in bowel habit , unexplained weight
loss in the absence of dieting, bowel obstruction, anemia and fatigue. = These symptoms
are not usually seen with hemorrhoids =it gose with Rectal cancer.
🌹 Hemorrhoids produce the symptom of pruritus (itching) in the rectal and/or anal area
while rectal cancers usually do not.

🌹 Patient came with painless blood after defecation, whats the diagnosis?
A. Abscess B. Fistula C. Hemorrhoids

🌹 Sclerotherapy use in management of ? A. Internal hemorrhoids ✅. B.External hemorrhoids

🌹Hemorrhoids type 4, what is the management: Hemorrhoidectomy


….
🌹case of hemorrhoid. what is the Indication of sclerotherapy? A- Internal✅. B- Externa

🌹Patient with perianal painful swelling -vitals normal, wbc normal Dx:
A-pile. B-perianal abscess C-perianal hematoma. D-anal fissure

🌹 Anal fissure with sentinel pile not responsive to medical therapy. Next step?
A)_Lateral external sphincterotomy. B)_ Lateral internal sphincterotomy(LIS) ✅
‫ ( = ر÷ﻂ‬Anal = internal ) ‫ ﺣﺮف‬n ‫ ﻣﻊ‬n .
Lateral internal sphincterotomy is an operation performed on the internal anal sphincter
muscle for the treatment of chronic anal fissure. The internal anal sphincter is one of two
muscles that comprise the anal sphincter which controls the passage of fece

🌹 streaks of blood after defecating and pain? Anal fissures ( lateral Sphincterotomy)✅
*s*treaks of blood = fi*ss*ures
...... ..... ...

🌹 pt with intermittent perianal pain and discharge, on P/E theres low-lying fistula
with an opening in posterior wall above anal verge. What to do?
A. MRI B. Fistulogram C. Fistulotomy D. Lateral sphincterotomy
....‫ وﻋﻼﺟﮭﺎ ﻧﺷﯾل اﻟﻔﯾﺳﺗوﻻ‬fistula ‫واﺿﺢ اﻧﮫ ﻋﻧده‬
🌹Perianal pain on examination no lesion, finger exam showed internal swelling
with purulent discharge on the anal canal , Dx: A. perianal hematoma B. internal abscess

🌹 Male adult with perianal swelling and discharge, then ruptured =


A. Anal fissure B. hemorrhoid C. anal abscess D. fistula
.....
🌹Pt with anal abscess then you found fistula on examination draining pus , the
opening is in the posterior rectum wall: management:
A. MRI B. fistulography C. fistulotomy/ectomy (one of them only)
....
🌹31 year old male with constipation and during perianal exam there was sever pain and
linear laceration at 6 and 12 oclock whats the dx:
A_ Anal fissure B. anal fistula. C. hemorrhoids
.....
🌹pt with bleeding after defecation ( painless ) A. Anal fissure B. Hemorrhoid
….
🍓pt presented with 3 months bleeding after stool, a perianal mass, and weight
loss. Histology confirmed adenocarcin oma. CT finding shows no lymph node
involvement and mass limited to 3cm from anal verge. What's treatment modality?
A. Radiotherapy B. Abdominoperineal resection✅ C. Chemotherapy D. Low resection only
Less than 6cm from anal verge > Abdominoperineal resection
More than 6cm > Lower anterior resection.!@
......
🌹 Case of perianal swelling , per rectal bleeding , investigated : it is 1 cm from anal verge
biopsy: adenocarcinoma (rectal) Mx:
A-Abdomen perineal resection ✅ ✔ B-low anterior resection C-chemo/radio
.....
🍓Adenocarcinoma of colon, 1cm from anal vege?
A. low anterior resection of rectum (LAR) B. Abdominoperineal resection!@
......
🍓patient with anal swelling and discharged, swelling is rapture, No fever:
A- fistula abscess. B- thrombosed pile

🍓Anal itching and pain post defecation and ask ddx?


A. internal hemorrhoid B. Anal fisure ✅!@
🍓Anal fissure failed medical Treatment? lateral internal anal sphincterotomy

🌹Anal fissures with skin tag not respond to drug, want procedure?
A- Internal sphernctomy✅✅ B- External sphrenctomy C- Anal curettage with remove tag

🌹Post partum woman with painful defecation and bleeding and pain after defecation;
what is the cause A. anal fissure ✅✅ B. Haemorrhoids C. thrombosed pile

🌹pt with Anal dischsge and he gave history of ruptured aanal abcess :
Anal fistual ✅. Anal' fissure Anal piles

🌹Patient have pain with and after defication and some blood with it diagnosis?
A-hemorrhoids. B-anal fistula. C-anal fissure ‫✅✅ اﻟﺗﻣزق ﯾﻌﻣل اﻟم‬. D-abscess

1wound
🍓Neck trauma stable anterior to the angle of the mandible A. CTA✅ B. exploration

🌹 aLots of questions on wound care/ exploration of wound/ abx — VERY


IMPROTANT TO KNOW WHEN TO DO EACH
🌹 First evaluate surgical wound by *inspection *✅
🌹 If there’s signs of infection* (pulurent discharge, redness, tenderness) then
wound exploration
if superficial > clean, dressing and you may give abx
🌹 If deep collection suspected >* order CT
🌹 If small collection less than 4cm >* abx
🌹 If large 4cm and more* > percuteanous drainage + abx
🌹 If signs of peritonitis > Laparotomy

🌹10 cm laceration in anterior thigh what to do?


a-Tourniquet b-Compress ✅. c-Vascular consult d-Band or tourniquet above femoral
artery
‫ﻒ‬£À‫ح ﻋﺸﺎن ﻳﻮﻗﻒ ﻧ‬ª‫ اﻟﺠ‬û‫ء ﺑﺘﺎﺧﺬون ﻗﻤﺎش وﻻ ﻣﻨﺪ˜ﻞ (ﺗﻀﻐﻄﻮن )ﻋ‬ÚÛ Û
• ‫ﺴﻮوﻧﻪ ؟؟ اول‬Ò ‫ء‬Ú
• ‫ﺶ اول‬2‫ ا‬±ä‫ وﻧﺰل دن ﻛﺜ‬±ä‫ح ﻛﺒ‬ª‫›ﻢ اي ﺟ‬Ä‫ﻠﻮا اﻟﻤﻮﻗﻒ ﻟﻮ اﺻﺎ‬‰‫ اﻧﺘﻮا ﺗﺨ‬.
... .... ....
🌹 Elderly pt w/t sacral ulcer .., management:
A- Depriment and skin graft(if necrosis) B- Primary closure C- Daily dressing
….
🌹 Elderly pt underwent for a major surgery ..he need blood transfusion 15 PRBCs .. after
that he start bleeding from wounds, nose, from NGT (stomach) .. what is the cause:
A- vWBD. B- thrombocytopenia✅. C- hemophilia. D- no DIC in choices
... ....
🌹 25 years old with perforated appendix did laparotomy, after 5 days wound is tender and
pus discharge what to do. a-wound dressing b-exploratory laparomtmy. c-iv abx✅
..
🌹 Girl with stab wound in the right anterior axillary line below the costal margin, stable
A. Observation. B. immediate surgical exploration. Perform FAST✅

🌹 Neck stap wound = if ask about Next step = wound explorontion (clinical exsmnation) =
if ask plan or managmant choice = C.T angio to neck ( stable pt) = if not stable Surgical
explorantion.
….
🌹trauma to neck asking about investigation to different zone:
Zone I and Zone III > C.Tangio
Zone II :- if asymptomatic > C.Tangio
if symptomatic > Surgical exploration

🌹 Pt with neck injury in zone I , vitaly stable , what’s next : A. neck exploration B. CT face and neck✅

🍓Neck injury->pt had Subcutaneous emphysema_> next step?


A. Ct neck✅. B. explore neck If zone I and III CTA Zone Il> surgical exploration

🌹18 yrs with stabbing wound 10cm of thigh what next step management:
A. compress on wound ✅B.tourniquet upper thigh
……
🌹 Neck trauma in area retro auricular above the angle of the mandible (area 1 was
not mentioned), patient stable and oriented. Most appropriate step?
A. CT angio. B. Neck exploration

🍓Patient has stab wound affect the duodenum and vital signs are stable what is your
management ? A. Ct scan✅ B. Laparotomy C. Conservative
.......
🍓pt with forearm fracture and open wound 1cm , what’s TTT :
A. close reduction. B. wire. C. cast D. debridement , irrigation , fixation✅
…..
🍓Pt post operative and leak 20ml fluid from the wound: !@
A. Dressing B. Wound exploration✅ C. Lap
......
🍓Post open appendectomy case with pain in wound site on examination u see
pus oozing from site of surgery what will u do next?
a. percutaneous b. US abd c. iv antibiotics d. open surgery
Antibiotic ‫ ﻧﺧﺗﺎر‬pus or ooze ‫اذا ﻗﺎﻟﻛم‬
open ‫ ﻧﺧﺗﺎر‬Fever ‫ واﻟﻣرﯾض ﻋﻧده‬absess ‫اذا ﻗﺎل ﻓﯾﮫ‬
🍓Pt has infected wound has pus come out from it what next step?
A-Inspection B-exploration of wound
....
🌹After herniotomy surgery for 5yrs boy came with fever and pus discharge and
part of mesh seen ? A.give iv antibiotics B.draining of pus
C. draining of pus and remove mesh ✅ D. observation

🍓X Ray showing both distal ulna and radius fracture with volar displacement On
examination a 1 cm wound was seen at the volar aspect of the wrist Asks about
initial management:
A. Closed reduction with above elbow cast B. Closed reduction with below elbow cast
C. Irrigation of the wound and Closed reduction with below elbow cast

🌹Young male with MVA came by ambulance to ER conscious, with thigh


wound covered with sucking gause : call surgeon in duty
......
🌹Trauma patient with a wound on his thigh subcutaneous fat is lost vasculature
underneath is exposed what provides the best management
A. Debridement with primary closure B. Primary repair
C. Debridemnt with secondary closure D. Debridement with vacuum assisted closure
....

....
🌹 25 year old male Pt 8th day post surgery with wound site redness & tenderness with
purulent discharge.. most appropriate?
A. IV antibiotics B. CT abdominal C.open drainage D.exploratory laparoscopy
.....
🌹Patient post surgery, there surgical wound redness, tenderness with no discharge,
abdomen is soft and lax .what to do? A. Antibiotic B. wound drainage C. CT abdomen

🌹Patient post surgery, there pus coming from surgical wound with tenderness and
leukocytosis, abdominal examination was done it was soft lax with no tenderness all over,
what to do? A. Antibiotics B. wound opening C. CT abdomen
open...‫ ﻧﺧﺗﺎر‬Fever ‫ واﻟﻣرﯾض ﻋﻧده‬absess ‫ اذا ﻗﺎل ﻓﯾﮫ‬Antibiotic ‫ ﻧﺧﺗﺎر‬pus or ooze ‫اذا ﻗﺎﻟﻛم‬

🌹Patient was hit with wood 5 days ago, now he comes with severe RLQ pain. On
examination there is a small opening with pus discharge, put when tried to extend his
thigh there was severe pain and you couldn't move it. What to do?
A. CT B. wound drainage C. antibiotics
...
🌹Patient post resection and colostomy presents with spiking fever for 1
week. Chest is clear. Abdomen is clear. Wound is clear. Fever 38.2 Next step?
A- CT abdomen B- Reassure
....
🌹person with car accident on left thigh show neurovessle fat and tissue out mange ?
debridment and skin grafting
🌹k/c HTN with lateral mules ulcer, next appropriate management?
A- wound examination ✅. B- duplex. C- Ankle brachial indexes

🌹Pt was stabbed by knife into his neck (ant to the right ear way through his mandibule )
Hé is stable with 1 or 2 cm wound ozing blood What is the next step in managment :
A-CXR. B-C.T angio. C-wound exploration ✅
‫ ﻣو‬forgen body ‫ ﻧﺷوف ھل ﻓﯾﮫ اي‬، ‫ ﻧﺳوف ﻋﻣﻘﮫ‬، ‫ اﻧﮫ ﻧﻔﺣص اﻟﺟرح‬stap wound ‫ اول ﺷﻲء ﻧﻌﻣل ل‬next step , ‫طﯾب ھو اﻟﺳؤوال ﯾﻘول‬
….‫ ﻛﻠﻣﺔ‬، . ‫ ف اﻧﺎ ﺑﺧﺗﺎر ﺳﻲ‬، ‫ﻋﻠﻰ طول ﺑدون ﻣﺎ اﻓﺣﺻﮫ اودﯾﮫ ﻟﻠﺳﻲ ﺗﻲ‬

.... .... ....


🌷Patient with perforated appendicitis after surgery had pus from wound, pain
localized to the surgical site. No guarding no fever what best initial treatment is:
A. Antibiotics✅✅ B. Open drainage. C. Imaging guided drainage D. Wound exploration
‫ﻣﺎزال ﻣﺳﺗﻣر وﻣﺎراح ﺑﺎﻟﻣﺿﺎد‬pus ‫ ﻟو ال‬culture ‫ﻧرﻛز ھو ﻗﺎل اول ﺷﻲء ﻧﺳوﯾﮫ واﻟﻣﺻﺎدر ﺗﻘول ﻧﻌطﯾﮫ ﻣﺿﺎد ﺣﯾوي ك اول ﺧطوة ﻣﻊ اﺧذ ﻣزرﻋﺔ‬
. . ‫ وﺧﺎاﺻﺔ ﻟو اﻟﻣرﯾض ﻗﺎﻟك ﻋﻧده ﺣرارة‬، ‫اﻟﺣﯾوي ﻻزم اﻓﺗﺢ اﻟﺟرح واﺷﯾل اﻟﻐرز‬

You may be started on antibiotics to treat the surgical wound infection. The length of time
you will need to take the antibiotics varies, but will typically be for at least 1 week. You may
be started on IV antibiotics and then changed to pills later. Take all of your antibiotics, even
if you feel better.

🌷- Patient post surgery presents with disch'rge from middle of the wound. Next step?
- Daily dressing. - Wound inspection✅✅ - Wound explor'tion
Û „
‫ء‬Ú
• ‫ اول ﺧﻄﻮة •¨ اي ﻓﺤﺺ ﻻاازم اول‬inspection
....... ....
Patient with right lower chest stab wound. Fast showed free abdominal fluid. What is your
next step? A. exploratory laparotomy ✅ B.thoracentesis C.chest tube
...
🌹Pt e deep wound 10 cm in front of thigh. What's your best next step =
A.tourniquet at level of femoral B.torniquet above wound level
C.apply good pressure at wound site ✅ D.call vascular surgery doctor
....... ...... .....
Patient with stab wound what to do next ? Local wound exploration✅

1radiology
🌹An elderly with IHD day 2 post cholecystectomy presented with sudden chest pain SOB and vitally
hypotension and tachycardia whats best ? A. CXR B. ECG C. CT angio D. LL duplex US (most likaly has PE)

🌹Elderly with weight loss (10kg) and anorexia in endoscopy there is large ulcer in stomach , biopsy was
taken and the result is invasive adenocarcinoma . What is the next step:
A. endoscopic US B. CT abdomen and chest and pelvis C. abdominal US

🌹Best diagnostic method for pulmonology embolism? A. D- dimer B. Spiral CT✅


... pulmonary angiography (CTPA): best definitive diagnostic test...

🌹Chronic thromboe-embolism confirme = A. QV ratio B. spiral CT C. D dimer

🌹 supraclavicular LN met what you do for primary site : Gastroduodenoscopy

🌹How to diagnose Adhesion post operative? CT, but Initially > abdominal X-ray (Air-fluid
level, Bowel dilatation)

🌹Old patient with worsening of his constipation, labs shows positive occult blood in stool.
Colon cancer suspected what to do?
A. Colonoscopy B. sigmoidoscopy C. CT abdomen

🌹 Pt melena and fresh blood per rectum, upper and lower endoscopy negative,
next investigation: Capsule endoscopic
‫ﺪاﻳﺘﻪ ﻟﻨﻬﺎﻳﺘﻪ‬Ä ‫ ﻇﻦ‬8 Û
• ‫ •ﻞ اﻟﺠﻬﺎز اﻟﻬﻀ‬û‫ ﻋ‬9 • ‫ﻤ‬£‫ و‬8
• ‫ة ﻧﺪﺧﻠﻬﺎ ﻟﻠﺠﻬﺎز اﻟﻬﻀ‬±ä‫ا ﺻﻐ‬±ä‫ﺎ¬ﺴﻮﻻ ﻫﺬە ﻓﻴﻬﺎ •ﺎﻣ‬7‫ اﻟ‬،
‫ﺴﺘﻌﻤﻠﻪ ﻟﻤﺎ ﻧﻜﻮن ﻋﻤﻠﻨﺎ‬v endscope ‫ﺴﻮﻟﺔ ﻫﺬە‬-ç‫ﻓﻨﻠﺘﺠﺎ ﻟﻠ‬. ‫ﻒ‬£±„ „ ‫ﺐ اﻟ‬-‫ وﻣﺎﻟﻘﻴﻨﺎ ﺳ‬.

🌹Old Man with falling trauma CT brain is normal but he is complaining of


bilateral foot pain: Check pulse > analgesia > X-ray

🌹best diagnostic for Coarctation of the aorta for neonate ? echocardiogram is the most
commonly used test to confirm the diagnosis ( Not ultrasound or C.t )

🌹 A man who is a known case of diabetes presented with hemiparesis 15 hours after
some procedure A. tpA B. Warfarin C. Ct angio
‫ ﻓﻧﻌﻣل ﺳﻲ ﺗﻲ اﻧﺟﯾوا‬small vessl brain ‫ ﻻن ﻗﺎل ﻋﻣل ﺑروﺳﯾﺟر ﻓﻣﻣﻛن ﺳﺎر ﻟﮫ ارﺗﻔﺎع ﺷدﯾد ﺑﺎﻟﺳﻛر واﺛر ﻋﻠﻰ‬strok ‫اﺣﺗﻣﺎل ﯾﻛون ﺟﺎه‬
blood vessel‫ ﻋﺷﺎن ﻧﺷوف‬angio ‫ اﺧﺗرﻧﺎ‬،
🌹Pt with bronchogenic carcinoma, presented with progressive SOB,
there’s elevated jvp, clear lung and quiet heart sounds. What will confirm your dx:
A. CXR✅ B. ECHO C. ECG
‫ ﯾﻌﻧﻲ ﺣﺻل ﻟﮫ ﻣﺗﻼزﻣﺔ اﺳﻣﮭﺎ‬elevated jvp ‫ وﻋﻧده‬bronchogenic carcinoma ‫اوﻻ اﻟﻣرﯾض ﻋﻧده‬
‫ طﯾب اھم‬، SVC ‫ ﻟل‬obstruction ‫ ﻻن اﻟﻛﺎﻧﺳر اﻟﻲ ﺑﺎﻟرﺋﺔ ﻋﻣل‬Superior vena cava syndrome
SVC.... ‫ﺑﻧﺷوف اﻟورم وھو ﺿﺎﻏط ﻋﻠﻰ‬. CHEST X.ray ‫ﺧطوة ﻟﺗﺷﺧﯾص اﻟﻣﺗﻼزﻣﺔ ھذه ھﻲ‬

🌹Pt presented with stabbed wound after wound exploration you found anterior
abdominal fascia penetration, (his vitals were stable) what's your next step?
A. CT abdomen B. MRI abdomen C. Exploratory laparotomy D. Diagnostic laparoscopy

🌹 Best way to clear show cervical spine:


A. Clinical judgement or assessment. B. Lateral cervical x-ray

🍓child episode of PR bleeding, fresh ; black stool (have both melena and hematochezia) ;
NGT showed greenish fluids ; colonoscopy was negative what is next to detect source of
bleeding ? A. upper endoscopy. B. Tc 99m✅. C. Barium. D. US
.... ......

🌹Chlid overweight with X-ray? slipped epiphyseal plate 🌹

🌹Old with back pain radiated to back , CXR air under diaghram = perforated peptic ulcer

🌹 Pt in ICU with coffee ground vomit? Stress gastritis

🌹 K/c of ulcerative on med presents with abd pain Invistigation show enlarged transver
colon 15 cm or mm? Management?
A.Total colectomy with ileostomy B.Proctocolsctomy with ileal pouch C- steroid

🌹 30 something morbidly obese male , how to decide best reduction surgery he will have
? A.Barium enema B. ct abdomen C. ultrasound abdomen ✅ D. GI endoscopy

🌹 farmer, wood brick in his abdomen 4days ago, 3 days ago he developed severe pain in
the same area, on exam there was green discharge coming out, what will you do?
A. take culture from the discharge. B. CT abdomen. C. Drainage

🌹Stap in abdomen right anterior axially line, below costal margin, patient stable
conscious what to do? a-Ct abdomen b-Us FAST

🌹cervical trauma in ICU patient after motor vehicle accident = C.T🌹

🌹 3 weeks post gastrectomy presented with progressive abdominal pain increasing


in intensity. What is the next appropriate investigation?
1_US 2_ X-ray ✅ 3_ CT 4_Laparotomy
‫ وﺑﺎﻻﻛس‬، suture ‫ ﻣن‬leak ‫ ﻣن اﻟﻣﺿﺎﻋﻔﺎت اﻟﻲ ﺗﺳﯾر ﺑﻌدھﺎ ھو ﺣﺻول‬، ‫ﻟﯾﮫ اﻛس راي ؟؟ ﻻن اﻟﻲ ﯾﻌل ﻋﻣﻠﯾﺔ ﺑﺎﻟﻣﻌدة ﺳواء ﻗص ﻣﻌدة او اي ﻋﻣﻠﯾﺔ‬
leak ‫ ﻟذﻟك اﻓﺿل طرﯾﻘﺔ ﻟﻠﻧﺎس اﻟﻲ ﺑﺗﻌﻣل ﺟراﺣﺔ ﺑﺎﻟﻣﻌدة ﻛل ﻓﺗرة ﻧﻌﻣل ﻟﮭم اﻛس راي ﻻن ﻟو ﺣﺻل‬، stable ‫ ﺑﯾن‬leak ‫راي راح ﯾوﺿﺢ ﺟدا اﻧﮫ ﻓﯾﮫ‬
.‫ﻓﻲ اﻟﺑداﯾﺔ ﻣﻣﻛن ﻣﺎ ﺗﻌﻣل اﻋراض‬
🌹 Patient complained of abdominal pain 3 weeks after sleeve gastrectomy, what is the
diagnostic test? 1-US 2-CT with contrast✅ 3- Plain abdominal X Ray( if ask next step )

🌹 Old male patient admitted as a case of large intestinal obstruction, underwent rigid
sigmoidoscopy which showed a mass in sigmoid region, biopsy was taken and resulted as
adenocarcinoma, what’s your next step?
A. Colonoscopy ✅ B. CT abdomen C. MRI pelvis D. Sigmoidectomy
............... ....
🌹 CT in truma important for = retroperitoneal injury✅

1spleen
Splenic laceration and thoracic aortic injury. You want to transfer the
patient. What is important?*
A. Transfer him to laparotomy B. Transfer him to thoracotomy C. Angiogram

🌹 Post splenectomy came complaining of left side pain, Reduce air entry in the left side,
Dx? A. Subphrenic abscess ✅✅ B.Post splenectomy overwhelming syndrome

🌹RTA patient, presents 2 days later with ascending aorta injury and splenich laceration,
the best immediate management is:
A.Take the patient for thoracotomy B. Take the patient for laprotomy ✅ C.Call vascular
surgeon

🌹 Patient after pacreatitis episode develops upper GI bleeding picture , scope was done,
gastric fundus bleeding was found ,sclerotherapy done Duplex ultrasound showed: splenic
vein thrombosis with patent portal vein whats is your management:
A-Splenctomy ✅ ✔. B-Distal splen renal shunt. c. Portocaval shun

🌹A 12 year old received a nonspecific blunt trauma on his abdomen and later presented
with generalized abdominal pain. Imaging of the spleen showed a 7mm hematoma and 4
cm tear(grade 3). Your management:
A. splenectomy B. Spleen preserving surgery C. Conservative
....
🌹Spleen injury 1= consertive treatment
.....
🌹Spleen injury 3=partial resect. ( preserving surgery )
....
🌹4 cm tear with hematoma 7cm grade 3
A. splenectomy B. splenic conservative surgery
......‫ ﺟﻣل‬3 ‫ ﻣﻊ‬3 . ‫ ﺣﺳب ﺟﺎﯾد ﻻﯾن‬3 ‫ﻻن ﺟراد‬spleen ‫ﯾﻌﻧﻲ ﻧﻌﻣل ﻟﮭم ﺟراﺣﺔ ﺑدون اﺳﺗﺋﺻﺎل ال‬
🍒Laceration spleen grade 4 ? Splenectomy 🍒

🍒You are in (( small hospital or something )) and you have a patient with aortic injury and
spleen laceration grade 3 or 4 and you want to transfer him , how do you call ?
A.ICU B-Vascular surgeon C. General surgeon ✅✅

🌹aortic injury and spleen laceration grade 3 or 4 and you want to transfer him , how do
you call = General surgeon ( not vascular )
...........
🍓30 years old male after RTA had a splenectomy, what will be low after hours from the
surgery?A. insulin✅ B. glucose C. vasopressin ‫ ﻻن ﻓﯾﮫ ﺟزء ﻣن اﻟﺑﻧﻛرﯾﺎس ﻻﺻق ﻓﻲ اﻟﺳﺑﻠﯾﯾن‬.
.......
🍓 Patient child with fever , abdominal pain and splenomegaly and the spleen is tender
what culture is important
A/Single blood culture B/Multiple blood cultures✅ C/bone marrow aspirate culture
most likely typhoid: Bone morrow as most Sensitive, but invasive so multiple blood
culture better .
…..

🌷If splenectomy > pneumococcal, HIB, meningococcal > given 2 weeks before the
surgery of splenectomy✅✅🌷
‫و اﻻﻧﻔﻠوﻧزا اﺣﺗﻣﺎل ﻛﺑﯾر ﺟدا ﺑﻌد ﻋﻣﻠﯾﺔ ازاﻟﺔ‬meningitis ‫ ﻟﻠﻧﯾﻣوﻧﯾﺎ و ال‬vaccin ‫ﻻن ازاﻟﺔ اﻟطﺣﺎل ﯾﻘﻠل اﻟﻣﻧﺎﻋﺔ ﺟدا ﺟدا ﻟو ﻣﺎ اﻋطﯾﻧﺎھم ھذا ال‬
‫ ﺳﺑﺣﺎن ﷲ‬spleen ‫ ﻋﺷﺎن ﺗﻌرﻓوا ﻧﻌﻣﺔ‬، ‫اﻟطﺣﺎل ﯾﺻﺎﺑوا ﺑﮭذه اﻟﻌدوى ﺑﺳﮭووووﻟﺔ‬

🌷 Patient 3 days post splenectomy develop fever 38.5 what is the cause of bacteremia?
A- UTI ✅ ✔ B- Peritoneal transfer N.B: fever day 3-5 post op is UTI.

-Pt postoperative day 3 has gram negative bacteremia how it rech to blood ?
A-translocation B-UTI C- gut

Alcoholic present with hematemesis. Imaging shows splenic vein thrombosis.


Management? ○Splenectomy✅

Splenectomy ‫ﻋﻠﻰ طﻮل اﻛﺘﺐ اﻟﻌﻼج‬ splenic vien thrombosi ‫📣اول ﻣﺎ ﻧﺸﻮف ﻛﻠﻤﺔ‬
. ‫ھﺬا اﻟﻌﻼج اﻟﻮﺣﯿﺪ ﻟﮭﺬه اﻟﺤﺎﻟﺔ‬.

1hernia
🌹 Woman in her fifties asymptomatic has a small femoral hernia, what is the best management:?
A. Observation B. Open with mesh C. Simple open D. Laparoscopic repair
*Femoral hernia*= We prefer to repair a femoral hernia *laparoscopically* because of its ease of
access. Old people > femoral we do open
….
🌹 Solder with bilateral inguinal hernia ?
A. Lap w mesh B. Open w mesh C. Open w/o mesh
…..
🌹30 year-old has a large indirect inguinal hernia sac that is reducible with mild effort. Bowel sounds are
heard in the hernia sac. This hernia is the result of which of the following?
A. A defect in the abdominal wall with protrusion of abdominal contents through
the internal inguinal ring, canal and external ring
B. A traumatic defect in the abdominal wall
C. A congenital defect in the abdominal wall allowing protrusion of abdominal
Contents through the inguinal canal
D. A defect in the abdominal wall caused by constipation
🌹Pt 56 c/ o sudden RLQ pain after lifting heavy object ; and mass in RLQ ; Cough negative ; abd muscle
tense even with clenching . He is on anticogulation due to A fib . What the most apporpieate mgx ? ^
here I am thinking of Rectus sheath hematomea ==A. angioempolizion and .. forget . B. rest and anaglesic

🌹Man developed rt abd swelling after lifting heavy object, painful irreducible , no
cough impulse ..dx? A. Ventral hernia B. hematoma

🌹hernia, how to manage:


A- laparoscopic repair with mesh ✅✅. B- open repair with mesh + same options without
mesh!

🌹pt did herinal repair with mesh 12y ago and came by obstruction sign and no sign hernia
repair ?A. Adhesion B. Late onset crhons
‫ ﺗﺤﺼﻞ ﻟﻬﻢ ﻫﻮ وﺟﻮد اﻟﺘﺼﺎﻗﺎت دﺗﺨﻞ‬±ä‫ﻄﻦ وﻣﺮ ﻋﻠﻴﻬﺎ (ﺳﻨﻮات )ﻣﻦ اﻫﻢ اﻟﻤﻀﺎﻋﻔﺎت ا•¾ ﻛﺼ‬Ç‫ﺔ ﺗﻜﻮن „ •¨ اﻟ‬‰‫اي ﻋﻤﻠ‬

‫ﺴﺪاد •¨ اﻻﻣﻌﺎء‬v‫ﺐ ا‬-‫ﺴ‬Ò ‫ﻄﻦ ﻫﺬە اﻻﻟﺘﺼﺎﻗﺎت‬Ç‫اﻟ‬
🌹What type of mesh used in ventral wall hernia? A. Subly B. Onlay C. Urblay

🌹 40 years old man underwent open hernia relain and 2 weeks later presented
with tenderness at the site of hernia repair and severe parasthesia/numbness/tingling
around his thigh that went down his leg. Your management?
A. remove mesh staples. B. Neurectomy and mesh removal. D.NSAID
....
🌹 soldier needs to undergo bilateral hernia repair. So what’s your management:
A. open with mesh B. Lap with mesh

🌹 Pt with Reducable hernia comes with intestinal obstruction manifestation and


on exam was redness and on Xray : multiple air fluid level =
A. incarcerated (very sever pain + nusia+ vomting + herniated tissue becomes trapped and
cannot easily be moved back into place
B.strangulated(come with shock)
C. obstructed(Bez say Reducable )✅
...
.....
🌹60 yo pt Post ventral hernia surgery with mesh 6yrs ago pt came with abdominal
distention and on radiology cut off sign and target sign and soft tissue mass
A. late onset crohns B. adhesions with mesh C. small bowel cancer (not large intestin).
...
🌹Elderly with signs of heart failure (lung crpeteaion, lower limb edema , raised jvp) has a
small ventral hernia booked for elective surgery :
A. do open hernial repair as booked B. do lap hernial repair as booked
C. wait until strangulation or obstruction D- delay
....
🌹 t for elective ventral hernia repair, HTN and DM, on examination high JVP, LL edema,
basal crepitation, your management:
A. open surgery B. lap with mesh C. delay operation until addressing current symptoms

🌹 65 year old patient came for elective surgery for ventral hernia , in examination he has
bilateral crickels and lower limb edema , what to do to him?
1- do laproscobic surgery as reschedule 2- do open surgery as reschedule
3- obtimate his symbtoms and surgery late✅✅
4- he is old and asymptomatic don't need surgery

🌹 Patient with femoral small hernia asymptomatic =Observe without surgery
....
🌹8 month infant with right inflamed red hemi scrotum, on examination there was red firm
irreducible painful scrotal swelling which is extended to to left inguinal region. Left testis
couldn't be palpated. What is the diagnosis? A. Torsion appendix testis. B. testicular
torsion c- epididymo orchitis. D. incarcerated inguinal hernia!@

🌹Inguinal intractable pain with hyperaesthesia with pain radiating to thigh 3 weeks post
inguinal hernia with mesh, upon physical exam, no recurrence or surgical site infection,
what's the mx:
A- Nerve block B- Systemic Anti-Inflammatory C- Neurectomy with mesh removal
D- mesh removal
....
🌹 after lap hernia repair with mesh, wound draining pus, your management:
A- explore wound + remove mesh B- put surgical drain C- something + daily dressing ✅

🌷patient has persistent pain after hernia repair, what to do?NSAIDS then nerve block ✅🌹

• 45-year-old female developed hernia located inferior and lateral to pubic tubercle. Type?
○ Femoral✅ ○Obturator. ○ Direct inguinal. Indirect inguinal
-- ----- ----- -----
🌷Patient came after 5days of hernial repair complain of inguinal mass , there is no
erythema , no tender and not associated with cough , what is most likely the cause :!@
A) Serosa✅ ‫ﺶ ﻣﻌﻨﺎﻫﺎ ﻣﺎﻋﺮﻓﺖ‬2‫ﻦ ا‬ç‫ ﻟ‬، ‫ اﺻﺤﻬﻢ‬/• ‫ﻧﺎﻫﺎ ﻻن‬±”‫اﺧ‬
B) Hematoma ‫ ﻣﺎ „ •¨ ﻋﻼﻣﺎت ﻟﻬﺎ‬C) wound infection ‫ﻣﺎ „ •¨ ﺣﺮارة او اﻟﻢ او اﺣﻤﺮار‬
D) recurrent hernia ‫ﺤﺔ‬ç‫ﻣﺎ˜ﻄﻠﻊ ﻣﻊ اﻟ‬
...... ....

-🌷Surgery for hernia w/t mesh, complains of neuropain radiates to thigh, he’s already on
analgesia and once he stop it pain come back, not improve next?
A- NSAIDs. B- Nerve block. C- Remove mesh. D- Remove stapes of mesh

Persistence pain after hernia repair & he’s already on analgesia and once he stop it pain
come back, not improve so we go to nerve block.
.... ......
🍒 Surgery for hernia w/t mesh .., after 3 weeks came to ER c/o pain and discharge x-ray
shows 3x3 cm collection of fluid .. high WBC .. NEXT?
A- Open and remove mesh B- I don’t think there was drainage ?!
if there’s collection we don’t remove the mesh. 1st if there’s collection we aspirate or if
open wound we do dressing, along with IV abx Removal of the mesh is the last step
either in abdominal hernia or inguinal
........
🍒 Bilateral inguinal hernia, how to manage:
A. lap repair with mesh✅ B. open repair with mesh C. lap repair without mesh
….
underwent hernia repair for right inguinal hernia, presented now complaining of that
ipsilateral testicular size is decreased after hernia repair. Dx?
A. Testicular artery occlusion. B. tight mesh C. pampiniform plexus compression✅✅
pampiniform plexus compression= lump in one of your testicles+ swelling in scrotum.
…..
varicocele is an enlargement of the veins that transport oxygen-depleted blood away from
the testicle.is believed to develop as a result of dilatation and tortuosity of veins of
the pampiniform plexus
..... .......
🌹Lower limb paresthesia after hernia repair with mesh ?
A. nerve block ✅. B. remove mesh C. neurectomy ........
.....
🌹Elderly pt e hx ( .. months) of open hernial repair operation with mesh , complaining of
groin swelling reach scrotum +ve cough impulse how to manage ? A. Lap with mesh ✅.
........
🌹 Pedia no family with inguinal hernia, pedia surgeon advise treatment
A.Refer to police B.Refer to social worker C. Ask ethical comity ✅🌹D.Refuse
........
🌹60 yrs old female with femoral hernia, asymptomatic, accidental finding mx?
A.observation B.open C. open with mesh . D.laparoscopic !@
.. .
🌹Old man came to you with Inguinal reducible hernia, and he was advised to do hernia
repair, and PE exam was normal he is asymptomatic what will you do:no surgical treatment
...
🌹Case of hiatus herni' and ask about immediate management: Lifestyle modific'tion ✅✅
….
🌹Making a few lifestyle changes may help control the symptoms and signs caused by
a hiatal hernia. Try to: Eat several smaller meals throughout the day rather than a few large
meals. Avoid foods that trigger heartburn, such as fatty or fried foods, tomato sauce,
alcohol, chocolate, mint, garlic, onion, and caffeine

🌹Pt do hernia repair "not sure about the type" then he present with mass in the inguinal
hernia firm, regular, transpulsation "there was no Erythema or tender"and no fever :
A. saphenous B. psudoanyresm✅✅ C. Abscess
…….
#Pseudoaneurysm = A complication of laparoscopic inguinal hernia repair
…..
🌹year-old weight lifter developed abdominal hernia that is irreducible, tender. Imaging
shows air-fluid levels in small bowel and no free air in large bowel. Type?
A- Irreducible. B- Incarcerated✅✅. C- Strangulated
🌹incarcerated hernia is a part of the intestine becomes trapped in the sac of a hernia—
the bulge of soft tissue that pushes through a weak spot in the abdominal wall. If part of
the intestine is trapped, stool may not be able to pass through the intestine.

🌹strangulated hernia occurs when the blood supply to the herniated tissue has been cut
off. This strangulated tissue can release toxins and infection into the bloodstream, which
could lead to sepsis or death. Strangulated hernias are medical emergencies.
An incarcerated hernia occurs when herniated tissue becomes trapped and cannot easily be
moved back into place. An incarcerated hernia can lead to a bowel obstruction or
strangulation. In other words, a strangulated hernia cannot cause an incarcerated hernia.
Incarcerated hernias are not life-threatening
‫ واﻟﻲ ﯾﻌﻣل اﻧﺳداد ھو‬obstruction ‫ ﻣﻌﻧﺎھﺎ ﺣﺻل اﻧﺳداد ﻓﻲ اﻻﻣﻌﺎء‬air-fluid levels in small bowel ‫ﻗﺎﻟك‬
‫ ﺑﯾﻧﻣﺎ ﻟو ﻗﺎﻟك‬، ‫ﯾﻧﺣﺑس وﻣﺎﯾﻘدر ﯾﺗﺣرك ﻓﯾﺣﺻل اﻧﺳدداد‬stool ‫ﻻن ﯾﻘوﻟك اﻻﻣﻌﺎء ﺗﻛون ﻣﺣﺻورة ﻓﺎل‬Incarcerated
‫ ھذه اﻟﻛﻠﻣﺔ ﻣﻌﻧﺎھﺎ ﻛﺎﻧﮫ ﺧﻧﻘت‬، strangulation ‫ وﻋﻧده ﺣرارة ﻋﺎﻟﯾﺔ ھﻧﺎ ﻧﻔﻛر ﻓﻲ‬sepsis shock ‫اﻟﻣرﯾض دﺧل ﻓﻲ‬
.‫ وﻏﺎﻟﺑﺎ ﻣﺎﯾﺟﯾك ﺑﺎﻻﻋراض اﻟﻲ ﻓﻲ اﻟﺳﯾﻧﺎرﯾوا‬، ‫ ﻣﻧﻌت اﻟدم ﺗﻣﺎﻣﺎ ﻣﻧﮫ‬، ‫اﻟﻣﻛﺎن وﻣﺎﺧﻠﯾت ﯾوﺻﻠﮫ دم اﺑدا‬
‫ ان‬. ‫ ﻓﺧذوا اﻟﺣل اﻟﺻﺣﯾﺢ اﻟﻲ ﻛﺗﺑﺗﮫ اﻻن‬، ‫ اﻟﺳؤوال ھذا ﻣﻣﻛن اﻛون وﺿﻌﺗﮫ ﺳﺎﺑﻘﺎ وﻛﺎن اﻟﺣل ﯾﺧﺗﻠف ﻋن ھذا‬: ‫ﺗﻧﺑﯾﮫ‬
‫ﺷﺎء ﷲ ﺗﻛوﻧوا اﻗﺗﻧﻌﺗوا ﺑﺎﻻﺟﺎﺑﺔ‬
. . . .. .
4 y/o came to clinic w/t umbilical hernia without any Sx .., next:
A- Reassure ✅✅✅. B- Surgery C- Band

18 year old healthy male was playing baseball and suddenly he felt abdominal pain. On
examination he has para-umbilical mass. His vital signs Bp 100/76 RR 30 HR 100 O2 sat.
95% oxygen mask. What is your next step in management?
A. Abdominal US. B. CT scan. C. Erect CXR. D. Reassurance and send home
** Seems umbilical hernia incarceration due to sudden pain and hypotension

1tumer
🌹Sclerotic lesion in distal femur: A. osteosarcoma B. chondrosarcoma C. ewing’s sacroma

🌹old with weight loss epigastric pain with supraclav lymph node : A. gastric cancer
🌹Patient w incidental finding of macroadenoma of pituitary during evaluation of
her headaches, physical and medical hx was unremarkable, what’s the best
next step? A. - ant pituitary hormone scan B. - Referral to neurosurgery
🌹Mid-thigh lump with normal overlying skin and +ve fluctuating test. diagnosis?
A. Lipoma B. Sarcoma C. Aneurysm D. Sebaceous cyst
...
🌹Patient came the weight loss and abdominal distention, Ct found: Soft tissue mass retro-
peritonal , and multiply hypo-dense or hyper (forget ) liver lesions :
A- liposarcome B- germ cell tumor C- lymphosarcoma

🌹what makes lymphadenopathy malignant more than infectious:


A. less than 5 y B. duration less than 4 weeks C. cervical lymphadenopathy
D. supraclavicular lymphadenopathy

🌹Old man has progressive dysphagia e mass in mid esophagus ,chest x-ray normal . What
is the most likely cause A. adenocarcinoman B. SSC C. Lymphoma
SCC > middle and upper Adeno > lower

🌹Man with high grade dysplasia of the esophagus. How will you manage?
Refer him for surgery

🌹Smoker with chronic dyspepsia, endoscopy biopsy is squamous cell with


low grade dysplasia, what is your management?
A. PPI B. Refer for distal Esophageal resection C. Observe

🌹 Most common/important risk factor for esophageal cancer is:


B. Heavy smoking D. Barrett's esophagus

🌹Low grade dysplasia barrett's esophagus, what is the management:


A. Pantoprazole B. ranitidine
....

🌹what is the most common small bowel cancer ? Adenocarcinoma

🌹Epigastric pain diarrhea and peptic ulcer with + secretin ? A. Gastrinoma B. Carcinoid

🌹klatskin tumor= is cholangiocarcinoma located at bifurcation of common hepatic duct =


CA 19-9.

🍓Pt known case of lung cancer and had history of epilepsy and on medication,
medication is contraindicantion to this pt ?
A _ nicotine replacement medication B_ Bupropion = epilepsy
🍓colon cancer surgery after that he had 7ml/h urine output for 8 hours , blood pressure
and heart rate was normal what is the best management :
A. 500 NS challenge✅✅. B. diuretic. C. Inotropes

🍓23 ylo women Typical IBS scenario her uncle in 60s has colon cancer, all blood
labs normal except mild low Hb 11 (12-15)!!!! Ask about Dx?
A. UC. B. Colon cancer✅. C. IBD

🌹Adult patient came to a clinic with a small mass on her forearm since childhood. Asking
about the management: A.Laser B.Excision C.follow up ✅ D.radiotherapy

🌹 Pt hypertensive have low abdominal pain they do adrenal hormone workup (normal)
and ct abdomen show hypoechoic and lipo.. A.Nothing to do B. Biopsy ✅.
C.Adrenalectomy
Indications for adrenalectomy may seem straightforward at first. However, with the
increasing rates of diagnosis of asymptomatic and likely benign adrenal masses,
disagreements regarding optimal management remain. [11]
Masses larger than 6 cm have a rate of adrenocortical carcinoma of 25% and should be
managed surgically. [9] Masses smaller than 4 cm may be safely observed because their
rate of adrenocortical carcinoma is 2%. [9]
🌹ct abdomen show hypoechoic on adrenal gland = biopsy ‫ﺚ او ﻻ‬Ì‫ﺪ ¿ﻞ ¿ﻮ ﺧﺒ‬Û‫ ﻧﺘﺎ‬ÜÉ‫ﺣ‬

🌹if mass more than 6 cm do Adrenalectomy if less than 4 cm , usully observantion . 🌹

🍒adrenal tumor with high cortisol symptoms next management :


A. Surgical resectio ✅🍒 B. Chemotherapy C. Radiation

🌹 Old man came with vague abdominal pain with 20cm×20cm mass , finding in u/s
multiple hypoechoic masses ( or nodule ? ) What is the diagnosis :
A) libosarcoma ✅ B) lymphosarcoma C&D) can't remember them
🌹abdominal pain with 20cm×20cm mass , finding in u/s multiple hypoechoic masses
== libosarcoma

🌹Sarcoma metastasis to ? lung

🌹Sarcoma Biopsy most Diagnostic test: A. Incisional biopsy B. Excisional C. Needle core
‫ =ﺗﻘﻄﻴﻊ‬0 „ ä ‫ = ﺳﺎرﻛﻮﻣﺎ =ﺳﻜ‬incistion
🌹Sarcoma "no incisional in choices "? Core needle biopsy

● Most important about retroperitoneal sarcoma :


A. compress over the organs . B. Met to the near organs
🌹 Sarcom' of the thigh. Whats the best imaging for staging work up?
- CT ✅ sarcoma = c.t = s=c

🌹pt with mass 2 cm away from anal verge on proctoscopy it appears cauliflower:
A. Anal cancer B.Chondalymia acuminta ✅

🌹70 year old male Weight loss fatigue proctoscopy shows mass 2 cm from anal verge
cauliflower like friable mass ? A. anal ca B. rectal ca C. colon ca D. condylomata acu

🌹Cancer in body of stomach with no metastasis and no lymph node =


A.gastrectomy B. wide local local excision with clear margins ✅

1Chest tube
🌹by x.ray there is pneumothorax 2 cm : A observanton B- chest tube

🌹Pt with trauma has mild respiratory symptoms, no deviation of trachea, ...etc (all mild
symptoms) , What is the dx? A- tension pneumothorax B- simple pneumothorax

🌹 25 years male involved in RTA in ER he is fully continues and no abnormality in vital signs
admitted for observation and a FAST scan done ,.what do you expect to find
A. Pneumothorax B. major vessel bleeding in the thorax C. peritoneum free fluid

🌹Pt came to ER after MVA with multiple trauma and hypotension, tachycardia, x ray shows
cardiomegaly and sternal fracture , Dx ? hemopericardium ✅

🍓Thoracocentesis , where to insert ( which rib) ? 8-9


Thoracocentesis: between 9 & 10 ribs midaxillary line. Needle thoracotomy: 2nd rib space
....

🌹Elderly smoker with progressive SOB, on CXR massive Rt pleural effusion what's your next
step? A. Thoracocentesis✅ B. Intercostal tube under water seal
Thoracocentesis .‫ ﻧطﻠﻊ اﻟﺳواﺋل ب‬plural effustion ‫ﻋﻧده‬

🌹 lateral pleural effusion ask about site of needle insertion ? 8-10🌹Latral = 10

🌹Case about pleural effusion on chest tube after a while there was blood in the water
under seal (Hemorrhagic Pleural Effusions and Hemothorax), how to manage?
A-Thoracentesis B-Tube thoracostomy (chest tube). ✅ ✔ C-thoracotomy

Hemorrhagic pleural effusion ttt usually by chest tube, if the drain produced > 1.5 L on
insertion *or* > 1 L within the first hour then we shift to thoracotomy..
....... ....... ....
🌹Case of fall truama Increase jvp Decrease air entry (Low bp) (tackycardic) No more infos!
> low bp mostly bleeding so hemothorax Tx? A. Thoracostomy ✅ B.Thoracotomy
C.Unrelated
……
Patient had fall 50 meter, absent lung sound on the right side, CXR was
provided but not clear: A. Intubate B. Thoracostomy
🌹pt fall from hight and Vitaly insatable and Pic of Cxr with symptoms and signs of
hemothorx and ask 'bout ttt : Chest tube ✅✅. Thor'cotomy. Abx Anelgesics
chest tube‫ ﻋﻠﻰ طول ﻋﻼﺟﮭﺎ‬hemothorx ‫ﻣﻌروف اول ﻣﺎﻧﺷوف‬
....... ....... ....
🌹A man presented with a gun shot wound in his chest that he received an hour ago, and
you’ve done needle thoracentesis. 15 mins later 100 ml of blood is drained, what is the next
most appropriate action? A. needle thoracentesis B. Thoracotomy
…..
🌹 Patient who had had multiple traumas in MVA, they mention presence of free
fluid in the abdomen and spleen laceration + thoracic aortic rupture. Next
A. Thoracotomy B. Laparotom

🍓RTA , presented with slow breath and decrease breath sound left lower lobe lung xray
picture showed infiltrate in lower left lobe and he has lower limb fracture His blood
pressure and HR was normal ,What is your management
A. thoracostomy tube✅ B. thoracotomy C. angiography with stent

🌹young adult was playing football with his friend, the ball strongly hit his chest, after
sometimes he had SOB, what is the diagnosis?
A-Pneumothorax B-Pulmonary Contusion ✅ ✔ C-Tension Pneumothorax
. .. . . .. . . .
🍓RTA pt in hospital with limited facilities Found to have tension pneumothorax and
fracture femur You insert chest tube And immobilize the fracture Pt stable Decided
to transfer him To tertiary hospital , On the way to ambulance He suddenly collapsed, What
is appropriate next step? A. Intubate. B. Check chest tube insertion and functioning!@

🌹Patient involved in RTA was transmitted to near by limited facilities hospital and the
physician decided to refer him to advanced facilities hospital Patient is unconscious:What
is the most important thing he should be maintain:-intubation ✅

🌹left hemithorax : A. Chest tube of left hemithorax ✅ B. Needle decompression

🌹A known Copd patient presented with sob but was not in distress vitals stable
examination was normal.. no tracheal deviation and equal air entry on both sides..cxr
shows a 2cm pneumothorax.. wat will u do
A. Oxygen and observation (bez 2cm) B. Chest tube C. Pleurodesis D. Needle
decompression

🌹Young patient comes with dyspnea and chest pain after a long flight. Upon examination:
patient is tall, thin. CXR finding: pneumothorax.management:
A. Thoracocentesis (with effustion) B. Thoracostomy tube C. Conservative management❌
...
🌹Patient with pneumothorax tube thoracostomy inserted after 15 min water seal bottle
is filled with blood PB: 90/60 HR: 100 Rr:22 =
A- thoracotomy. B- thoracostomy tube ( it's already in the q I don't know why it's in the choices ) C- ct
abdomen

*Patinet who was hit in the chest while playing football, after it SOB, tracheal shift ,
hypotension and raised JVP , diagnosis : -Tension pneumothorax ✅

1intupation
Child with CXR finding hyperlucency of upper left zone with compression of adjacent
lobe with shift to the right, what to do?
A. thoracotomy B. Thoracostomy C. Intubation

Pt with small stab wound in anterior aspect of neck. On exam he is Alert
and conscious but in labs oxygen sat 82%. What to do
A. Oxygen mask B. Cricothyroidotomy C. Endotracheal intubation

🌹Proper ET position how to chick :
A. co2 monitor (my choice) B. chest movement. C. easy insertion.

🌹RTA , facial fractures , unconscious, what is opportunity in mx?
A. Cervical spine. B. airway

🌹Multi trauma pt, with many facial fractures, next step?
A. secure airway B. C-collar

🍓man with facial bones fractures after RTA, in the ER what is the first thing to do?
A. Airway✅ B. Call neurosurgeon

🍓 man after accident and resuscitation in small hospital u need to transfer to another
hospital after stabilization, it is 30 mins far. on xray u see fracture of 2-5 left ribs. no
pneumothorax what will u do?
a. intubate✅ b. call the other hospital to inform the surgeon on call
c. chest tube insertion

🍓A scenario of a woman trapped in house that was burning and lost consciousness, Some
labs were given which I can't recall. How do you manage? intubation🍓

🍓Patient have a car crash , he had an injury in the neck , the is tissue visible , how to
manage him ? A. Intubation. B. Cricothyroectomy. C. O2 mask✅

🌹 Pt came with trauma and from the explanation he have “flail chest” what to do?
A. iv fluid resuscitation ✅ B. Intubation

🌹 Case of flail chest ask about initial thing to do? he was stable
A. IV fluid B. Assistant ventilation
....

🌹40 something old pt came with anterior thigh stab wound 1 cm with active bleeding and
unconscious, most appropriate Mx (I’m not sure if it was written most next appropriate mx or not
but i think it was not written)
A. apply tourniquet B. Blood transfusion/ IV fluid C. intubation ✅✅

🌹patient with trauma to face with fracture mandible resuscitation with?
A.laryngeal mask B. cricothyroidotomy ✅✅ C.Oral endotracheal tube
..
🌹Mva with mandible fracture = Cricothyrotomy

🌹After an accident 18 yrs fracture of mandible and maxilla oropharyngeal tube /
tracheostomyhow we improve breathing? cricothyroidotomy

🌹Absolute contraindications for NG intubation include the following:
1_Severe midface trauma. 2_ Recent nasal surgery.

🌹Chemical product burn ttt ulcer in back = A. wash with water for 30 mints ✅
B.intubation C.dressing

3_Burn on the face , GCS 15 he was alert and speaking. the two striking choosies were=
a. Elective intubation✅ b. ICU admission for 24 hrs
‫ﺔ‬£‫ﺎر‬‰‫ﺎت اﻟﻮﺟﻪ ﺗﻜﻮن اﺧﺘ‬‰‫ر÷ﻂ =اﻏﻠﺐ ﻋﻤﻠ‬

Face burns expose patients to a higher respiratory risk, and early prophylactic intubation
before they enter the burn unit might be life-saving✅

Pt had chest trauma nad 3⁄4 rib fracture paradoxical chest


movement? ‫ﻪ ﻛ× ﻋﻈﻢ اﻟﺼﺪر ﻧﺺ ﻣﻜﺴﻮر وﻧﺺ ﻻ‬‰‫˜ﻌ „ •“ ﻓ‬
•analgesia
•intubation and ventilation are usually required or high concentration oxygen by mask...
. ...... ..... ......
🍒 Patient after RTA was conscious GCS 15 then suddenly collapsed u see a temporal
fracture what caused his loss of consciousness
a. subdural hematoma b. epidural hematoma✅ c. fracture base of the skull

🌹Patient after RTA was conscious GCS 15 then suddenly collapsed u see a temporal
fracture what caused his loss of consciousness?
A. subdural hematoma B. epidural hematoma C. fracture base of the skull

🍒2nd degrea burn of anterior lower limbs..wt is70 kg ..fluid replacement


A-Normal saline 200ml/hr
B-Ringer lactate 1 boluse then 120ml/hr
C-Normal saline 5 L in the 1st hr then
D-Ringer lactate 2.5 L through the 1st 6hrs then 2.5 L in the next 6 hrs

🌹Young male with bilateral anterior without post lower limbs 2nd degree burn who
weighs 70 kgs Asks about fluid replacement according to parkland formula:
A. 2.5L to be given in the first 8 hrs and 2.5 L in 16 hrs
B. 5L in first 6 and in 16 hrs
‫ﺴﺘﻌﻤﻞ‬v parkland form=
4ml x TBSA (%) x body weight (kg)
‫ال‬lower limp 4 ‫ﺖ ا•¾ ﻫﻮ‬Ä‫ﺎﻟﺮﻗﻢ اﻟﺜﺎ‬Ä ‫÷ ـﻬﺎ‬ö„‫ ﻧ‬70 ‫ﺎﻟﻮزن ا•¾ ﻫﻮ‬Ä ‫÷ ـﻬﺎ‬ö„‫ ﻧ‬18= ‫ﻌﺾ‬Ä ‫ ﻣﻊ‬0 „ ä ‫ اﻻﺛﻨ‬٩= ‫اﻟﻮﺣﺪة ﻓﻴﻬﻢ‬
„ „
2500 ‫ ﺳﺎﻋﺔ ﻓ"ﺴﺎوي‬١٦ ¨• ð• „ ‫ ﺳﺎﻋﺎت واﻟﺠﺰء اﻟﺜﺎ‬٨ ‫ اول ﺟﺰء •¨ اول‬ï „
• ‫ ﻋﺸﺎن ﻧﻌ‬0ä ‫ اﺛﻨ‬û‫ „ﻧﻘﺴﻤﻬﺎ ﻋ‬5040 ‫ﺴﺎوي‬Ò
‫ ﺳﺎﻋﺎت‬٨ ‫ •¨ اول‬±”‫ ﻟ‬0 „ ä ‫˜ﻌ„“ اﺛﻨ‬

🌹pt with 20% back burn which indicate good response== Urine output 0.9/kg/hr

🌹Burn case black soot over nostrils and mouth 40% carboxyhemoglobin.
(Carbon monoxide toxicity):
A. Hyperbaric oxygen. ‫ﻄﻠﻊ‬€ ÜÉ‫• ﺣ‬x‫ﻴ‬ß‫ ﻛﺜ‬w
y x ‫ﻪ اوﻛﺴﺠ‬Ì‫ ﻧﻌﻄ‬co
B. Intubation and ventilation with 100% o2. C. Carbonic anhydrase inhibitors.

1skin
● Dibetic has progressive painful lump on back of neck with multiple openings :
A. cellulitis B. Abscess C. Furuncle D. carbuncle ‫ﻣرض اﻟﺳﻛر = ﻛرب‬
🌹diabetic patients with unhealed ulcer for 5 years and biopsy showed
pseudoepitheliomatous hyperplasia.
A. Biopsy B. Debridement C. -ve pressure dressing

🌹Patient with liver mass Ct first stage filling periphery late washout centrally :
A-Hepatoma B-hemangioma C-metastatic

🍒-25yr old girl with skin lesions since childhood= -Laser -Excision -Observer✅
🍒Open fracture came 4 days later to ER with signs of infection ( blue, necrosis) what's the
most common organism ?
clos‫ ھو‬necrosis ‫اﻟﺑﻛﺗﯾرﯾﺎ اﻟﻲ ﺗﯾﺟﻲ ﺑﺳﺑب اﻟﺟرح اﻟﻣﻔﺗوح وﻋدم اھﺗﻣﺎم ﺑﻧظﺎﻓﺗﮫ وﻓﯾﮫ ﻋﻼﻣﺎات‬
A-clostridium✅🍒 B- Staph aureus C- Actinomyces israelii

🌹 case about gas gangrene due to open leg fracture infected by? Clostridium perfringens

🌹Case of melanoma = Excsion with safety margins

🍓Patient with limp sarcoma confirm the diagnosis :


A. incision biopsy✅. B. core biopsy. C. fNA biopsy
Sarcoma > incisional biobsy
Melanoma > excisional biobsy

🌹Skin lesion irregular and dark over the knee


A.bunch biopsy B.Chemo C.Radiation D. excisional biopsy ✅

🌹 Old patient with 3*4 dark elevated skin lesion at the ant of his thigh, what’s the
next appropriate step? A.excision B.Punch biopsy C.Chemotherapy

🌹 girl complains of rash in perianal area with pic provided (typically the same) Dx:
molluscum contagiosum
‫ﻛﺰ ﺷ›ﻠﻬﺎ ﻋﻨﺪ•ﻢ‬±”‫ﻢ ﻛﺬا ﺻﻮرة ﻋﺸﺎن ﻳ‬7‫ﺖ ﻟ‬Ç‫ ﺟ‬، ‫ة‬±ä‫ ﺷ›ﻠﻬﺎ ﻣﺜﻞ اﻟﺤﺒﻮب اﻟﺼﻐ‬.
‫ = رﺑط‬molluscum = ‫ ﻛﻠﻣﺔ‬moll ‫ ﻣﻌﻧﺎھﺎ ﺣﺑﯾﺑﺎت ﺻﻐﯾرة و‬contag ‫ اﻟﻣول ﺳﺎر ﻣﻠوث ﺑﺎل‬، ‫ﻣﻌﻧﺎھﺎ ﻣﻠوﺛﺔ‬..

. .. . . . . .
🌹ptient fall on his leg with skin loss vascular structure appear and ask 'bout ttt :
Debritment and 2ry skin graft ✅✅ Debritment 'nd prim'ry closure Abx
‫ ﻣﻌﻧﺎھﺎ اﻟﺟﻠد ﻣﺎھﯾﻘدر ﯾﻘﻔل ﻋﻠﻰ ھذا اﻟﺟرح ﻻن ﺟدا ﻋﻛﯾق ﻓﻼزم ﻧﺎﺧذ‬vascular appear ‫ ﻓﻲ اﻟﺟروح ؟ اذا‬skin graft ‫ﻣﺗﻰ ﻧﻠﺗﺟﺎ اﻟﻰ‬
... . .. .‫وﻧﻐطﻲ ﻓﯾﮭﺎ اﻟﺟرح‬skin graft

🌹Run over case, exposed neurovascular, most appropriate mx?


A.something then primary closure B.debridement + vacuum C. debridement + graft ✅✅
🌹patient with post operetive 'abdominal destintion and small and large bowel
obstruction and no fever 'nd a'bdomen not tender :
Hypo Mg Hypo CL Hypo k

:‫ ﯾﻌﻣل‬k = hypokalemia ‫طﯾب ﻣن اﻋراض اﻧﺧﻔﺎض‬

constipation or abdominal distention, muscle weekness .


. . .. . .
🍓 What's the cause of pain in compartment syndrome?
• Nerve ischemia✅✅ • Muscle enlargement • Bone ischemia

🍓55 pt present to the ER after a car accident, there's a pic of CXR show aortic dissection
and lung contusion , what’s next step :A. chest tube. B. aortic repair and stent✅

🍓 Patient lost sensation over medial side of leg but motor function is intact.
Which nerve is injured?A-Obturator B-Femoral C-Saphenous✅✅ D-Sciatic
🌹lost sensation over = Mid thigh = obturator
🌹lost sensation over = medial side of leg = saphenous

1 Diaphragmatic
🌹Child Diaphragmatic hernia first thing you do? NGT first thing to do after stabilization 🌹
Newborn just delivered having diaphragmatic hernia , what’s the first thing to do: NGT
Diaphragmatic Hernia
Treatment
1_ immediate intubation required at birth: DO NOT bag mask ventilate because air will
enter stomach and further compress lungs.
2_ place large bore orogastric tube to decompress bowel.
3_ initial stabilization and management of pulmonary hypoplasia = hemodynamic support
and surgery when stable.
Clinical Presentation
• respiratory distress, cyanosis
• scaphoid abdomen and barrel-shaped chest
• affected side dull to percussion and breath sounds absent, may hear bowel sounds
instead
• often associated with other anomalies (cardiovascular, CNS, chromosomal abnormalities)
• CXR: bowel loops in thorax (usually left side), displaced mediastinum
.........
Weber in right and bilateral + rinne test?
A-conductive hearing loss B-right schwannoma C- right presbycusis D/ Left sensory ✅
Positive rinne = normal or sensorineural Weber shift to right so it’s either right conductive
or left sensorineural

🍓Colon cancer common metastasis to? A. Liver✅ B. Lung


” “ ‫ﻠﻬﻢ ﻣﻦ اﺟﺰاء اﻟﺠﻬﺎز اﻟﻬﻀ— ﻣﺴﺆوﻟﻴ‬a= ‫|ﺪ‬ž‫ﻂ =اﻟﻘﻮﻟﻮن واﻟ‬¢‫ر‬
‫’ ﻋﻦ اﻻ‹ﻞ‬ ˜

🌹Patient complained of Rt groin pain for 2 weeks and exacerbated by exercise,


physical examination showed normal groin and genitalia, next step?
A. CT abdomen B. MRI abdomen C. Us D. Re-evaluate after 2 wks
‫ﺎ ﺣﺼﻞ ﻟﻠﻌﻀﻠﺔ ﺷﺪ‬Ç‫ﺎﺿﺔ ف ﻏﺎﻟ‬£ª‫ﺪ ﻣﻊ اﻟ‬£À‫ﻗﺎﻟﻚ اﻧﻪ ﻳ‬
overstretching the muscles during sport.t'
‫ﻣﺎ˜ﺤﺘﺎج اﺷﻌﺔ‬

🌹Pt diagnosed with small cell lung cance, presented with dehydration, serum osmo low,
urine osmo high. ‫ ﻫﺬە اﻋﺮاض اﻟﺠﻔﺎف‬. Ttt:
A. 5% dextrose B. Normal saline C. Hypotonic saline

🌹Male have problem initiating urination , bladder fell un empty after , no dribbling or
urge felling , type of incontinence ?A. Overflow B. Reflex C. Urge D. Sterss

🌹 Bradycardia (40 bpm) during laparoscopic procedure Is due to :


A. Cold gas insufflation B. Rapid expansion of parietal peritoneum C. Increase venous return

🌹60 years old patient come with sudden onset of upper abdominal pain after a
few bouts of vomiting. Examination confirme sick patient with tenderness in
epigastrium and supraclavicular subconscious emphysema . What’s Dx ?
A. esophagitis B. acute gastritis C. perforated peptic ulcer D. boerhaav’s syndrome(is
spontaneous perforation of the esophagus )
subconscious emphysema ....‫ﻻن ﻗﺎﻟﻛم‬
🌹Pt k/c of CKD undergo apendecyomy K was 6.5 with tened T wave ??
Calcium gluconate

🌹 Pt with dysuria and cloudy urine with bubbling for 2 months, hx of recurrent
left iliac fossa pain for the past 2 years ! Colonoscopy: no diverticulosis or
polyp Cystoscopy: erythema on dome of the bladder. Most likely diagnosis:
(‫ﻂ‬šà‫ﺎﻟ‬‰ ‫ )ﻛﺬا ﻣﻜﺘﻮب‬disease Diverticular.
B. sq. Cell bladder cancer C. transitional cell bladder cancer

🌹Female with history of regurgitation and heartburn.. all conservative management failed
include PPI ..endoscopy done showed erythema and erosion suggest esophagitis
A. manometry B. 24 h PH monitoring C. Lifestyle modification

🌹45yo female has dysphagia with liquids only retrosternal pain and regurgitation
of food, what is the test of high diagnostic value?
A. UGD. B. . Barium swallow. C. Low esophageal manometry. D. CT with contrast
🌹Patient had a hyperextension trauma, complains of distal phalanx pain and tenderness in
the volar aspect, he also feels tenderness in the palm, what's the dx:
A- Rupture of flexor profundus B- Rupture of flexor superficialis
C- intra-articular fracture of proximal phalanx D- extra-articular fracture of distal phalanx
Memorize them as follow :
P roximal > superficial. D istal > dee

🌹50 male came with 4 time hematochezia , no weight loss , no abdominal pain dx:
A- diverticulosis B- internal piles C- cecum cancer

🌹patient on TPN will increased INR what to give= vit K

🌹Perforated appendicitis post laparotomy, presented with fever, abdominal pain CT


done showed foreign material , reexploration done, gauze was found intraabdominal ،
What to do?
-Issue complaint against assistant
-Call your lawyer and report the incident
-((apologize to the patient and tell her what happened ✅))
……
● Patient with jerky carotid? A. Hypertrophic cardio myopathy
…..
● patient came with melena and they did upper gi endoscopy and found 1cm
bleeding duodenal ulcer and they ask about the duration of ppi ?
A. oral ppi B. iv ppi for 24 hours then convert to oral
C. iv ppi for 72 hours then convert to oral
Answer is C
● Patient post graham patch surgery, what is the most appropriate management?
A. High dose PPI B. IV ppi for 24 hours followed by oral PPI
C. IV ppi for 48 hours followed by oral PPI
…..
6 yrs old, female presented with nausea and recurrent vomiting of every intake , pt
became lethargy with decreased skin turgor and cry with tears.Investigation
revealed Normal investigations apart from :High Na Low glucose
What is the best next management :
A. isotonic saline 20ml/kg B. D10% with 1/2 saline C. ORS
Hypovolemic Hypernatremia First correct the volume with half normal saline then once the
patient euvolemic shift to slow D5W infusion.
….
Old man with slow progression dysphagia, came with vomiting without blood, abd
pain associate with heartburn at eating. Abd ex normal DRE revealed slightly
change stool with blood. What is suspected diagnosis :
A. PUD. B. CA esophagus. C. CA rectum
….
Young female c/o fatigue and jaundice high ALP and high bilirubin Us: no any finding MRCP :
multiple stricture Next step to *confirm* Dx ?
A. Liver biopsy B. Antinuclear antibody C. Repeat US D. Colonoscopy
…..
Case of established diagnosis of biliary sludge of postoperative sleeve pt with jaundice and US
report of biliary sludge. What is the management?
A. Expectant. B. Endoscopic sphincterotomy C. Cholecystectomy
….
What is the Pathophysiology of (case of) bacterial peritonitis due to E. Coli
A. translocation of bacteria B. bowel Perforation
….
young male k/c of ulcerative colitis c/o bloody diarrhea, abd pain and S/s
of intestinal obstruction X-ray shows: megacolon What is appropriate management:
A. CT abdomen B. Ileocolectomy with. Answer is : IV steroids
….
65 male patient present with SOB and generalized fatiguability, On Ex: non tender mass in right
iliac fossa, Hemoglobin: low What the most important step in Mx:
A. colonoscopy ‫اﺷﺗﺑﺎه ب ورم ﻓﻲ اﻟﻘوﻟون‬B. Percutaneous biopsy C. Abdominal CT D. Ultrasound
…..
Surgeon at elective laparoscopic cholecystectomy find large mass in
Stomach= A. stop oper B. cholecystectomy only C. chlesrecomy and resection mass
….
Patient underwent esophageal dilation. After 12h patient had symptoms and they did
gastrografin which showed leak. Temperature is 39. BP 100/50*=
A. Stent B. Esophagectomy C. Surgical drainage!@
(esophageal rupture treat by Tube thoracostomy (drainage with a chest tube or operative
drainage alone)

● neck mass LN FBA normal thyroid cell dx A. Follicular ca C. Papillary ca D. Ectopic

● *Sacral ulcer, skin necrosis and exposed subcutaneous fat manage?*
A. Debridement with dressing B. Debridement with primary closure C. Debridement with
secondary closure graft

*Penetrating stab wound. Patient is stable. What is the next step*
A. US. B. CT
….
● Case of cystic fibrosis Wit abd pain , bloating , pale stool Abd scraled or curve
A. Chronic pancreatitis * B. Acute pancreatitis C. Pseudocyst

Patient claudication long case, then after full treatment with thrombolysis, patient is risk for
what while staying in hospital? A. Stroke B. PE C. MI

Scenario Liver mass 3 cm next with splenomegaly
A. AFP B. triphasic CT * C. Biopsy
..
Epilepsy pt posterior dislocation, which joint? A. Subacromial

Tamoxifen for breast ca the came with abnormal uterine bleeding or something
like that?? A. Endometrial ca B. Endometriosis

Patient has crushed by a car came with 10cm deep laceration of his thigh with
continues bleeding what you will do
A. tourniquet B. compress at the site of the injury C. compress the femoral area
….
Same scenario but with out mentioning of the depth and the patient were
Hypotensive A. normal saline B. compress the site of the laceration C.
Meigs syndrome How to confirm the Dx?
A. Tissue histopathology B. Tumor marker C. Mri D. Or Us I think

Old pt after hysterectomy what may cause her to stay more in the hospital? Her
UO is 35ml per hr = A. Decrease oral fluid intake B. Decrease urine output
N.B :35ml/h = more than 800 per days Oligouria < 500m

Patient has hx of rta since 2 weeks came today with sob the chest x ray has
shown haemothorax you have put him on chest tube and want to transfer him
to higher center while he was in the ambulance the pulse oximetry drop
suddenly what to do
A. continue the journey to the higher center B. check the potency of the chest tube
C. triphasic chest intubation
…..
Sigmoid volvulus case (s&s), ttt: A. total colectomy with ileostomy B. sigmoidoscopy detorsion if
stable
….
Hepatitis B and lesion 6 cm what to do next?
A. TACE Transcatheter arterial chemoembolization) B. ExcisionC. systemic chemo therapy

● 24 years old with history of appendectomy 5 years ago present with abdominal
pain , distintion , vomitting for 3 days Ct scan show obstruction signs And
peritonitis , Whats contraindicated in this patient ?
A. Propofol. B. Ketamine. C. Sevoflorane. D. Nitros Oxide
….
🌹 patient with s&s of bowel obstructions, investigations shows air fluid level and
dilated loop and colon collapse , asking about what drug is C/I?
A. Propofol. B. drug started by *sul* I think. C. nitric oxide


🌹A baby boy came for circumcision in the clinic. The urine was coming from the
mid shaft (I think hypospedias) what to do next?
A. open circumcision B. plastibell circumcision C. circumcision with gomco
D. refer to pediatric surgery

🌹What is associated with Biliary colic?
A. Bilirubin B. Amylase C. Alkaline phosphatase. D*cholecystokinin*

🌹 pt. k/c of crohn's after colon surgery developed severe diarrhea what you will give ?
A. mesalamine. B. cholystaimne
….
🌹 Low grade dysplasia of oesophagus
A. Resection B. Panzole C. Ranitidine
….
🌹Morbid obese for consultation. What is the investigation to determine the
appropriate surgical procedure?
A. US B. CT C. UGI endoscopy D. Barium enema
….
🌹45 YO male underwent sleeve gastrectomy 3 weeks ago , came
complaining of severe abdominal pain, What the next management ?
A. endoscopy. B. NPO. C. exploration. D. forget the choice but something irrelevant
🌹Patient post-surgery came for follow up develop seroma near the wound or superficial
can't remember ( no pain, no erythema, no any sign of inflammation) what your
management is: A. percutaneous drainage. B. daily dressing

🌹55 year old with constipation and distention, on examination there is , CXR
pic with coffee bean appearance, where is the site of obstruction?
A. Ceccal B. Sigmoid

🌹Old pt male presents with sudden severe central abdominal pain he is kc of
cardiomyopathy his vital I think all is normal x -ray normal what is Dx ?
A. mesenteric vascular occlusive B. aortic dissection C. pancreases
..
🌹Euvolemic hyponatremia fluid replacement:
A. Normal saline B. Half normal saline C. Hypertonic saline D- Water Restriction

🌹Post colectomy for colon cancer she is Diabetic, received dextrose and
Insulin for 2 days, then she developed confusion and agitation. Lab :
hypoNA hypoK, urine osmolality normal, serum 270. Most likely cause?
A-Water overload. B-Addison disease. C-SIADH

🌹What of the following pass through deep inguinal ring:
A- Round ligament B- illo-inguinal nerve
….
🌹burn patient and resuscitation done, which of the following reflect a good
resuscitation has been achieved?
A-normalization of heart rate B-normalization of blood pressure
C-Urine out of 0.6ml/kg/h D-central venous pressure 12

December1
🌹Pt with hypertriglyceridemia came epigasttic pain radiated to back, hemodynamicaly unstable and other
syx, ct show dilated lobe in leÿt hypockondrium,lap show: lapase and amylase within normal range
A. pancritits (my answer) B. perÿorated ulcer C. lower lobe pneumonia
NB:Why would pancreatitis presents with normal lipase and amylase?
Because enzymes sensitivity can be *reduced by*:
-late presentation. -hypertriglyceridaemia. -chronic alcoholism

🌹Patient is going to undergo lap chole and incidental found AAA size 4.5
WhAt you are going to do?
A- Do surgery follow up by US B- Do surgery follow up by CTA C-Do CTA before surgery
…..
🌹pt with pelvic pain and ct shows low density adrenal mass lipid rich what is the treatment:
A. lap adrenalectomy
….
🌹15 years old pt going to surgery what to do:
A. both assent from pt and consent ÿrom parent.
🌹 surgeon operate on rt kidney and resident think lt kidney:
A. inform surgen
……
🌹 drowsy despeic child victim on chest trauma with trachea shift to left decrease breath sound
hypotension raised JVP what's Dx: A. tension pneumothorax

🌹 post sleeve gastrectomy pt has change bowel habit what's treatment: reassurance

🌹 surgeon inj CBD during surgery and repair what to do: reassure and tell pt
….
🌹 q. agter appendicitis pt reoperated due to missed gause inside pt
abdomen what to do: A. tell and apologize to pt
….
🌹 mid shaft femur # with 30 degree anterior angulation what is treatment:
A. closed reduction and intramedullary ifxation
……
🌹 pt few days post cholecystectomy developed inspiration chest pain, LUQ pain and tenderness
with dull percussion, tem 38 basal crepitation what is the likely Dx:
A. sup-phrenic abscess. B. lower lobe pneumonia.
…..

You might also like